adasap2 reduced pdf

94
American Diabetes Association Self-Assessment Program Multiple Choice Questions Assessment and Educational (Learning) Critiques Components ADA-SAP Module 2 Pharmacological Treatment of Hyperglycemia Cure Care Commitment ® American Diabetes Association® ® © 2008 American Diabetes Association. All rights reserved.

Upload: alarajpoetry

Post on 03-Oct-2014

194 views

Category:

Documents


5 download

TRANSCRIPT

Page 1: ADASAP2 Reduced PDF

American Diabetes AssociationSelf-Assessment Program

Multiple Choice Questions Assessment and Educational (Learning) Critiques Components

ADA-SAP™

� Module 2Pharmacological Treatment of Hyperglycemia

C u r e C a r e C o m m i t m e n t ®

AmericanDiabetesAssociation®®

© 2008 American Diabetes Association. All rights reserved.

Page 2: ADASAP2 Reduced PDF

PEIDeveloped and Published by: Professional Evaluation, Inc.Developing Medical Specialty Board Category 1 CME Programs for Over Three Decades

Professional Evaluation Inc.4 Midland Avenue, Suite 105, Berwyn, PA 19312-1687 | www.proevalinc.com

Page 3: ADASAP2 Reduced PDF

Table of Contents

3 Learning Objectives

3 Accreditation

4 Faculty

5 Disclosure Statement

6 General Instructions

7 Syllabus

15 Multiple Choice Questions (MCQs)

45 Appendix

46 I. Diagnosis of Impaired Glucose Tolerance (IGT) and Impaired Fasting Glucose (IFG)

47 II. Diagnosis of Type 2 Diabetes

48 III. Summary of Recommendations for Adults with Diabetes

53 Educational Critiques

87 Answer Sheet & Program Evaluation Form

American Diabetes AssociationSelf-Assessment Program (ADA-SAPTM)

� Module 2Pharmacological Treatment of Hyperglycemia

Page 4: ADASAP2 Reduced PDF

American Diabetes Association Self-Assessment Program2

Dear Health Care Professional:

The American Diabetes Association (ADA), in cooperation with Professional Evaluation, Inc. (PEI),has developed the American Diabetes Association Self-Assessment Program (ADA-SAP TM) series.Module 2 of ADA-SAPTM series covers the Pharmacological Treatment of Hyperglycemia. Future modules in the ADA-SAPTM series will cover topics such as, combination therapy for type 2 diabetes, insulin use,and management of co-morbidities.

Module 2 of ADA-SAPTM is sponsored by the ADA for up to 5.0 AMA PRA Category 1 Credit(s)TM

for physicians and for continuing education credit(s) for nurses, nurse practitioners, pharmacists, dietitians,and diabetes educators.

The multiple choice questions component of Module 2 of ADA-SAPTM was developed by PEI at the upper level of difficulty similar in type and format to a board certifying examination. It was designed to objectively assess, strengthen and reinforce your knowledge of diabetes and to provide you with an in-depthlearning experience of active engagement. Once you have properly completed Module 2 in it’s entirety (see General Instructions on page 6), your knowledge of the options available for managing patients with advanced type 2 diabetes mellitus should be greatly enhanced.

The ADA looks forward to your participation in this important new program of continuing education,as part of your commitment to lifelong learning.

Sincerely,

Michael H. Davidson M.D., FACC, FACPProgram ChairChief Medical Officer, PEIClinical Professor of Medicine, Director of Preventive CardiologyThe University of Chicago Pritzker School of Medicine

M. Sue Kirkman, M.D.Vice President, Clinical AffairsAmerican Diabetes Association

C u r e C a r e C o m m i t m e n t ®

AmericanDiabetesAssociation®®

Page 5: ADASAP2 Reduced PDF

3American Diabetes Association Self-Assessment Program

Target AudiencePrimary Care Physicians, General Internists, FamilyPhysicians, Doctors of Osteopathy, Registered Nurses, NursePractitioners, Pharmacists, Physician Assistants, Dietitians,and Certified Diabetes Educators.

Learning ObjectivesBy completing this CME/CE enduring materials activity(Module 2), participants will be better able to:� Interpret and utilize ADA Guidelines and

Treatment Algorithms.� Incorporate combination therapy for management

of patients with advanced type 2 diabetes and achieve A1C targets.

� Utilize insulin for management of patients with acute hyperglycemia.

� Manage medication-induced hypoglycemia.� Regulate secondary disorders in patients with severe

hyperglycemia, such as those with chronic renal insufficiency, congestive heart failure, and hepaticdysfunction.

AccreditationPhysicians: The American Diabetes Association (ADA) is accredited by the Accreditation Council for ContinuingMedical Education (ACCME) to provide continuing medicaleducation for physicians. The ADA designates this educationactivity for a maximum of 5 AMA PRA Category 1 Credit(s)™.Physicians should only claim credit commensurate with theextent of their participation in the activity.

Family Physicians: An application for CME credit has beenfiled with the American Academy of Family Physicians.Determination of credit is pending.

International Physicians: The American Medical Association(AMA) has determined that physicians not licensed in the USwho participate in this CME activity are eligible for AMAPRA category 1 credit™.

Doctors of Osteopathy: Category 2 credit will be awarded for formal educational programs that are ACCME-accreditedor AAFP-approved.

Physician Assistants: The American Academy of PhysicianAssistants (AAPA) accepts Category I credit fromAOACCME, Prescribed credit from AAFP, and AMA PRACategory 1 CME credit for the organizations accredited byACCME. ADA is an accredited provider through theACCME.

Nurses: The ADA is approved as a provider of continuingeducation in nursing by the Virginia Nurses Association(VNA) which is accredited as an approver of continuingeducation in nursing by the American Nurses’ CredentialingCenter’s Commission on Accreditation. The ADA is locatedat 1701 North Beauregard Street, Alexandria, VA 22311.VNA Provider Number: 07-03-02. This educational activity is approved by the VNA which is accredited by the AmericanNurses Credentialing Center’s Commission on Accreditationas an approver of Continuing Education in Nursing for amaximum 5.0 VNA Contact Hours. The VNA is located at7113 Three Chopt Road, Suite 204, Richmond, VA 23226.

California Board of Registered Nursing: The ADA is also a provider approved by the California Board of RegisteredNursing, Provider No. CEP-12196. This activity is approvedfor 5.0 contact hours.

Nurse Practitioners: An application will be made to theAmerican Academy of Nurse Practitioners for continuingeducation credit. Determination of credit is pending.

Pharmacists: The ADA is accredited by theAccreditation Council for Pharmacy Education(ACPE) as a provider of continuing pharmacyeducation. This activity provides up to 5.0

contact hours of continuing pharmacy education credit.The ACPE UPN is 239-000-08-006-H01-P. Each pharmacistshould claim only those hours of credit that he/she spent inthe education activity.

Dietitians: The ADA is a Continuing Professional Education (CPE) AccreditedProvider with the Commission on DieteticRegistration (CDR). Registered Dietitians (RDs)and dietetic technicians, registered (DTRs) willreceive 5.0 continuing professional education

units (CPEUs) for completion of this program/material.

Certified Diabetes Educators: To satisfy the requirement for renewal of certification by continuing education for

the National Certification Board for Diabetes Educators(NCBDE), continuing education activities must be diabetesrelated and approved by a provider on the NCBDE List of Approved Providers (www.ncbde.org). NCBDE does not approve continuing education. The ADA is on theNCBDE List of Approved Providers.

The estimated time for completion of this activity is 5 hours. This continuing education activity was planned and produced in accordance with ACCME Standards.

Release Date: October, 2008.Credits valid through October 31, 2010.Certificates will be sent to you via email 6–8 weeks upon receipt of post-test evaluation.

®

Page 6: ADASAP2 Reduced PDF

American Diabetes Association Self-Assessment Program4

Faculty� ChairMichael H. Davidson, M.D., FACC, FACPClinical ProfessorDirector of Preventive Cardiology The University of ChicagoExecutive Medical DirectorRadiant ResearchChicago, IL

� FacultySuma Dronavalli, M.D.Instructor of MedicineThe University of Chicago Pritzker School of MedicineChicago, IL

Ben Gerber, M.D., MPHAssistant Professor of MedicineSection of Health Promotion ResearchUniversity of Illinois at ChicagoChicago, IL

Stanley Schwartz, M.D., FACPClinical Associate Professor of MedicineDirector, Diabetes Disease ManagementUniversity of Pennsylvania Health SystemPhiladelphia, PA

� Chief Editor and PublisherDante S. LaRocca, Ph.D.PresidentProfessional Evaluation, Inc.Ambler, PA

� ReviewersMartha M. Funnell, MS, RN, CDEResearch InvestigatorMichigan Diabetes Research and Training Center Juvenile Diabetes Research Foundation Center for the Study of the Complications in Diabetes

Ann Arbor, MI

M. Sue Kirkman, M.D.Vice President, Clinical AffairsAmerican Diabetes AssociationAlexandria, VA

Russell D. White, M.D., FAFP, FACSM, MACEProfessor of MedicineDirector,Sports Medicine Fellowship ProgramUniversity of MissouriKansas City School of MedicineTruman Medical Center—LakewoodKansas City, MO

Craig Williams, Pharm.D.Associate Professor of PharmacyAssociate Professor of MedicineOSU School of PharmacyOHSU School of MedicinePortland, OR

Page 7: ADASAP2 Reduced PDF

5American Diabetes Association Self-Assessment Program

Disclaimer StatementThe suggestions and information contained in this publi-cation are generally consistent with the Clinical PracticeRecommendations and other policies of the AmericanDiabetes Association, but they do not represent the policyor position of the Association or any of its boards orcommittees. Reasonable steps have been taken to ensurethe accuracy of the information presented. However, theAmerican Diabetes Association cannot ensure the safety or efficacy of any product or service described in thispublication. Individuals are advised to consult a physicianbefore undertaking any diet or exercise program or takingany medication referred to in this publication. Professionalsmust use and apply their own professional judgment,experience and training and should not rely solely on theinformation contained in this publication beforeprescribing any diet, exercise or medication. TheAmerican Diabetes Association—its officers, directors,employees, volunteers, and members—assumes no responsibility or liability for personal or other injury,loss, or damage that may result from the suggestions or information in this publication.

Content Validation Statement � All recommendations involving clinical medicine are

based on evidence accepted within the profession ofmedicine as adequate justification for their indicationsand contraindications in the care of patients; AND/OR

� All scientific research referred to or reported in supportor justification of a patient care recommendationconforms to generally accepted standards of experi-mental design, data collection, and analysis.

Disclosure Statement As a provider of continuing education through theAccreditation Council for Continuing Medical Education,Virginia Nurses Association, Accreditation Council forPharmacy Education, the American PsychologicalAssociation, and the Commission on DieteticRegistration, it is the Association’s policy to ensurebalance, independence, objectivity, and scientific rigor in all of its educational activities. All participating faculty,course directors, and planning committee members arerequired to disclose to the program audience any financialrelationships related to the subject matter of this program.Disclosure information is reviewed in advance in order to manage and resolve any possible conflicts of interest.The intent of this disclosure is not to prevent a planner or presenter from being involved in the activity, but ratherto provide participants with information on which theycan make their own judgments.

DisclosuresMichael H. Davidson, M.D., FACC, FACPAdvisory Board/Board of Directors: Abbott Laboratories,Access Health, Angiogen, AstraZeneca Pharmaceuticals,Atherogenics, Daiichi-Sankyo, Inc., Kinemed,Merck & Co., Inc., Merck/Schering-Plough, OscientPharmaceuticals, Pfizer Laboratories, PreEmptive Meds,Professional Evaluation, Inc., Sonogene, TakedaPharmaceuticals. Consultant: Abbott Laboratories,AstraZeneca Pharmaceuticals, Daiichi-Sankyo, Inc,diaDexus, Inc., Merck & Co., Inc., Merck/Schering-Plough, Pfizer Laboratories, Roche Pharmaceuticals,sanofi-aventis, Synarc, Takeda Pharmaceuticals.Research Support: Abbott Laboratories, AstraZenecaPharmaceuticals, Daiichi-Sankyo, Inc., Merck & Co., Inc.,Merck/Schering-Plough, Pfizer Laboratories, RochePharmaceuticals, and Takeda Pharmaceuticals.Speaker’s Bureau: Abbott Laboratories, AstraZenecaPharmaceuticals, Daiichi-Sankyo, Inc, diaDexus, Inc.,Merck & Co., Inc., Merck/Schering-Plough, OscientPharmaceuticals, Pfizer Laboratories, TakedaPharmaceuticals.

Suma Dronavalli, M.D.Research Support: Novo Nordisk, Inc.

Martha M. Funnell, MS, RN, CDEBoard Member/Advisory Panel: Eli Lilly, HDI,Intuity Medical, Nodensa, Novo Nordisk, Inc.Consultant: sanofi-aventis.

Ben Gerber, M.D., MPHDisclosed no conflict of interest.

M. Sue Kirkman, M.D.Disclosed no conflict of interest.

Dante S. LaRocca, Ph.D.Disclosed no conflict of interest.

Stanley Schwartz, M.D.Board Member/Advisory Panel: CVT, Eli Lilly, TakedaPharmaceuticals. Speaker’s Bureau: Amylin, Eli Lilly,Merck & Co., Inc., sanofi-aventis, Takeda Pharmaceuticals.

Russell D. White, M.D., FAFP, FACSM, MACEConsultant: Novo Nordisk, Inc. Speaker’s Bureau:American Academy of Family Physicians, AmericanDiabetes Association, The National Procedures Institute,University of Missouri-Kansas City.

Craig Williams, Pharm.D.Research Support: Merck & Co., Inc.Speaker’s Bureau: Merck & Co., Inc., Schering-Plough.

Page 8: ADASAP2 Reduced PDF

American Diabetes Association Self-Assessment Program6

General Instructions for Completion of ADA-SAP™

Module 2 of ADA-SAP™ consists of the following components:

� Multiple Choice Questions (MCQs) Assessment ComponentThis consists of both multiple true false questions and one best choice questions. The first 15 questions are multiple true false and require that you respond either T (true) or F (false) to each of the three, four or five lettered options for each item. The remaining 76 are clinical problem-solving MCQs. Here, your task is to select the one lettered option among four or five offered that BEST answers each MCQ.

� AppendixThis is a comprehensive summary of the recommendations of the ADA for diagnosis and management of type 2 diabetes. It is based on peer-reviewed publications and is designed to assist you to complete the Assessment and Educational Components of ADA-SAP™.

� Computer-Scored Answer SheetThis is located on page 88. Please detach it along the perforation. Your selection of all true or false optionsand of the one lettered option that BEST answers each MCQ is to be marked on the answer sheet adjacentto the number for that MCQ. Please refer to the sample for how to mark your answers on the answer sheet.Your completed answer sheet must be returned to Professional Evaluation, Inc. (PEI) in the accompanyingpostage-paid envelope in order to receive CME/CE Credit.

� Educational (Learning) Critiques ComponentIN ORDER TO MAXIMIZE YOUR LEARNING EXPERIENCE IT IS IMPORTANT THATYOU DO NOT LOOK AT THE EDUCATIONAL CRITIQUES COMPONENT UNTIL AFTERYOU HAVE COMPLETED MARKING YOUR ANSWERS FOR THE MCQs ON THE ANSWERSHEET. The critiques component contains detailed explanations for the correct and incorrect answers for the MCQs based on the most current peer-reviewed published information. Once you have read theEducational Critiques, the bibliographic references should be utilized as follow-up study for those MCQswhich you answered incorrectly. The critiques are the teaching and learning component of ADA-SAP™.They are to be used in combination with the MCQ assessment component to provide you with a positive,active learning experience.

DO NOT CHANGE THE ANSWERS MARKED ON YOUR ANSWER SHEET WHILE READINGTHE CRITIQUES. THE INTENT OF ADA-SAP™ IS LEARNING NEW KNOWLEDGE AND REINFORCING PREVIOUSLY LEARNED KNOWLEDGE. THERE IS NO PASS-FAIL SCORE.

� Program Evaluation Form ComponentAfter you have completed the MCQ and Critique components as instructed, turn to the reverse side of your computer-scored answer sheet. Please select the one lettered option that BEST answers eachprogram evaluation question and mark it on the answer sheet.

NoteAfter you have completed the Answer Sheet & Program Evaluation Form it is important that you returnthem to Professional Evaluation, Inc. in the enclosed self-addressed, no postage required envelope,in order to receive CME/CE credit from the ADA for up to 5.0 AMA PRA Category 1 Credit(s)™.PLEASE ALLOW 6–8 WEEKS FOR RECEIPT OF CREDIT FROM THE ADA. Certificates will be sent via email.

Page 9: ADASAP2 Reduced PDF

7Syllabus American Diabetes Association Self-Assessment Program

American Diabetes AssociationSelf-Assessment ProgramSyllabus

Page 10: ADASAP2 Reduced PDF

American Diabetes Association Self-Assessment Program Syllabus8

A1C� Perform the A1C test at least two times a year in

patients who are meeting treatment goals (and whohave stable glycemic control). (E)

� Perform the A1C test quarterly in patients whosetherapy has changed or who are not meetingglycemic goals. (E)

� Use of point-of-care testing for A1C allows for timelydecisions on therapy changes, when needed. (E)

Glycemic Goals� Lowering A1C to an average of ~7% has clearly

been shown to reduce microvascular and neuro-pathic complications of diabetes and, possibly,macrovascular disease. Therefore, the A1C goal for nonpregnant adults in general is �7%. (A)

� Epidemiologic studies have suggested an incre-mental (albeit, in absolute terms, a small) benefit to lowering A1C from 7% into the normal range.Therefore, the A1C goal for selected individualpatients is as close to normal (�6%) as possiblewithout significant hypoglycemia. (B)

� Less stringent A1C goals may be appropriate forpatients with a history of severe hypoglycemia,patients with limited life expectancies, children,individuals with comorbid conditions, and thosewith longstanding diabetes and minimal or stablemicrovascular complications. (E)

Medical Nutrition Therapy (MNT)

General Recommendations� Individuals who have pre-diabetes or diabetes

should receive individualized MNT as needed toachieve treatment goals, preferably provided by aregistered dietitian familiar with the components of diabetes MNT. (B)

� MNT should be covered by insurance and otherpayors. (E)

Energy Balance, Overweight, and Obesity� In overweight and obese insulin resistant individuals,

modest weight loss has been shown to reduce insulinresistance. Thus, weight loss is recommended for alloverweight or obese individuals who have or are at risk for diabetes. (A)

� For weight loss, either low-carbohydrate or low-fatcalorie-restricted diets may be effective in the shortterm (up to 1 year). (A)

� For patients on low-carbohydrate diets, monitor lipidprofiles, renal function and protein intake (in thosewith nephropathy), and adjust hypoglycemic therapyas needed. (E)

� Physical activity and behavior modification areimportant components of weight loss programs andare most helpful in maintenance of weight loss. (B)

Primary Prevention of Diabetes� Among individuals at high risk for developing type 2

diabetes, structured programs that emphasizelifestyle changes that include moderate weight loss(7% body weight) and regular physical activity (150 min/week), with dietary strategies includingreduced calories and reduced intake of dietary fat,can reduce the risk for developing diabetes and aretherefore recommended. (A)

An Overview of Standards of Medical Care in Diabetes — 2008These standards of care are intended to provide clinicians, patients, researchers, payors, and other interestedindividuals with the components of diabetes care, treatment goals, and tools to evaluate the quality of care.While individual preferences, comorbidities, and other patient factors may require modification of goals, targetsthat are desirable for most patients with diabetes are provided. These standards are not intended to preclude more extensive evaluation and management of the patient by other specialists as needed.

The recommendations included are screening, diagnostic, and therapeutic actions that are known or believed tofavorably affect health outcomes of patients with diabetes. A grading system developed by the American DiabetesAssociation and modeled after existing methods was utilized to clarify and codify the evidence that forms the basisfor the recommendations. The level of evidence that supports each recommendation is listed after each recommen-dation using the letters A, B, C, or E.

For more detailed information, refer to the full document: “Standards of Medical Care in Diabetes—2008.”

Page 11: ADASAP2 Reduced PDF

9Syllabus American Diabetes Association Self-Assessment Program

� Individuals at high risk for type 2 diabetes should beencouraged to achieve the U.S. Department ofAgriculture (USDA) recommendation for dietaryfiber (14 g fiber/1000 kcal) and foods containingwhole grains (one-half of grain intake). (B)

Dietary Fat Intake in Diabetes Management� Saturated fat intake should be �7% of total

calories. (A)

� Intake of trans fat should be minimized. (E)

Carbohydrate Intake in Diabetes Management� Monitoring carbohydrate, whether by carbohydrate

counting, exchanges, or experience-basedestimation, remains a key strategy in achievingglycemic control. (A)

� For individuals with diabetes, the use of the glycemicindex and glycemic load may provide a modestadditional benefit for glycemic control over thatobserved when total carbohydrate is consideredalone. (B)

Other Nutrition Recommendations� Sugar alcohols and nonnutritive sweeteners are safe

when consumed within the acceptable daily intakelevels established by the Food and Drug Adminis-tration (FDA). (A)

� If adults with diabetes choose to use alcohol, dailyintake should be limited to a moderate amount (one drink/day or less for adult women and twodrinks/day or less for adult men). (E)

� Routine supplementation with antioxidants, such as vitamins E and C and carotene, is not advisedbecause of lack of evidence of efficacy and concernrelated to long-term safety. (A)

� Benefit from chromium supplementation in peoplewith diabetes or obesity has not been conclusivelydemonstrated and, therefore, cannot be recom-mended. (E)

Diabetes Self-Management Education (DSME)� People with diabetes should receive DSME

according to national standards when their diabetesis diagnosed and as needed thereafter. (B)

� Self-management behavior change is the keyoutcome of DSME and should be measured andmonitored as part of care. (E)

� DSME should address psychosocial issues, sinceemotional well-being is strongly associated withpositive diabetes outcomes. (C)

� DSME should be reimbursed by third party payors. (E)

Physical Activity� People with diabetes should be advised to perform

at least 150 min/week of moderate-intensity aerobic physical activity (50–70% of maximum heart rate). (A)

� In the absence of contraindications, people with type 2 diabetes should be encouraged to performresistance training three times per week. (A)

Psychosocial Assessment and Care� Assessment of psychological and social situation

should be included as an ongoing part of the medicalmanagement of diabetes. (E)

� Psychosocial screening and follow-up shouldinclude, but is not limited to, attitudes about theillness, expectations for medical management andoutcomes, affect/mood, general and diabetes-relatedquality of life, resources (financial, social, andemotional), and psychiatric history. (E)

� Screen for psychosocial problems such asdepression, anxiety, eating disorders, and cognitiveimpairment when adherence to the medical regimenis poor. (E)

Hypoglycemia� Glucose (15–20 g) is the preferred treatment for the

conscious individual with hypoglycemia, althoughany form of carbohydrate that contains glucose may be used. If self-monitoring of blood glucose(SMBG) 15 min after treatment shows continuedhypoglycemia, the treatment should be repeated.Once SMBG glucose returns to normal, theindividual should consume a meal or snack toprevent recurrence of hypoglycemia. (E)

� Glucagon should be prescribed for all individuals at significant risk of severe hypoglycemia, andcaregivers or family members of these individualsshould be instructed in its administration. Glucagonadministration is not limited to health care profes-sionals. (E)

� Individuals with hypoglycemia unawareness or oneor more episodes of severe hypoglycemia should beadvised to raise their glycemic targets to strictly avoidfurther hypoglycemia for at least several weeks inorder to partially reverse hypoglycemia unawarenessand reduce risk of future episodes. (B)

Page 12: ADASAP2 Reduced PDF

American Diabetes Association Self-Assessment Program Syllabus10

Immunization� Annually provide an influenza vaccine to all diabetic

patients �6 months of age. (C)

� Provide at least one lifetime pneumococcal vaccinefor adults with diabetes. A one-time revaccination isrecommended for individuals �65 years of age previ-ously immunized when they were �65 years of age ifthe vaccine was administered �5 years ago. Otherindications for repeat vaccination include nephroticsyndrome, chronic renal disease, and other immuno-compromised states, such as after transplantation. (C)

Hypertension/Blood Pressure Control

Screening and Diagnosis� Blood pressure should be measured at every

routine diabetes visit. Patients found to have systolicblood pressure �130 mm Hg or diastolic bloodpressure �80 mm Hg should have blood pressureconfirmed on a separate day. Repeat systolic bloodpressure �130 mm Hg or diastolic blood pressure�80 mm Hg confirms a diagnosis of hypertension.(C)

Goals� Patients with diabetes should be treated to a systolic

blood pressure �130 mm Hg. (C)

� Patients with diabetes should be treated to a diastolicblood pressure �80 mm Hg. (B)

Treatment� Patients with a systolic blood pressure of

130–139 mm Hg or a diastolic blood pressure of80–89 mm Hg may be given lifestyle therapy alonefor a maximum of 3 months, and then, if targets arenot achieved, be treated with addition of pharmaco-logical agents. (E)

� Patients with more severe hypertension (systolicblood pressure �140 or diastolic blood pressure�90 mm Hg) at diagnosis or follow-up shouldreceive pharmacologic therapy in addition to lifestyletherapy. (A)

� Pharmacologic therapy for patients with diabetesand hypertension should be with a regimen thatincludes either an ACE inhibitor or an angiotensinreceptor blocker (ARB). If one class is not tolerated,the other should be substituted. If needed to achieveblood pressure targets, a thiazide diuretic should beadded to those with an estimated glomerularfiltration rate (GFR) �50 ml/min per 1.73 m2 and a loop diuretic for those with an estimated GFR �50 ml/min per 1.73 m2. (E)

� Multiple drug therapy (two or more agents atmaximal doses) is generally required to achieveblood pressure targets. (B)

� If ACE inhibitors, ARBs, or diuretics are used,kidney function and serum potassium levels shouldbe closely monitored. (E)

� In pregnant patients with diabetes and chronichypertension, blood pressure target goals of110–129/65–79 mm Hg are suggested in the interestof long-term maternal health and minimizingimpaired fetal growth. ACE inhibitors and ARBs are contraindicated during pregnancy. (E)

Dyslipidemia/Lipid Management

Screening� In most adult patients, measure fasting lipid

profile at least annually. In adults with low-risk lipidvalues (LDL cholesterol �100 mg/dL, HDL-C �50 mg/dL, and triglycerides �150 mg/dL), lipidassessments may be repeated every 2 years. (E)

Treatment Recommendations and Goals� Lifestyle modification focusing on the reduction of

saturated fat, trans fat, and cholesterol intake; weightloss (if indicated); and increased physical activityshould be recommended to improve the lipid profilein patients with diabetes. (A)

� Statin therapy should be added to lifestyle therapy,regardless of baseline lipid levels, for diabeticpatients: � With overt cardiovascular disease (CVD). (A)� Without CVD who are over the age of 40 and

have one or more other CVD risk factors. (A)

� For patients at lower risk than those mentionedabove (e.g., without overt CVD and under the age of40), statin therapy should be considered in additionto lifestyle therapy if LDL-C remains �100 mg/dLor in those with multiple CVD risk factors. (E)

� In individuals without overt CVD, the primary goalis an LDL-C �100 mg/dL (2.6 µmol/l). (A)

� In individuals with overt CVD, a lower LDL-C goalof �70 mg/dL (1.8 µmol/l), using a high dose of astatin, is an option. (E)

� If drug-treated patients do not reach the abovetargets on maximal tolerated statin therapy, areduction in LDL-C of ~40% from baseline is an alternative therapeutic goal. (A)

Page 13: ADASAP2 Reduced PDF

11Syllabus American Diabetes Association Self-Assessment Program

� Triglyceride levels �150 mg/dL (1.7 µmol/l) andHDL cholesterol levels �40 mg/dL (1.0 µmol/l) inmen and �50 mg/dL (1.3 µmol/l) in women aredesirable. However, LDL-C–targeted statin therapyremains the preferred strategy. (C)

� Combination therapy using statins and other lipid-lowering agents may be considered to achieve lipidtargets but has not been evaluated in outcomestudies for either CVD outcomes or safety. (E)

� Statin therapy is contraindicated in pregnancy. (E)

Antiplatelet Agents� Use aspirin therapy (75–162 mg/day) as a secondary

prevention strategy in diabetic individuals with ahistory of CVD. (A)

� Use aspirin therapy (75–162 mg/day) as a primaryprevention strategy in those with type 1 or type 2diabetes at increased cardiovascular risk, includingthose who are �40 years of age or who haveadditional risk factors (family history of CVD, hyper-tension, smoking, dyslipidemia, or albuminuria). (A)

� Aspirin therapy is not recommended in peopleunder 30 years of age, due to lack of evidence ofbenefit, and is contraindicated in patients under theage of 21 years because of the associated risk of Reye syndrome. (E)

� Combination therapy using other antiplatelet agentssuch as clopidrogel in addition to aspirin should be used in patients with severe and progressiveCVD. (C)

� Other antiplatelet agents may be a reasonable alter-native for high-risk patients with aspirin allergy, withbleeding tendency, who are receiving anticoagulanttherapy, with recent gastrointestinal bleeding, andwith clinically active hepatic disease who are notcandidates for aspirin therapy. (E)

Smoking Cessation� Advise all patients not to smoke. (A)

� Include smoking cessation counseling and otherforms of treatment as a routine component ofdiabetes care. (B)

Coronary Heart Disease (CHD) Screening and Treatment

Screening� In asymptomatic patients, evaluate risk factors to

stratify patients by 10-year risk, and treat risk factorsaccordingly. (B)

Treatment� In patients with known CVD, ACE inhibitor, aspirin,

and statin therapy (if not contraindicated) should beused to reduce the risk of cardiovascular events. (A)

� In patients with a prior myocardial infarction, add β-blockers (if not contraindicated) to reducemortality. (A)

� In patients �40 years of age with another cardio-vascular risk factor (hypertension, family history,dyslipidemia, microalbuminuria, cardiac autonomicneuropathy, or smoking), ACE inhibitor, aspirin, andstatin therapy (if not contraindicated) should be usedto reduce the risk of cardiovascular events. (B)

� In patients with treated congestive heart failure(CHF), metformin and thiazolidinedione (TZD) use are contraindicated. (C)

Nephropathy Screening and Treatment

General Recommendations� To reduce the risk or slow the progression of

nephropathy, optimize glucose control. (A)

� To reduce the risk or slow the progression ofnephropathy, optimize blood pressure control. (A)

Screening� Perform an annual test to assess urine albumin

excretion in type 1 diabetic patients with diabetesduration of �5 years and in all type 2 diabeticpatients, starting at diagnosis. (E)

� Measure serum creatinine at least annually in alladults with diabetes regardless of the degree of urinealbumin excretion. The serum creatinine should beused to estimate GFR and stage the level of chronickidney disease (CKD), if present. (E)

Page 14: ADASAP2 Reduced PDF

American Diabetes Association Self-Assessment Program Syllabus12

Treatment� In the treatment of the nonpregnant patient with

micro- or macroalbuminuria, either ACE inhibitorsor ARBs should be used. (A)

� While there are no adequate head-to-head compar-isons of ACE inhibitors and ARBs, there is clinicaltrial support for each of the following statements:

� In patients with type 1 diabetes, with hyper-tension and any degree of albuminuria, ACEinhibitors have been shown to delay theprogression of nephropathy. (A)

� In patients with type 2 diabetes, hypertension,and microalbuminuria, both ACE inhibitors andARBs have been shown to delay the progressionto macroalbuminuria. (A)

� In patients with type 2 diabetes, hypertension,macroalbuminuria, and renal insufficiency (serumcreatinine �1.5 mg/dL), ARBs have been shownto delay the progression of nephropathy. (A)

� If one class is not tolerated, the other should besubstituted. (E)

� Reduction of protein intake to 0.8–1.0 g • kg bodywt -1 • day -1 in individuals with diabetes and theearlier stages of chronic kidney disease (CKD) and to 0.8 g • kg body wt -1 • day -1 in the later stages of CKD may improve measures of renal function(e.g., urine albumin excretion rate and GFR) and is recommended. (B)

� When ACE inhibitors, ARBs, or diuretics are used, monitor serum creatinine and potassium levelsfor the development of acute kidney disease andhyperkalemia. (E)

� Continued monitoring of urine albumin excretion to assess both the response to therapy and theprogression of disease is recommended. (E)

� Consider referral to a physician experienced in thecare of kidney disease when there is uncertaintyabout the etiology of kidney disease (active urinesediment, absence of retinopathy, rapid decline inGFR), difficult management issues, or advancedkidney disease. (B)

Retinopathy Screening and Treatment

General Recommendations� To reduce the risk or slow the progression of

retinopathy, optimize glycemic control. (A)

� To reduce the risk or slow the progression ofretinopathy, optimize blood pressure control. (A)

Screening� Adults and adolescents with type 1 diabetes should

have an initial dilated and comprehensive eye exami-nation by an ophthalmologist or optometrist within 5 years after the onset of diabetes. (B)

� Patients with type 2 diabetes should have an initialdilated and comprehensive eye examination by anophthalmologist or optometrist shortly after thediagnosis of diabetes. (B)

� Subsequent examinations for type 1 and type 2diabetic patients should be repeated annually by anophthalmologist or optometrist. Less frequent exams(every 2–3 years) may be considered following oneor more normal eye exams. Examinations will berequired more frequently if retinopathy isprogressing. (B)

� Women with preexisting diabetes who are planning pregnancy or who have become pregnantshould have a comprehensive eye examination andbe counseled on the risk of development and/orprogression of diabetic retinopathy. Eye examinationshould occur in the first trimester with close follow-up throughout pregnancy and for 1 yearpostpartum. (B)

Treatment� Promptly refer patients with any level of macular

edema, severe nonproliferative diabetic retinopathy(NPDR), or any proliferative diabetic retinopathy(PDR) to an ophthalmologist who is knowledgeableand experienced in the management and treatment of diabetic retinopathy. (A)

� Laser photocoagulation therapy is indicated toreduce the risk of vision loss in patients with high-risk PDR, clinically significant macular edema, andin some cases of severe NPDR. (A)

� The presence of retinopathy is not a contraindi-cation to aspirin therapy for cardioprotection, as this therapy does not increase the risk of retinalhemorrhage. (A)

Page 15: ADASAP2 Reduced PDF

13Syllabus American Diabetes Association Self-Assessment Program

Neuropathy Screening and Treatment� All patients should be screened for distal symmetric

polyneuropathy (DPN) at diagnosis and at leastannually thereafter, using simple clinical tests. (B)

� Electrophysiological testing is rarely needed,except in situations where the clinical features are atypical. (E)

� Educate all patients about self-care of the feet.For those with DPN, facilitate enhanced foot careeducation and refer for special footware. (B)

� Screening for signs and symptoms of autonomicneuropathy should be instituted at diagnosis of type2 diabetes and 5 years after the diagnosis of type 1diabetes. Special testing is rarely needed and may not affect management or outcomes. (E)

� Medications for the relief of specific symptomsrelated to DPN and autonomic neuropathy arerecommended, as they improve the quality of life of the patient. (E)

Foot Care� For all patients with diabetes, perform an annual

comprehensive foot examination to identify riskfactors predictive of ulcers and amputations.The foot examination can be accomplished in aprimary care setting and should include the use of a monofilament, tuning fork, palpation, and a visual examination. (B)

� Provide general foot self-care education to allpatients with diabetes. (B)

� A multidisciplinary approach is recommended for individuals with foot ulcers and high-risk feet,especially those with a history of prior ulcer oramputation. (B)

� Refer patients who smoke, have loss of protectivesensation and structural abnormalities, or havehistory of prior lower-extremity complications tofoot care specialists for ongoing preventive care andlife-long surveillance. (C)

� Initial screening for peripheral arterial disease (PAD)should include a history for claudication and anassessment of the pedal pulses. Consider obtainingan ankle-brachial index (ABI), as many patients withPAD are asymptomatic. (C)

� Refer patients with significant claudication or a positive ABI for further vascular assessment and consider exercise, medications, and surgicaloptions. (C)

Children and Adolescents

Glycemic Control� Consider age when setting glycemic goals in children

and adolescents with type 1 diabetes, with lessstringent goals for younger children. (E)

Nephropathy� Annual screening for microalbuminuria, with a

random spot urine sample for microalbumin-to-creatinine ratio, should be initiated once the child is10 years of age and has had diabetes for 5 years. (E)

� Confirmed, persistently elevated microalbuminlevels on two additional urine specimens should betreated with an ACE inhibitor, titrated to normal-ization of microalbumin excretion if possible. (E)

Hypertension� Treatment of high-normal blood pressure (systolic

or diastolic blood pressure consistently above the90th percentile for age, sex, and height) shouldinclude dietary intervention and exercise, aimed atweight control and increased physical activity ifappropriate. If target blood pressure is not reachedwith 3–6 months of lifestyle intervention, pharmaco-logic treatment should be initiated. (E)

� Pharmacologic treatment of hypertension (systolic ordiastolic blood pressure consistently above the 95thpercentile for age, sex, and height or consistently�130/80 mm Hg, if 95% exceeds that value) shouldbe initiated as soon as the diagnosis is confirmed. (E)

� ACE inhibitors should be considered for the initialtreatment of hypertension. (E)

Dyslipidemia

Screening

� If there is a family history of hypercholesterolemia(total cholesterol �240 mg/dL) or a cardiovascularevent before age 55 years, or if family history isunknown, then a fasting lipid profile should beperformed on children �2 years of age soon afterdiagnosis (after glucose control has been estab-lished). If family history is not of concern, then thefirst lipid screening should be performed at puberty(�10 years). All children diagnosed with diabetes ator after puberty should have a fasting lipid profileperformed soon after diagnosis (after glucose controlhas been established). (E)

Page 16: ADASAP2 Reduced PDF

American Diabetes Association Self-Assessment Program Syllabus14

� For both age groups, if lipids are abnormal, annualmonitoring is recommended. If LDL-C values are within the accepted risk levels (�100 mg/dL [2.6 µmol/l]), a lipid profile should be repeatedevery 5 years. (E)

Treatment

� Initial therapy should consist of optimization ofglucose control and MNT using a Step 2 AmericanHeart Association diet aimed at a decrease in theamount of saturated fat in the diet. (E)

� After the age of 10 years, the addition of a statin isrecommended in patients who, after MNT andlifestyle changes, have LDL cholesterol �160 mg/dL(4.1 µmol/l) or LDL cholesterol �130 mg/dL (3.4 µmol/l) and one or more CVD risk factors. (E)

� The goal of therapy is an LDL cholesterol value�100 mg/dL (2.6 µmol/l). (E)

Retinopathy� The first ophthalmologic examination should be

obtained once the child is 10 years of age and hashad diabetes for 3–5 years. (E)

� After the initial examination, annual routine follow-up is generally recommended. Less frequentexaminations may be acceptable on the advice of an eye care professional. (E)

Page 17: ADASAP2 Reduced PDF

American Diabetes AssociationSelf-Assessment ProgramMultiple Choice Questions (MCQs)

15MCQs American Diabetes Association Self-Assessment Program

Page 18: ADASAP2 Reduced PDF

16 American Diabetes Association Self-Assessment Program MCQs

Section A:Multiple True False Questions

DirectionsFor items 1–15 only please answer TRUE or FALSE to each of the four or five lettered options in each item. In a given item all, some, or none of the lettered options may be true.For each option that you think is TRUE in each item, completely blacken the correspondinglettered circle in the column labeled T for that item on Section A of the answer sheet.For each option that you think is FALSE, completely blacken the corresponding lettered circle in the column labeled F.

� Items 1–15

A 52-year-old African American woman with type 2 diabetes mellitus for 3 years presents to you for the first timefor evaluation and treatment. Medications are metformin 1000 mg b.i.d.; pioglitazone 30 mg/qd; and glimepiride 4 mg qd. She has not seen a physician in over 12 months. She recently changed employment with a new health plan in which her former primary care physician is no longer in the network. She is married with 4 grown childrenand 6 grandchildren. She drinks 1–2 glasses of wine on social occasions and quit smoking 1 year ago, with aprevious 40 packs/year history (2 packs/day from age 22 to 51). Both her parents are deceased, mother at age 63 of a stroke and father at age 70 of congestive heart failure. Two siblings, a brother age 56 and sister age 58, have type2 diabetes mellitus. Her only complaint is occasional lightheadedness, especially after eating meals.

Vitals:

Significant physical findings:

Fundoscopic Exam: dot hemorrhages

Heart: positive for S4

Neuro: decreased monofilament sensation in both feet

Heart Rate 80

Blood Pressure 152/86 mm Hg

Respiratory Rate 14

Height 5’2”

Weight 210 lbs.

A AT F

B B

C C

D D

E E

1

Page 19: ADASAP2 Reduced PDF

17MCQs American Diabetes Association Self-Assessment Program

Lab results:

1. Which of the following statements regarding A1C testing and goals is/are TRUE based on ADA guidelines?

(A) Perform the A1C test at least 2 times a year in patients who are meeting treatment goals and who havestable glycemic control.

(B) The A1C goal for patients in general is �6.5%.

(C) The goal for the selected individual patient is an A1C as close to normal (�6%) as possible without significant hypoglycemia.

(D) Perform the A1C test bimonthly in patients whose therapy has changed or who are not meeting glycemia goals.

(E) Postprandial glucose levels should be checked in patients with A1C �8.0%.

2. Which of the following additional test(s) or referrals in this patient is/are recommended according to ADAguidelines?

(A) Dilated fundoscopic exam.

(B) Urine albumin measurement.

(C) Foot exam by podiatrist.

(D) Liver ultrasound for hepatic fat.

(E) Referral for medical nutrition therapy.

Total Cholesterol 180 mg/dL

HDL-C 35 mg/dL

LDL-C 95 mg/dL

Triglycerides 300 mg/dL

A1C 8.5%

Fasting Glucose 150 mg/dL

TSH 1.0 U/L

ALT 56 U/L (normal �40)

AST 40 U/L (normal �30)

Creatinine 1.8 mg/dL

Glomerular 45 mL/min/1.73 m2

Filtratration Rate (GFR) body sensitive area

Page 20: ADASAP2 Reduced PDF

American Diabetes Association Self-Assessment Program MCQs18

3. In regard to nutritional counseling, which of the following statements is/are TRUE?

(A) Monitoring carbohydrates whether by carbohydrate counting, exchange, or experience-basedestimation, remains a key strategy in achieving glycemic control.

(B) Sugar alcohols and other non-nutritive sweeteners should be discouraged.

(C) Routine supplementation with vitamin E 400 IU is recommended for patients with type 2 diabetesmellitus with at least one other CVD risk factor.

(D) Chromium supplementation for patients with diabetes or obesity is not recommended.

(E) Saturated fat intake should be �7% of total calories.

4. Which of the following additional treatments is/are recommended for this patient according to ADA guidelines?

(A) Statins to achieve at least a 40% reduction in LDL-C level.

(B) An ACE inhibitor or ARB.

(C) Aspirin 81 mg.

(D) Influenza vaccine.

(E) Pneumococcal vaccine.

5. Which of the following is/are her recommended goal(s) or desirable level(s) with treatment?

(A) A1C �7.0%.

(B) Blood pressure �130/80 mm Hg.

(C) LDL �100 mg/dL.

(D) Triglycerides �150 mg/dL.

(E) HDL �40 mg/dL.

6. Which of the following modifications of treatment is/are appropriate to improve her A1C levels?

(A) Continue her present medications and initiate basal insulin.

(B) Discontinue the metformin and increase the doses of pioglitazone and glimepiride.

(C) Discontinue metformin and add sitagliptin.

(D) Discontinue metformin and add exenatide.

(E) Discontinue metformin, pioglitazone, and glimepiride and initiate basal insulin.

7. She has been placed on a statin and continues to have elevated triglycerides (TGs) and low HDL-C level.Which of the following potential drug interactions or safety concerns regarding fibrate therapy is/are TRUE?

(A) The dose of simvastatin should not exceed 10 mg if used in conjunction with gemfibrozil.

(B) Gemfibrozil will increase the AUC (area under the curve) and, therefore, increase the hypoglycemiceffect of repaglimide and rosiglitazone.

(C) In conjunction with fenofibrate, the dose of simvastatin should not exceed 20 mg.

(D) Fenofibrate increases both creatinine and homocysteine levels.

(E) Fenofibrate does not require a dosage adjustment in patients with severe renal impairment.

Page 21: ADASAP2 Reduced PDF

19MCQs American Diabetes Association Self-Assessment Program

8. Which of the following statements is/are TRUE in regard to her hypertension and renal impairment?

(A) In patients with type 2 diabetes mellitus, hypertension, and microalbuminurea, both ACE inhibitors and ARBs have been shown to delay the progression of nephropathy.

(B) A combination of ACE and ARB has been shown to reduce CVD events more than either treatment alone.

(C) In patients with a GFR �50 mL/min per 1.73 m2, a loop diuretic should be used rather than a thiazide diuretic to improve blood pressure control.

(D) Reduction in protein intake to 0.8–1.0 g per kg body weight per 1 day in individuals with diabetes and earlier stages of CKD is recommended.

(E) Once urine albumin is found to be increased, continued monitoring to assess the progression of diseaseis not necessary.

9. In regard to her retinopathy, which of the following statements is/are TRUE?

(A) Patients with type 2 diabetes mellitus should have an initial dilated and comprehensive eye exam by an ophthalmologist or optometrist shortly after the diagnosis of diabetes and annually thereafter.

(B) Optimizing both glycemic control and blood pressure reduces the risk or slows the progression of retinopathy.

(C) Aspirin therapy is contraindicated if retinopathy is present due to the increased risk of retinal hemorrhage.

(D) Laser photocoagulation therapy is indicated to reduce vision loss in patients with severe non-prolifer-ative diabetic retinopathy (NPDR) but not in patients with proliferative diabetic retinopathy (PDR).

(E) At present, there is a lack of evidence that photocoagulation surgery decreases the risk for severe visual loss.

10. Which of the following statements is/are TRUE regarding her neuropathy treatment and foot care?

(A) Patients with type 2 diabetes mellitus should have electrophysiological testing performed at diagnosisand biannually thereafter.

(B) Medications for the relief of specific symptoms related to distal symmetric polyneuropathy are ineffectiveand, therefore, not recommended.

(C) An ankle-brachial index (ABI) is not recommended unless the patients with type 2 diabetes mellitus havesymptoms of claudication.

(D) Referral to a foot care specialist is recommended if the patient with type 2 diabetes mellitus smokes, hasloss of protective sensation, or has a history of prior lower extremity complications.

(E) Evaluation of bladder dysfunction should be considered for patients with diabetes who have recurrenturinary tract infections, pyelonephrotic incontinence, or palpable bladder.

Page 22: ADASAP2 Reduced PDF

American Diabetes Association Self-Assessment Program MCQs20

� Items 11–12

This patient’s managed care plan has just instituted “pay for performance” for achievement of the HEDIS 2008NCQA guidelines. Which of the following is/are utilized to assess the quality of comprehensive care in adultpatients with diabetes?

11. (A) A1C �7.0%.

(B) LDL-C �100 mg/dL.

(C) TGs �150 mg/dL.

(D) Medical attention for nephropathy.

(E) Foot examination.

12. (A) Eye exam (retinal) performed.

(B) Blood pressure �130/80 mm Hg.

(C) Smoking status and cessation advice or treatment.

(D) Influenza vaccination.

(E) HDL �40 mg/dL.

� Items 13–15

Based on an ADA consensus group algorithm in regard to her treatment, which of the following statements is/are TRUE for each of the categories of pharmacological therapies?

13. For the use of insulin:

(A) Advantages: most effective, relatively inexpensive, and improved lipid profile.

(B) Disadvantages: injections, monitoring, hypoglycemia, and weight gain.

(C) Insulin therapy has beneficial effects on peripheral insulin sensitivity.

(D) Insulin plus glitazones may increase the risk of fluid retention.

(E) Insulin should only be used third line if 2 oral agents, including metformin, have failed to achieve the A1C target.

14. For the use of thiazolidinediones:

(A) Advantages: pioglitazone increases HDL-C and lowers triglycerides.

(B) Disadvantages: two-fold increased risk of CHF and weight gain.

(C) Black-box warning for both rosiglitazone and for pioglitazone for potential increased risk of MI.

(D) Increased risk for fractures.

(E) Dosage adjustment for patients with severe renal impairment.

Page 23: ADASAP2 Reduced PDF

21MCQs American Diabetes Association Self-Assessment Program

15. For the use of sulfonyluryea:

(A) Advantage: inexpensive.

(B) Disadvantages: increased risk of hypoglycemia and weight gain.

(C) No dosage adjustment required in patients with severe renal impairment.

(D) Increases β-cell survival.

(E) Proven to reduce cardiovascular events.

Page 24: ADASAP2 Reduced PDF

22 American Diabetes Association Self-Assessment Program MCQs

Section B:One Best Choice Questions

DirectionsItems 16–91 consist of a question or an incomplete statement followed by three, four or five lettered options.Your task is to select the one BEST lettered option that answers each item. After you have selected the onelettered option that BEST answers each item, completely blacken the corresponding lettered circle for that item on Section B of the answer sheet. Please use only a #2 soft lead pencil.

16. A 68-year-old man with a history of type 2 diabetes mellitus is admitted to the hospital with increaseddyspnea, extremity edema, and a 10 lb. weight gain. He has a history of coronary artery disease and experi-enced a myocardial infarction four years ago. During the hospitalization, there were no changes noted in EKGs, and CPK-mb and troponin were negative. He was treated with lasix with substantial improvement.A stress test showed a fixed inferior defect and a reversible anterolateral defect, with estimated ejection fractionof 30%. The patient currently takes aspirin, metoprolol, furosemide, enalapril, atorvastatin, and glyburide.Metformin and pioglitazone were discontinued at time of admission. Glucose levels in the hospital rangedfrom 98–231 mg/dL. Results of laboratory studies include an A1C level of 7.9%, BUN of 43, and creatinine of 1.6 mg/dL.

Which one of the following is the BEST approach for glycemic management for this patient upon discharge?

(A) Reinstitute metformin only.

(B) Reinstitute both metformin and pioglitazone.

(C) Reinstitute pioglitazone only.

(D) Stop glyburide and start basal insulin.

(E) Continue glyburide only and monitor glucose levels.

17. A 34-year-old obese woman with a history of type 2 diabetes presents to her physician for continuity of care.She reports that she is overall doing well and maintains good adherence to prescribed therapies. Sheoccasionally has low blood glucose readings in the mornings, especially when she does not eat a late nightsnack. Upon further questioning, she reports nightmares that wake her up from sleep, and this has beencausing her some distress. Her medications include an ACE inhibitor, glargine insulin 30 U at bedtime,and aspart insulin, 4 U before meals daily. Her blood glucose log is reviewed and reveals fasting glucose levels before breakfast 49–238 mg/dL (mean 208), before lunch 72–188 mg/dL (mean 160), before dinner61–183 mg/dL (mean 145), and at bedtime 63–177 mg/dL (mean 133). One glucose level at 3 a.m. was 52.Her A1C test was 7.0%.

Which one of the following insulin recommendations is MOST appropriate to improve her glycemic control?

(A) Decrease aspart insulin at dinner.

(B) Increase aspart insulin at meals.

(C) Decrease aspart insulin at meals.

(D) Decrease glargine insulin at bedtime.

(E) Increase glargine insulin at bedtime.

Page 25: ADASAP2 Reduced PDF

23MCQs American Diabetes Association Self-Assessment Program

18. A 51-year-old woman with a history of type 2 diabetes, hypertension, and hyperlipidemia presents to herdoctor for chronic disease management. She has no complaints and takes her medications regularly. She hashad no recent problems with hypoglycemia. She takes metformin 1 g b.i.d. and glargine insulin 32 U/day in the morning. In the past she took glyburide, but this was discontinued due to a skin rash. She takes threeantihypertensive medications, including an ACE inhibitor. On physical exam, the patient is obese (BMI 32 kg/m2), pulse rate 78/minute, and blood pressure 122/76 mm Hg. She has no JVD, clear lungs, and an unremarkable cardiovascular exam, with trace, symmetrical lower extremity edema. Her labs reveal an A1C level of 9.5%. She brings her glucose logbook which reveals the following: fasting breakfast glucose range88–127 mg/dL (mean 106), pre-lunch 90–176 mg/dL (mean 140), and pre-dinner 111–193 mg/dL (mean152). On occasion she has checked her glucose levels after eating; her postprandial glucose levels werebetween 235–288 mg/dL when obtained.

Which one of following medication adjustments will be the BEST option in improving her glycemic control?

(A) Add NPH insulin in the morning.

(B) Add bolus insulin (aspart or lispro) with meals.

(C) Add second dose of glargine insulin at bedtime.

(D) Change glargine insulin to detemir.

(E) Increase metformin dose.

19. Which one of the following statements is INCORRECT regarding postprandial hyperglycemia?

(A) There are randomized control trials demonstrating that lowering postprandial glucose levelsindependent of fasting glucose levels will reduce the risk of cardiovascular disease.

(B) Postprandial hyperglycemia results in overproduction of free radicals and increases endothelialdysfunction.

(C) Individuals with pre-meal glucose levels within target range but have A1C levels above goal should havetreatment directed at reducing postprandial glucose levels.

(D) None of the above.

20. A 60-year-old man with a history of significant mitral valve stenosis is admitted to the hospital for valvereplacement. He has a history of impaired glucose tolerance, and has followed a diet with a low glycemicindex. The surgery itself was uneventful; however, two days postoperative he developed fever, respiratorydistress, and hypotension, and was then transferred to the surgical intensive care unit, where he was intubatedand started on pressors and antibiotic therapy. Laboratory studies demonstrated a white blood cell count of 14.8 and creatinine of 1.3 mg/dL, and an initial plasma glucose level of 183 mg/dL.

Which one of the following is the BEST management of his blood glucose levels at this time?

(A) Sliding scale of regular insulin.

(B) Glargine insulin, 10 U/day.

(C) Continuous IV insulin.

(D) Glyburide 5 mg/day.

(E) Observation.

Page 26: ADASAP2 Reduced PDF

American Diabetes Association Self-Assessment Program MCQs24

21. An 81-year-old African American woman with a history of type 2 diabetes mellitus for over 10 years, coronaryartery disease, NYHA class III congestive heart failure, osteoporosis and emphysema is brought to thephysician by her granddaughter for follow up chronic disease management. The patient has poor mobilityrelated to weakness and imbalance; however, she lives with two family members who are very supportive inher medical care and housework. More recently, she has been feeling depressed with complaints of fatigueduring the day and poor sleep at night. She has fallen twice at home during the past three months. Hermedications include lisinopril, furosemide, nitroglycerin, simvastatin, aspirin, glyburide, calcium, and vitaminD. On physical examination, her blood pressure is 142/78 mm Hg. She has bibasilar rales on lung exam.Cardiac exam reveals an S3 gallop. She has symmetric 1+ lower extremity edema that has not changed sincethe previous visit. She has decreased sensation in her feet by monofilament testing. Laboratory studies revealan A1C level of 8.0%. A three-hour postprandial glucose level in the office is 224 mg/dL.

Which one of the following is the best recommendation regarding her blood glucose management?

(A) Begin low-dose basal insulin once daily.

(B) Add pioglitazone 15 mg/day.

(C) Add metformin 500 mg/day.

(D) Increase glyburide to twice daily.

(E) Continue current therapy.

22. A 56-year-old woman presents to the office for follow up chronic disease management. She has a 3-yearhistory of type 2 diabetes. She takes maximum doses of metformin and glimepiride. She has refused insulintherapy because she is concerned about experiencing weight gain. Her provider has provided suggestions toimprove her diet, including caloric restriction to promote weight loss. She walks regularly three times a weekat a brisk pace. Her fasting glucose levels are usually above 180 mg/dL and her A1C is 8.0%.

Which one of the following would be the MOST appropriate medication to add to her current oral therapy?

(A) Acarbose.

(B) Exenatide.

(C) Pioglitazone.

(D) Sibutramine.

(E) Continue current therapy.

� Items 23–25

A 44-year-old man was recently diagnosed with type 2 diabetes three months prior to presentation to the physician.He currently takes no medications. He is obese (BMI 32 kg/m2) and he reports that his weight has been increasingover the past two years. Since he was diagnosed with diabetes, he has been trying to implement various strategies tolose weight. He has been referred to a dietitian for nutritional counseling and weight management (he has made twovisits so far to the dietitian). His A1C is currently 8.2%.

23. Which one of the following is the BEST management option for his type 2 diabetes at this point?

(A) Continue lifestyle modifications.

(B) Start glipizide.

(C) Start metformin.

(D) Start exenatide.

(E) Start daily glargine insulin.

Page 27: ADASAP2 Reduced PDF

25MCQs American Diabetes Association Self-Assessment Program

24. Which one of the following would be MOST likely to provide the greatest reduction in his A1C level as monotherapy?

(A) Alpha-glucosidase inhibitor.

(B) Exenatide.

(C) Metformin.

(D) Pramlintide.

25. Based on the ADOPT trial, which one of the following drugs, as monotherapy, delays the need for additionalpharmacologic therapy the LEAST for patients with type 2 diabetes?

(A) Biguanide.

(B) Thiazolidinedione.

(C) Sulfonylurea.

(D) No difference between oral agents.

26. Which one of the following is NOT an advantage to using sitagliptin, a dipeptidyl peptidase-4 (DPP-4)inhibitor, for glycemic control in diabetes management?

(A) It provides a modest level of weight loss.

(B) It has low risk of hypoglycemia compared with sulfonylureas.

(C) It can be used as monotherapy.

(D) It can be used in combination with metformin.

(E) It improves β-cell function.

27. A 46-year-old man with a history of type 2 diabetes presents to his primary care physician for follow updiabetes management. He has had diabetes for nine years and has no known complications. He has nocomplaints and denies having any episodes of hypoglycemia. His current medications include metformin 1 g b.i.d., pioglitazone 30 mg/day, and insulin (glargine 30 U every morning, and aspart 20 U before each meal). At home he monitors his glucose levels at various times during the day on different days.On physical exam, he is afebrile, and has a blood pressure of 153/92 mm Hg and pulse of 72. He is obese with a BMI of 36 kg/m2. His cardiovascular exam is unremarkable. He has no extremity edema.Monofilament testing was negative for deficits in sensation. Deep tendon reflexes were intact.

His logbook is reviewed and range values (mg/dL) for various times of the day are as follows:

Time Mean Minimum Maximum

Pre-breakfast 211 160 241

Pre-lunch 231 188 284

Post-lunch (2 hrs.) 201 183 229

Pre-dinner 217 165 237

Post-dinner (2 hrs.) 204 194 255

Page 28: ADASAP2 Reduced PDF

American Diabetes Association Self-Assessment Program MCQs26

Which one of the following measures is MOST appropriate to improve his glycemic control?

(A) Increase pre-meal aspart insulin.

(B) Decrease pre-meal aspart insulin.

(C) Increase morning glargine insulin.

(D) Decrease morning glargine insulin.

(E) Add morning NPH insulin.

28. A 48-year-old man presents to his primary care physician for management of newly-diagnosed type 2diabetes. He denies drinking any alcohol or using tobacco. He is obese with a BMI of 36 kg/m2. Lab resultsinclude an ALT 102, AST 98, alkaline phosphatase 74, BUN 45, and creatinine 1.8 mg/dL. A1C is 7.5%.Viral hepatitis B and C serologies are negative. Ultrasound of the liver shows a diffuse increase in echogenicity,or a “bright” liver. A liver biopsy reveals steatosis with inflammation, fibrosis, and necrosis.

Which one of the following has been shown to improve liver biopsy results in patients with NASH?

(A) Low fat diet.

(B) Pioglitazone.

(C) Insulin.

(D) Glyburide.

(E) Metformin.

29. A 58-year-old man presents to the office for follow up of diabetes care. He has a history of type 2 diabetes for 12 years. He is now taking metformin 1 g b.i.d. and glyburide 10 mg b.i.d.. He reports good adherence to this therapy, and states he has not missed any doses of medication in the past week. He has no specificcomplaints and overall feels well, maintaining an active lifestyle. His last A1C test result was 9.2% two monthsprior. In the office, his blood glucose level is 210 mg/dL fasting. He denies any history of hypoglycemia.

Which one of the following is the BEST option to improve his blood glucose management?

(A) Add a thiazolidenedione.

(B) Add a thiazolidenedione and increase metformin.

(C) Start exenatide.

(D) Start insulin.

(E) Repeat A1C, monitor home glucose levels, and continue current therapy.

Page 29: ADASAP2 Reduced PDF

27MCQs American Diabetes Association Self-Assessment Program

30. A 34-year-old obese woman with a history of type 2 diabetes presents to the office for follow up. She has been on oral medication therapy for two years, including glyburide and pioglitazone (the latter started six months ago). She reports that her morning fasting blood glucose was 163 mg/dL. She denies any recentepisodes of hypoglycemia. Point of care testing A1C is 8.1%, and last level was 7.9% three months ago. Shehas normal renal function. Her last dilated eye exam was without evidence of retinopathy. She requests a urinepregnancy screen and test result is positive for pregnancy. Her last menstrual period was six weeks ago.

What is the BEST treatment choice for this patient at this time?

(A) Stop glyburide and pioglitazone, monitor glucose levels frequently at home, and refer for intensive nutritional and physical activity counseling.

(B) Stop glyburide and pioglitazone and start insulin, NPH at bedtime with lispro before meals.

(C) Add insulin, 70/30 premixed every morning.

(D) Stop glyburide and start glargine insulin, every morning.

(E) Stop pioglitazone and start detemir insulin, every morning.

31. A 50-year-old woman with a history of type 2 diabetes for three years, hypertension, and hyperlipidemiapresents for routine chronic disease care. She has no specific complaints during the visit and reports greateffort in following recommended dietary and exercise plans. She eats three moderate-sized meals each daywith snacks between meals. She walks for 30 minutes on alternate weekdays every week, usually in theevenings on her home treadmill. She works full-time and reports a busy schedule during the day. Her medications include metformin 850 mg b.i.d., lisinopril 20 mg/day, atorvastatin 20 mg at nighttime,and a daily baby aspirin.

On physical exam, her BMI is 30 kg/m2 and her blood pressure is 125/78 mm Hg. Her cardiopulmonaryexam is unremarkable. Abdomen is soft and non-tender. She has trace lower extremity edema. Monofilamenttesting is without sensory deficits.

Her A1C level is 7.4%, higher than the previous level of 6.6% six months ago. Her glucose level in the office is 236 mg/dL, two hours after breakfast. She monitors her blood glucose frequently at home and at work.Her logbook includes the following (mg/dL):

Which one is the following is the BEST option to improve her glycemic control?

(A) Add repaglinide.

(B) Add glargine.

(C) Add aspart insulin before meals on days where patients does not exercise.

(D) Increase metformin to 1000 mg b.i.d..

(E) Increase exercise therapy to 45 minutes per session.

Time Minimum Maximum

Pre-breakfast 82 132

Post-breakfast (2 hrs.) 154 251

Pre-lunch 106 140

Post-lunch (2 hrs.) 85 159

Pre-dinner 78 153

Post-dinner (2 hrs.) 165 288

Page 30: ADASAP2 Reduced PDF

American Diabetes Association Self-Assessment Program MCQs28

32. A 67-year-old man with a history of obesity, type 2 diabetes, hypertension, and hyperlipidemia presents to his physician after recently having dyspnea with exertion. He has since had an abnormal stress test and is scheduled for a heart catheterization in one week. He has a creatinine level of 1.3 mg/dL and microalbuminuria. He currently takes metformin 850 mg b.i.d. and pioglitazone 15 mg/day.

Which one of the following is recommended for this patient prior to catheterization?

(A) Discontinue all diabetes medications.

(B) Hold pioglitazone only the morning of procedure and maintain adequate hydration.

(C) Hold metformin only the morning of procedure and maintain adequate hydration.

(D) None of the above.

� Items 33–37

For each numbered clinical scenario (33–37), select the one lettered medication (A, B, C, D, E) that is MOST appropriate to reach target glucose levels. Each lettered medication man be selected only once.

(A) Metformin.

(B) Glipizide.

(C) Pioglitazone.

(D) Basal insulin.

(E) No additional medication therapy.

33. A 52-year-old obese woman was recently diagnosed with type 2 diabetes. She has not yet tried makinglifestyle modifications, but is motivated to do so. Her A1C level is 8.2% and creatinine level is 1.1 mg/dL.

34. A 38-year-old man has a history of type 2 diabetes for four years. He has not tolerated metformin due togastrointestinal side effects. His A1C is 8.0% and postprandial glucose level in the office is 233 mg/dL.He has no health insurance or drug coverage plan.

35. A 65-year-old woman has a history of type 2 diabetes for nearly 20 years, has no complications, and isconcerned about increased blood glucose levels. She actively volunteers in her community. She is currentlytaking combination therapy of metformin/glipizide. Her A1C level is 8.7%.

36. A 76-year-old man has a history of type 2 diabetes for 12 years and Alzheimer disease with dementia for six years and was recently hospitalized three months ago for congestive heart failure. Rosiglitazone wasdiscontinued at the time of admission. At his follow up appointment, his A1C level is 7.9%. He now takes nomedications for diabetes and his spouse is concerned about his overall current burden of medication therapyfor other conditions.

37. A 72 year women with a history of type 2 diabetes has elevated glucose levels and a A1C of 7.5%. Her lastcreatinine level was 1.5 mg/dL. She stopped taking low-dose glyburide six months ago due to hypoglycemia,and has not taken any medications for diabetes since then with concern for low blood glucose levels.

Page 31: ADASAP2 Reduced PDF

29MCQs American Diabetes Association Self-Assessment Program

38. A 60-year-old woman with a history of type 2 diabetes for 12 years, hypertension and hyperlipidemia presents to her physician for glycemic management. She has no known complications of diabetes, but had micro-albuminuria noted on the most recent urinalysis. She currently takes a baby aspirin, glipizide, metformin,lisinopril, hydrochlorothiazide, and simvastatin. Exenatide was initiated and was discontinued because ofintractable vomiting. She is making a strong attempt at reducing her weight through dietary modifications and regular walking on weekdays. On physical exam, she is obese (BMI 31 kg/m2) with a blood pressure of124/72 mm Hg. Funduscopic exam is unremarkable. Lungs are clear. Cardiovascular exam with normal heart sounds, no S3 or S4. She has trace lower extremity edema. Distal lower extremity reflexes are intact.Monofilament is negative for sensation loss. Her fasting glucose level in the office is 191 mg/dL and her A1C is 8.4%. Most recent laboratory values include a creatinine level of 1.1 mg/dL, AST 34, ALT 42,LDL 98 mg/dL, and HDL 39 mg/dL. Which one of the following is the MOST appropriate change to hermedical therapy for optimal glycemic control?

(A) Stop metformin and add pioglitazone.

(B) Start detemir insulin every morning.

(C) Start 70/30 premixed insulin every morning.

(D) Start aspart insulin with each meal.

� Items 39–41

A 70-year-old man with a history of type 2 diabetes, hypertension, and chronic obstructive pulmonary diseasepresents to the emergency room after being found unresponsive by his wife at home. His wife provides additionalhistory, stating that he was doing well until three days ago when he saw his physician for increasing cough, dyspnea,and associated changes in sputum. She states that his chest x-ray did not show any new changes, and he wasprescribed ciprofloxacin for a flare of chronic bronchitis. Since then, he has improved with less dyspnea, but has aresidual cough. His appetite has recently been low but she reports he ate a good breakfast and lunch prior to calling911 (after he was found unresponsive).

He has a history of type 2 diabetes for three years and does not have any known complications. He takes glyburide5 mg b.i.d., lisinopril 20 mg/day, aspirin 81 mg/day, atorvastatin 20 mg at night, and a combivent inhaler. He hasbeen on a stable medication regimen for the past year. He does not smoke or drink any alcohol. His wife reportsthat his fasting glucose levels range from 90 to 160 mg/dL in the mornings. His last A1C level was 6.8% one month ago.

On physical exam in the emergency room, he is afebrile, with blood pressure of 132/78 mm Hg, and pulse regularat 110. He was not found to be orthostatic. He is conscious in the emergency room but somewhat confused. Hisskin is mildly diaphoretic. Cardiac exam was tachycardic rate and regular rhythm. Lungs with decreased breathsounds throughout, prolonged expiratory phase, and no rales. No lower extremity edema.

Review of the ambulance record in the emergency room reveals that the patient had a fingerstick glucose level of 27 mg/dL when found at this home, and he was started on IV dextrose solution.

39. Which one of the following is the MOST likely cause of his hypoglycemia?

(A) Overdose of glyburide.

(B) Fluoroquinolone-sulfonylurea interaction.

(C) Acute renal failure.

(D) Limited oral intake.

(E) Adrenal insufficiency.

Page 32: ADASAP2 Reduced PDF

American Diabetes Association Self-Assessment Program MCQs30

40. Which one of the following is the MOST appropriate management for this patient?

(A) Hospitalization and continued intravenous dextrose.

(B) Hospitalization, continue intravenous dextrose, and add solumedrol.

(C) Continue intravenous dextrose in the emergency room and discharge when glucose level normalizes and patient is able to eat.

(D) Glucagon, continued intravenous dextrose, and discharge when glucose level normalizes and patient is able to eat.

(E) Hospitalization, octreotide, and switch intravenous fluids to 0.9 normal saline.

41. Which one of the following factors is LEAST likely to influence the development of severe hypoglycemia in this patient?

(A) Longer duration of diabetes and patient age.

(B) Hypoglycemic unawareness.

(C) Use of intensive insulin therapy.

(D) Use of exenatide therapy.

(E) Use of excessive alcohol.

42. A 59-year-old man with a long-standing history of type 2 diabetes, hypertension, and hyperlipidemia presentsfor diabetes management. He has developed mild background retinopathy. For his diabetes, he is takingmetformin 850 mg t.i.d., pioglitazone 30 mg/day, glyburide 10 mg b.i.d., and glargine insulin 30 U everymorning. For his hypertension, he takes amlodipine 5 mg/day, lisinopril 40 mg/day, and hydrochlorothiazide 12.5 mg/day. On physical exam, his blood pressure is 122/69 mm Hg and pulse rate is 77. His exam is notable for 1+ bilateral lower extremity edema. His A1C level is 8.3%. His glucose logbook is reviewed and shows (mg/dL):

Which one of the following is the BEST option for blood glucose management in this patient at this time?

(A) Stop glyburide and start repaglinide before meals.

(B) Stop glyburide and start aspart insulin before meals.

(C) Stop glyburide and start glimepiride 8 mg/day.

(D) Increase pioglitazone to 45 mg/day.

(E) Add a second dose of glargine (10 U at night).

Time Minimum Maximum

Pre-breakfast 90 164

Post-breakfast (2 hrs.) 242 285

Pre-lunch 103 212

Pre-dinner 134 217

Post-dinner (2 hrs.) 215 292

Page 33: ADASAP2 Reduced PDF

31MCQs American Diabetes Association Self-Assessment Program

43. A 53-year-old woman with a history of recently-diagnosed type 2 diabetes presents to her physician with concerns regarding her blood glucose levels when checked at home. Her fasting glucose levels werebetween 123 and 174 mg/dL, while postprandial glucose level ranged from 135 to 206 mg/dL. She currentlytakes metformin 1 g b.i.d. She states that she wants to avoid taking insulin and having to self-administer injections. She has made significant changes in her diet over the past year, eating portion-controlled mealsand drinking diet instead of regular cola beverages. She has met with a dietitian regularly over six months and lost 10 lbs. during this time period (her BMI is now 31 kg/m2). She has attempted aerobic exercises(walking, biking) but has had trouble due to knee pain related to a previous injury. She plans to try eitherswimming or water aerobics in the near future. Her A1C level is currently 7.1%. She is not satisfied with herweight or blood glucose levels, and wishes to pursue additional therapy.

Which one of the following would be the BEST recommendation for management of her type 2 diabetes?

(A) Add pioglitazone.

(B) Add exenatide.

(C) Add glipizide.

(D) Increase metformin.

(E) Add resistance training.

44. Hyperglycemia experienced by hospitalized patients is associated with increased length of stay as well asmorbidity and mortality. A scheduled sliding scale of short-acting insulin for patients in non-critical caresettings has been available for years, and its use remains prevalent today. In general, which one of the followingstatements is CORRECT regarding sliding scale insulin therapy vs basal-bolus insulin therapy?

(A) It is associated with greater hypoglycemia .

(B) It is associated with less hyperglycemia.

(C) It is associated with improved clinical outcomes.

(D) None of the above.

Page 34: ADASAP2 Reduced PDF

American Diabetes Association Self-Assessment Program MCQs32

� Items 45–47

Twenty-four adult patients with uncontrolled type 2 diabetes, aged 30–70 years of age, were enrolled in a researchstudy. All of the subjects received extended-release glipizide 5 mg/day for two weeks during a washout period priorto an insulin-based intervention. Then, patients were randomized to receive either glipizide + NPH insulin versusglipizide + glargine insulin for 12 weeks. Insulin doses were provided at bedtime and titrated according to aprotocol. During the final week of the intervention, continuous glucose monitoring was performed for three days.

For each numbered treatment (45–47), select the one lettered continuous glucose monitoring recording (A, B, C)associated with it. Each lettered continuous glucose monitoring recording may be selected only once.

Wang XL, et al: Evaluation of the superiority of insulin glargine as basal insulin replacement by continuous glucose monitoring system. Diabetes Res Clin Pract Apr 2007;76(1):30–36.

45. Extended-release glipizide + NPH insulin at bedtime.

46. Extended-release glipizide + glargine insulin at bedtime.

47. Extended-release glipizide only (during washout).

400

300

200

100

0

03:00

Gluc

ose

– m

g/dL

06:00 09:00 12:00 15:00 18:00 21:00

68

180

400

300

200

100

0

03:00

Gluc

ose

– m

g/dL

06:00 09:00 12:00 15:00 18:00 21:00

36

36

180

400

300

200

100

0

03:00

Gluc

ose

– m

g/dL

06:00 09:00 12:00 15:00 18:00 21:00

180

(A)

(B)

(C)

Page 35: ADASAP2 Reduced PDF

33MCQs American Diabetes Association Self-Assessment Program

48. A patient with type 2 diabetes who has been on multiple non-insulin treatments, (pioglitazone, metformin,exenatide, and a sulfonylurea), still has fasting fingerstick blood glucose levels ~180–190 mg/dL (none were�140), postprandial fingerstick glucose levels �200 mg/dL and A1C 8.1% after more than 3 months.

Which one of the following courses of action would you recommend for this patient?

(A) Begin insulin.

(B) Observation on better diet, encourage compliance by threatening need for insulin.

(C) Observation on better exercise, encourage compliance by threatening need for insulin.

(D) Observation and hope external life stresses dissipate.

(E) Observation and look for unrecognized hypoglycemia.

� Items 49–53

For each numbered logical explanation for maintaining non-insulin modality treatment (49–53) in patients withtype 2 diabetes being treated with basal-bolus insulin for both fasting and postprandial elevated glucose levels,select the one lettered non-insulin modality MOST closely associated with it. Each lettered non-insulin modalitymay be selected only once.

(A) Metformin.

(B) Pioglitazone.

(C) Incretins.

(D) Rosiglitazone/sulfonylurea.

(E) Acarbose.

49. Decrease postprandial glucose levels by improving glucose-dependent insulin release.

50. Decrease advanced glycosylation end-product (AGE) production.

51. Delay glucose absorption by shifting absorption to distal small intestinal sites.

52. Decrease TGs and increase high density lipoproteins.

53. No logic.

Page 36: ADASAP2 Reduced PDF

American Diabetes Association Self-Assessment Program MCQs34

54. ‘Ideal’ insulin therapy in patients with type 1 diabetes does NOT include which one of the following?

(A) Use of NPH insulin.

(B) Use of fast analog insulin.

(C) Use of regular human insulin.

(D) Use of an insulin pump.

(E) Matching insulin pharmacokinetics to the patient’s lifestyle and nutritional source or eating pattern.

55. Which one of the following is NOT a rationale for the use of fast analog insulin over regular human insulin inany regimen which involves prandial dosing?

(A) It addresses the issue of gastroparesis better than regular insulin.

(B) It is associated with less 4 hr hypoglycemia.

(C) It is associated with less postprandial area under curve hyperglycemia.

(D) It may be given immediately after eating at times when the patient doesn’t know how much they plan to eat.

(E) It is associated with less 2 hr hyperglycemia.

56. Advantages of appropriate doses of a fast analog insulin versus regular insulin in hospital settings does NOT include which one of the following?

(A) Reduction of the occurrence of hypoglycemia several hours after eating when the next meal is delayedbecause the patient comes back to the floor late from a study.

(B) Reduced risk of hypoglycemia due to stacking effects of recurrent insulin doses every 3–4 hours.

(C) It can be given after eating at reduced doses in patients with inconsistent oral intake.

(D) It can be given IV with reduced risk of hypoglycemia.

Page 37: ADASAP2 Reduced PDF

35MCQs American Diabetes Association Self-Assessment Program

� Items 57–59

A physician and his 45-year-old male patient with type 2 diabetes chose to start a basal insulin glargine whilecontinuing to use 3 oral agents with a goal of controlling his fasting glucose with the basal insulin and seeing if the metformin pioglitazone, sitagliptin combination also controlled postprandial glucose levels.

Glargine was started at 10 U at night and was titrated by phone calls to the physician’s office so that at 26 U, hisfasting glucose was 90–120 mg/dL; most of his postprandials were 120–160 mg/dL. He came back 1 month laterand his weight was up 1 lb. However, when he had noticed that occasional fasting glucose levels were elevated, he,on his own, increased his glargine insulin to 34 U. It did not help and he still saw intermittent elevated fastingglucose levels sometimes even worse than before and he gained 4 lbs. in a month. He frequently wakes up withheadaches and complains of nightmares and night sweats.

57. Which one of the following is the MOST likely explanation for this patient’s problem?

(A) Noncompliance with diet, intermittently.

(B) Dawn effect.

(C) Middle of night hypoglycemia with a.m. rebound.

(D) Misinjection/administration insulin.

58. Which one of the following is the MOST important therapeutic intervention for this patient?

(A) Decrease glargine dosage.

(B) Advise the patient on high glycemic index foods.

(C) Advise the patient to monitor 3 a.m. glucose levels.

(D) Check the patient’s insulin injection technique.

59. You see the patient 3 months later. His insulin glargine was adjusted to 23 U with good control of the fastingglucose levels roughly 100 to 120 mg/dL. He was maintained on metformin, pioglitazone, and sitagliptin.However, gradually over time his postprandial glucose levels increased to 160 mg/dL and not infrequentlyover 180 mg/dL.

You recommend basal-bolus insulin q.i.d., but he refuses. He agrees to begin a premixed preparation of intermediate-acting insulin with fast analog, twice daily, before breakfast and dinner.

Which one of the following statements is INCORRECT regarding premixed insulin preparations?

(A) They provide a more accurate injection of correct dose than self-mixture of NPH and a short acting insulin.

(B) They offer no problem in titrating doses.

(C) They require eating a consistent amount of food at approximately the same time each day.

(D) They are more convenient to use.

(E) They are available with regular as well as fast analogs.

Page 38: ADASAP2 Reduced PDF

American Diabetes Association Self-Assessment Program MCQs36

60. Which one of the following is NOT an advantage of insulin pump therapy over multiple daily injections (MDI)?

(A) It covers dawn phenomena.

(B) It is easier to deal with changes in glucose levels with intermittent exercise.

(C) There is no difference in risk of diabetic ketoacidosis (DKA).

(D) It is easier to manage delayed carbohydrate absorption with meals containing complex carbohydrates;higher fat content meals; higher fiber meals; and gastroparesis.

(E) It results in less hypoglycemia.

� Items 61–66

For each numbered mechanism (61–66), select the one lettered oral hypoglycemic agent (A, B, C, D, E) associatedwith it. Each lettered oral hypoglycemic agent may be selected once, more than one, or not at all.

(A) Metformin.

(B) Pioglitazone/Rosiglitazone.

(C) Sulfonylurea/Glinide.

(D) Exenatide/Sitagliptin.

(E) Acarbose.

61. Decreases insulin resistance at the liver.

62. Increases insulin sensitivity at muscle/fat.

63. Decreases rate of carbohydrate breakdown in the proximal small intestine.

64. Decreases glucagon.

65. Increases insulin secretion (of remaining β-cells), glucose-dependent manner.

66. Increases insulin secretion (of remaining β-cells), glucose-independent manner.

Page 39: ADASAP2 Reduced PDF

37MCQs American Diabetes Association Self-Assessment Program

67. According to an ADA consensus group algorithm, initial therapy for patients with type 2 diabetes includesdiet, lifestyle changes, and which one of the following oral agents?

(A) Sulfonylureas.

(B) Metformin.

(C) Thiazolidinediones.

(D) Alpha glucosidase inhibitors.

68. Which one of the following is NOT a risk factor for hypoglycemia in patients with type 2 diabetes?

(A) Missed or delayed meal.

(B) Elderly age.

(C) Substitution of intermediate-acting insulins (such as NPH or 70/30) with long-acting insulins (such as glargine or detemir).

(D) Renal insufficiency.

(E) Alcohol consumption.

69. Intensive treatment of patients with type 1 diabetes has been shown to decrease the incidence of which one of the following complications over time?

(A) Retinopathy.

(B) Nephropathy.

(C) Neuropathy.

(D) Cardiovascular disease.

(E) All of the above.

70. Which one of the following statements regarding acute hyperglycemic crises is INCORRECT?

(A) The most common precipitating factors in the development of diabetic ketoacidosis (DKA) and hyperos-molar hyperglycemic state (HHS) and are inadequate/inappropriate insulin therapy and infection.

(B) The mortality rates for both DKA and HHS have both decreased over time and are roughly equivalent.

(C) Patients with low normal potassium at presentation have severe depletion of total body potassium storesand need to be aggressively repleted and carefully monitored.

(D) The average fluid deficit is greater in HHS (8–10 liters) compared to DKA (3–6 liters).

(E) The treatment of choice for both DKA and HHS is intravenous insulin.

Page 40: ADASAP2 Reduced PDF

American Diabetes Association Self-Assessment Program MCQs38

� Items 71–74

A 62-year-old Caucasian man seeks to establish care with you. His past medical history includes type 2 diabetes,hypertension, and renal insufficiency. He reports his diabetes was initially controlled with diet but an oral agent,glyburide, was added 4 years ago. For his hypertension he takes ramipril 20 mg/day. He states he watches his carbohydrate and salt intake. He does not get any regular exercise.

His physical exam is remarkable for BMI of 34 kg/m2, blood pressure of 132/88 mm Hg, decreased sensation bilaterally with monofilament testing. The remainder of his exam is normal.

Labs results are as follows:

71. Since he is overweight and does not get any regular exercise, you suggest that increasing his physical activitymay be helpful in controlling his diabetes. Which one of the following statements is INCORRECT regardingdiabetes control, weight, and activity?

(A) Weight loss of at least 7% of initial body weight with diet and exercise would likely improve glycemic control.

(B) A mean sustained weight loss of �20 kg has resulted in remission of hyperglycemia in studies of obese subjects undergoing bariatric surgery.

(C) Long-term adherence and injury are major problems in continuing exercise therapy.

(D) Diet and exercise therapy, if maintained long-term, will likely allow maintenance of euglycemia.

(E) The major environmental risk factors that increase the risk of type 2 diabetes are overnutrition andsedentary lifestyle.

72. The patient does not feel he can take time to increase his activity over the next several months because of his current job situation. According to the ADA treatment algorithm for type 2 diabetes, the next MOST appropriate treatment option would be which one of the following?

(A) Add metformin.

(B) Add exenatide as this will likely significantly improve his fasting blood glucose levels.

(C) Add either a basal insulin or a thiazolidinodione.

Total Cholesterol 181 mg/dL

HDL-C 42 mg/dL

Fasting Plasma Glucose 175 mg/dL

A1C 8.3%

Creatinine 1.6 mg/dL

Urinary Albumin/Creatinine 100 mg/g

Page 41: ADASAP2 Reduced PDF

39MCQs American Diabetes Association Self-Assessment Program

73. The patient develops hypoglycemia with the new regimen. Which one of the following statements is INCORRECT regarding his hypoglycemia?

(A) Glyburide is more likely to cause hypoglycemia than other sulfonylureas.

(B) Utilization of sulfonylureas in the setting of renal insufficiency increases his risk of hypoglycemia.

(C) The incidence of hypoglycemia increases with higher dosages of sulfonylureas.

(D) Because of its long half-life, glyburide suppresses hepatic glucose output.

(E) Sulfonylureas are more likely to cause hypoglycemia in patients who are overweight.

74. Upon review of his blood pressure and urine results, which one of the following statements is CORRECT?

(A) He has macroalbuminuria.

(B) An additional agent (a thiazide diuretic) should be added.

(C) An additional agent (an angiotensin receptor blocker [ARB]) should be added.

(D) His blood pressure is at goal and he does not need any additional agents.

(E) His angiotensin-converting enzyme inhibitor (ACE-I) should be changed to an ARB.

� Items 75–76

You are consulted for glycemic control in a 51-year-old obese woman who is being discharged after hospitalizationfor deep venous thrombosis. Her past medical history includes type 2 diabetes for the last 2 years, hypertension,mild COPD, and hyperlipidemia. Her BMI is 34 kg/m2. She had been off all diabetic medications for the last 6 months because of inability to afford her medications.

Her lab tests are as follows:

75. Which one of the following statements regarding her condition is INCORRECT?

(A) Weight loss is likely to improve her liver abnormalities.

(B) Her condition is benign and will not progress to more advanced liver disease.

(C) She is more likely to have positive hepatitis C antibodies than the general population.

(D) Alcohol should be avoided because of its toxic effects on the liver.

A1C 7.9%

Creatinine 1.2 mg/dL

AST 93 (normal �45)

ALT 88 (normal �40)

Page 42: ADASAP2 Reduced PDF

American Diabetes Association Self-Assessment Program MCQs40

76. Which one of the following statements about liver disease and diabetes is INCORRECT?

(A) Diabetes is the most common co-morbidity for liver disease in the U.S.

(B) Statins appear to be safe in patients with diabetes and liver disease.

(C) Patients with hepatitis C are more likely to develop diabetes than patients with hepatitis B.

(D) Liver biopsy is necessary to make the diagnosis of nonalcoholic fatty liver disease (NAFLD).

(E) There have been case reports of idiosyncratic heptotoxicity with ACE inhibitors.

77. Which one of the following conditions is NOT a risk factor for the development of heart failure duringtreatment of patients with type 2 diabetes with thiazolidinediones (TZDs)?

(A) Hypertension.

(B) Advanced age (�70 years old).

(C) Mild renal insufficiency (creatinine of 1.5 mg/dL).

(D) Co-administration with insulin.

(E) Coronary artery disease.

� Items 78–83

You have admitted a 54-year-old obese man with new diagnosis of heart failure. He has an 8-year history of well-controlled type 2 diabetes. His outpatient diabetes regimen includes metformin XR 2000 mg/day,pioglitazone 30 mg/day and glipizide XL 20 mg/day.

78. Which one of the following statements about diabetes and heart failure is INCORRECT?

(A) An independent association between diabetes and the incidence of heart failure has not been proven.

(B) Diabetes is a risk factor for the progression of heart failure.

(C) Diabetes is a predictor of mortality in patients with heart failure.

(D) Heart failure itself is associated with insulin resistance and the development of diabetes.

79. Which one of the following courses of action will allow you to BEST manage his diabetes as an inpatient?

(A) Discontinue the pioglitazone only.

(B) Discontinue pioglitazone and metformin.

(C) Discontinue all oral medications and start insulin.

(D) Continue metformin and glipizide and add insulin.

(E) Continue glipizide and add insulin.

Page 43: ADASAP2 Reduced PDF

41MCQs American Diabetes Association Self-Assessment Program

80. During his hospitalization, he has an echocardiogram which shows an ejection fraction of 40%. He is placedon a heart failure regimen including diuretics, ACE inhibitor and a β-blocker. At discharge, he has nosymptoms of heart failure at rest or with exertion (New York Heart Association Class I) and has normal renaland hepatic function. Which one of the following will you order on discharge if his main goal is to lose weight?

(A) Insulin since it is the only medication that is safe in patients with heart failure.

(B) Metformin, glipizide and pioglitazone.

(C) Metformin, glipizide and exenatide.

(D) Metformin, glipizide, and sitagliptin.

(E) Metformin, glipizide and insulin.

81. What is his A1C goal as an outpatient?

(A) �7%.

(B) 7–8%.

(C) 8–9%.

(D) �9%.

82. What is the increased risk of fluid retention and heart failure in patients with type 2 diabetes treated with thiazolidinediones?

(A) No increased risk.

(B) 2-fold.

(C) 4-fold.

(D) 8-fold.

83. Which one of the following is a recently recognized side effect of thiazolidinediones?

(A) Increased incidence of fractures.

(B) Increase in visceral fat.

(C) Increased risk of hypoglycemia.

(D) Weight gain only if used in combination with insulin.

84. DPP-IV inhibitors are currently FDA approved for treatment of patients with type 2 diabetes in all thefollowing settings EXCEPT:

(A) As monotherapy.

(B) In combination with metformin.

(C) In combination with pioglitazone.

(D) In combination glimepiride.

(E) In combination with exenatide.

Page 44: ADASAP2 Reduced PDF

American Diabetes Association Self-Assessment Program MCQs42

85. Which one of the following regimens has the HIGHEST risk of hypoglycemia in patients with type 2 diabetes?

(A) Sitagliptin and sulfonylurea.

(B) Exenatide, metformin, and sulfonylurea.

(C) Metformin and sulfonylurea.

(D) Exenatide and thiazolidinedione.

86. Which one of the following is a precaution against using exenatide in patients with type 2 diabetes?

(A) Renal insufficiency (creatinine clearance less than 30 ml/min).

(B) Normal weight.

(C) Mild chronic hepatitis C.

(D) Current treatment with insulin.

87. For a patient with type 2 diabetes who is inadequately controlled on metformin monotherapy, by how muchwould the addition of sitagliptin be expected to drop A1C level (absolute %)?

(A) 0.2%.

(B) 0.4%.

(C) 0.7%.

(D) 1%.

(E) 1.5%.

88. In deciding on how to improve A1C control in a patient with type 2 diabetes, currently on maximal doses of metformin and sulfonylurea, you are deciding between the addition of basal insulin versus exenatide.Which one of the following statements is INCORRECT?

(A) Addition of exenatide is more likely to cause weight loss compared to addition of basal insulin.

(B) Addition of exenatide is more likely to cause GI side effects compared to addition of basal insulin.

(C) Both treatments are equally likely to decrease fasting plasma glucose.

(D) Both treatments are likely to cause an increased risk of hypoglycemia.

(E) Both treatments are likely to decrease A1C by a roughly equivalent amount.

89. In examining the long-term studies of exenatide as a treatment addition to metformin and/or sulfonylurea,which one of the following statements is MOST accurate?

(A) The effects of exenatide decrease over time.

(B) Only one half of patients lose weight.

(C) Patients with the greatest weight loss will likely have the greatest reduction in A1C.

(D) The average decrease in A1C will be about 2%.

(E) At 1 year, the average weight loss will be about 10 kg.

Page 45: ADASAP2 Reduced PDF

43MCQs American Diabetes Association Self-Assessment Program

90. You are seeing a 57-year-old obese woman on hemodialysis with a 13-year history of type 2 diabetes. She hasbeen on insulin in the remote past but has had good control over the last year on a short-acting sulfonylurea.However, she had a recent hospitalization for severe hypoglycemia. She is discharged without any medicationsfor diabetes and in your office her fasting fingerstick glucose is 114 mg/dL and her A1C is 7.6%.

What treatment should you recommend for her?

(A) Restart the sulfonylurea.

(B) Start sitagliptin.

(C) Start basal insulin.

(D) Metformin.

91. Which one of the following statements regarding pramlintide is INCORRECT?

(A) It is only approved as adjunctive treatment to insulin.

(B) It is approved for use only in patients with type 1 diabetes.

(C) The main side effect is nausea.

(D) It is injected prior to each meal.

(E) Pramlintide can be associated with mild weight loss.

Page 46: ADASAP2 Reduced PDF

� Notes

American Diabetes Association Self-Assessment Program44

Page 47: ADASAP2 Reduced PDF

45

AppendixI. American Diabetes Association Guidelines

and Treatment Algorithms At-a-Glance

Appendix American Diabetes Association Self-Assessment Program

Page 48: ADASAP2 Reduced PDF

American Diabetes Association Guidelines and Treatment Algorithms At-A-Glance Appendix46

Definition and Description of Diabetes MellitusDiabetes mellitus is a group of metabolic diseases characterized by hyperglycemia resulting from defects in insulinsecretion, insulin action, or both. The chronic hyperglycemia of diabetes is associated with long-term damage,dysfunction, and failure of various organs, especially the eyes, kidneys, nerves, heart, and blood vessels.

I. Diagnosis of Impaired Glucose Tolerance (IGT) and Impaired Fasting Glucose (IFG)This group is defined as having fasting plasma glucose (FPG) levels �100 mg/dL (5.6 mmol/L) but �126 mg/dL (7.0 mmol/L) or 2-h values in the oral glucose tolerance test (OGTT) of �140 mg/dL (7.8 mmol/L)but �200 mg/dL (11.1 mmol/L). Thus, the categories of FPG values are as follows:

� FPG �100 mg/dL (5.6 mmol/L) normal fasting glucose;

� FPG 100–125 mg/dL (5.6–6.9 mmol/L) = IFG;

� FPG �126 mg/dL (7.0 mmol/L) provisional diagnosis of diabetes (the diagnosis must be confirmed,as described below).

The corresponding categories when the OGTT is used are the following:

� 2-h post load glucose �140 mg/dL (7.8 mmol/L) normal glucose tolerance;

� 2-h post load glucose 140–199 mg/dL (7.8–11.1 mmol/L) = IGT;

� 2-h post load glucose �200 mg/dL (11.1 mmol/L) provisional diagnosis of diabetes (the diagnosis must be confirmed, as described below).

Patients with IFG and/or IGT are now referred to as having “pre-diabetes” indicating the relatively high risk for development of diabetes in these patients.

Table 1. Treatment Recommendation for Individuals with IFG, IGT, or Both.

* Metformin 850 mg twice/day.

Population Treatment

IFG or IGT. Lifestyle modification (i.e., 5–10% weight loss and moderateintensity physical activity �30 min/day).

Individuals with IFG and IGT and any of the following: Lifestyle modification (as above) and consider metformin*.� �60 years of age.� BMI �35 kg/m2.� Family history of diabetes in first-degree relatives.� Elevated triglycerides.� Reduced HDL-C.� Hypertension.� A1C �6.0%.

Page 49: ADASAP2 Reduced PDF

47

II. Diagnosis of Type 2 DiabetesTable 2. Criteria for Testing for Diabetes in Asymptomatic Adult Individuals.

* May not be correct for all ethnic groups.

Table 3. Testing for Type 2 Diabetes in Children.

Clinical judgment should be used to test for diabetes in high-risk patients who do not meet these criteria.

Table 4. Criteria for the Diagnosis of Diabetes Mellitus.

In the absence of unequivocal hyperglycemia, these criteria should be confirmed by repeat testing on a different day. The third measure (OGTT) is not recommended for routine clinical use.

1. Symptoms of diabetes plus casual plasma glucose concentration �200 mg/dL (11.1 mmol/L).1). Casual is defined as any time of day without regard to time since last meal. The classic symptoms of diabetes includepolyuria, polydipsia, and unexplained weight loss.

OR

2. FPG �126 mg/dL (7.0 mmol/L). Fasting is defined as no caloric intake for at least 8 hours.

OR

3. 2-h post load glucose �200 mg/dL (11.1 mmol/L) during an OGTT. The test should be performed as described by the World Health Organization, using a glucose load containing the equivalent of 75 g anhydrous glucose dissolved in water.

Criteria

� Overweight (BMI �85th percentile for age and sex, weight for height �85th percentile, or weight �120% of ideal for height).

Plus any two of the following risk factors:

� Family history of type 2 diabetes in first- or second- degree relative.� Race/ethnicity (Native American, African-American, Latino, Asian American, Pacific Islander).� Signs of insulin resistance or conditions associated with insulin resistance (acanthosis nigricans, hypertension,

dyslipidemia, or PCOS).� Maternal history of diabetes or GDM.

Age of initiation: age 10 years or at onset of puberty, if puberty occurs at a younger age.Frequency: every 2 years.Test: FPG preferred.

1. Testing for diabetes should be considered in all individuals at age 45 years and above, particularly in those with a BMI �25 kg/m2*, and, if normal, should be repeated at 3-year intervals.

2. Testing should be considered at a younger age or be carried out more frequently in individuals who are overweight (BMI �25 kg/m2*) and have additional risk factors:� Are habitually physically inactive.� Have a first-degree relative with diabetes.� Are members of a high-risk ethnic population (e.g., African-American, Latino, Native American, Asian American,

Pacific Islander).� Have delivered a baby weighing �9 lbs or have been diagnosed with GDM.� Are hypertensive (�140/90 mm Hg).� Have an HDL-C level �35 mg/dL (0.90 mmol/L) and/or a triglyceride level �250 mg/dL (2.82 mmol/L).� Have PCOS (polycystic ovary syndrome).� On previous testing, had IGT or IFG.� Have other clinical conditions associated with insulin resistance (e.g., PCOS or acanthosis nigricans).� Have a history of vascular disease.

Appendix American Diabetes Association Guidelines and Treatment Algorithms At-A-Glance

Page 50: ADASAP2 Reduced PDF

American Diabetes Association Guidelines and Treatment Algorithms At-A-Glance Appendix48

III. Summary of Recommendations for Adults with Diabetes.

* Referenced to a non-diabetic range of 4.0–6.0% using a DCCT-based assay.† Postprandial glucose measurements should be made 1–2 hours after the beginning of the meal, generally peak levels in patients with diabetes.‡ Current NCEP/ATP III guidelines suggest that in patients with triglycerides �200 mg/dL, the “non-HDL-C” (total cholesterol minus HDL) be utilized.

The goal is �130 mg/dL (121).§ For women, it has been suggested that the HDL-C goal be increased by 10 mg/dL.

Table 5. Detailed Diabetes History and Evaluation.

Adapted from American Diabetes Association Clinical Practice Recommendations, 2006. Diabetes Care 2006;29(suppl. 1):S9.

Past medical history should include:

� Prior glucose values including A1C, symptoms, treatments, and history of self-monitoring.� History of complications: hypoglycemia and/or ketoacidosis – including frequency, severity, and causes.� Self-management, self-monitoring education, attitudes, health beliefs.� Mood disorder.� Prior or current infections, particularly skin, foot, dental, and genitourinary.� Current microvascular symptoms and complications (eye, kidney, nerves).� Current macrovascular symptoms and complications (evaluation of cardiovascular risk factors and cardiovascular disease).� Sexual dysfunction.� Review of past/present lifestyle modification

� Eating patterns � Smoking history� Diet history � Alcohol history� Weight history � Controlled substances� Exercise history

� Other medications – evaluate for effect on blood glucose.� Family history of diabetes and CVD.

Referrals

� Eye exam. � Diabetes educator.� Family planning for women of reproductive age. � Foot specialist.� Dietitian for medical nutrition therapy (MNT). � Dentist.

Glycemic Control

A1C �7.0%*

Preprandial capillary plasma glucose 90–130 mg/dL (5.0–7.2 mmol/L)

Peak postprandial capillary plasma glucose† �180 mg/dL (�10.0 mmol/L)

Blood pressure �130/80 mm Hg

Lipids‡

LDL-C �100 mg/dL (�2.6 mmol/L)

Triglycerides �150 mg/dL (�1.7 mmol/L)

HDL-C �40 mg/dL (�1.0 mmol/L)§

Key concepts in setting glycemic goals:� A1C is the primary target for glycemic control.� Goals should be individualized.� Certain populations (children, pregnant women, and elderly) require special considerations.� More stringent glycemic goals (i.e., a normal A1C, �6%) may further reduce complications at the cost

of increased risk of hypoglycemia.� Less intensive glycemic goals may be indicated in patients with severe or frequent hypoglycemia.� Postprandial glucose may be targeted if A1C goals are not met despite reaching preprandial glucose goals.

Page 51: ADASAP2 Reduced PDF

49

Table 6. Follow-up Testing Recommendations with Diabetes.

Adapted from American Diabetes Association Clinical Practice Recommendations, 2006. Diabetes Care 2006;29(suppl. 1):S4–42.

Potential Complications Test Frequency of Evaluation

Hyperglycemia A1C Measure A1C 2 times/year if at goal, quarterly if therapy changed, or not at goal

Fasting blood glucose Varies depending on prescription regime

Self-monitoring of blood glucose Frequency/timing should be individualizedto facilitate reaching goals

Hypertension Blood pressure Measure at every visit

Autonomic Neuropathy Sitting and standing blood pressure Appropriate patients/each visit(Orthostatic Hypotension)

Dyslipidemia Lipid panel Test annually – More often if not at goal or when changing therapy

Foot Care Quantitative somato-sensory threshold Perform a visual inspection at eachtest using Semmes-Weinstein 5.07 (10 g) routine visit for people with neuropathy.monofilament, tuning fork, palpation, and visual inspection

Nephropathy Serum creatinine Annually

Albumin/Creatinine ratio Type 2 diabetes mellitus – Annual screening

(Random spot urine) Type 1 diabetes mellitus – Annual screening with duration of diabetes �5 years

Type 1 & 2 diabetes mellitus – 2 of 3 specimens within a 3–6 month period should be abnormal before consideringa new diagnosis

Retinopathy Comprehensive eye exam Type 2 diabetes mellitus – Annual dilated &comprehensive exam

Type 1 diabetes mellitus – Initial dilated &comprehensive exam within 5 years of onset;annual exams thereafter

Periodontal disease (common Dental evaluation Annuallyin type 2 diabetes mellitus)

Influenza and pneumonia Immunization Annual influenza vaccination

Annual comprehensive foot exam

Appendix American Diabetes Association Guidelines and Treatment Algorithms At-A-Glance

Page 52: ADASAP2 Reduced PDF

American Diabetes Association Guidelines and Treatment Algorithms At-A-Glance Appendix50

Figure 1. Consensus Algorithm for the Metabolic Management of Type 2 Diabetes. Reinforce Lifestyle Intervention at Every Visit.

* Check A1C every 3 months until �7% and then at least every 6 months.† Although three oral agents can be used, initiation and intensification of insulin therapy is preferred based on effectiveness and expense.

Each anti-diabetic medication class possesses differences with regard to contraindications, adverse effects andmechanisms of action that may dictate which medication is the best for an individual patient (Table 7). Sulfony-lurea therapy may be most useful in thin patients with insulinopenia, while metformin may be most useful in obese patients with dyslipidemia. The α-glucosidase inhibitors and meglitinides may be most useful in patients with exaggerated postprandial increases in blood glucose, however the α-glucosidase inhibitors can be used withsulfonylureas while the meglitinides can be used with metformin. The thiazolidinediones may be most useful inpatients with insulin resistance or azotemia. Insulin may be useful in patients who will not be able to reach theirglycemic goal with the use of the other adjunct therapies just mentioned.

Exenatide injection is the first in a new class of drugs for the treatment of type 2 diabetes called incretin mimetics.Exenatide increases glucose-dependent insulin secretion, suppresses inappropriately elevated glucagon secretion,and slows gastric emptying. Exenatide is indicated as adjunctive therapy to improve glycemic control in patientswith type 2 diabetes who are taking sulfonylurea, metformin, a TZD or combinations of these agents but have notachieved adequate glycemic control.

Lifestyle Intervention + Metformin

Diagnosis

Add Sulfonylurea(Least expensive)

Add Glitazone(No hypoglycemia)

Add Basal Insulin(Most effective)

Yes*No A1C �7%

Yes*No A1C �7% Yes*No A1C �7% Yes*No A1C �7%

Intensify Insulin Add Glitazone† Add Basal Insulin Add Sulfonylurea†

Yes*No A1C �7%Yes*No A1C �7%

Add Basal or Intensify Insulin

Intensive Insulin + Metformin +/- Glitazone

Page 53: ADASAP2 Reduced PDF

51

Table 7. Drug Choices Based on Drug & Patient Characteristics

Other Drug Information

Primary failure rate

20% 20% 20% 20% No data

Cost xx xxx Xgeneric, XXbrand xx xxx xxx xx

? Ø long termcomplications

Ø AGE products in vitroØ endothelialdysfunction

Ø endothelialdysfunctionPreserve beta cell function.Only for Pio= advantageous for lipid profile

Metformin Thiazolidinedione SulfonylureaRepaglinide/Nateglinide* Exenatide

Sitagliptin(Dpp-4inhibtor)

Acarbose/Miglitol

Dosingfrequency

2–3x daily Piogli- Rosigli-tazone: tazone:1x daily 2x daily

Glime- Glipizide/pramide: Glyburide:1x daily 2x daily

* 1 with each meal

Inject beforebreakfast and dinner

1x daily 3x daily with meals

Speed of actionFBS/ppg

Weeks/weeks Weeks Day Day Day Day Months

EfficacyPostprandialFBSHgA1

xxxxxxxx

xxxxxxxx

xxxxxxxx

xxx xxxxx xxxx x

xx

xx

xxxxx

xxxxx

Concern/side-effects

GI upset, lacticacidosis: avoid inpatients with CHF,liver disease, acuteillness, otheracidosis, Cr >1.4,hospitalizedpatients and hold 2 days after IV contrast untilCreatinine stable

EdemaAvoid with no saltdiet & avoid NSAIDChannel blocker,COX 2 inhibitorsCHF 1%

wt gain +++ wt gain ++/+ Nausea –Avoid by stopeating whenfeel full

None GI side- effectsTreat hypowith glucose

Hypoglycemicrisk when used alone

None None + + + + +/ + None None None

Monitor for Creatinine &creatinine clearance

Edema/Heart Failure

Hypoglycemia Hypoglycemia Hypoglcemiaif use withSU/insulin

Hypglocemiaif use withSU/insulin

LFTs

Weight Ø OR NC ≠ OR NC ≠ ≠ Decrease Neutral NC

LipidsHDLLDLTG

NCØ ØØ Ø

≠ ≠≠ ≠Ø Decrease

bothNCNCØ OR NC Decrease Decrease

NCNCNC

Insulin levels Ø Ø ≠ (less with glim.) ≠ Glucose-dependentrise

Glucose-dependentrise

NC

Elderly AVOID if >70 years(because Øcreatinineclearance) avoid Cr decrease <70

No special risks Watch for Hypoglycemia

Watch forHypo-glycemia

Ideal Ideal No specialrisks (GI side-effects maybe beneficial)

Preg risk in pts.with PolycysticOvary Syndrome

+ +

Appendix American Diabetes Association Guidelines and Treatment Algorithms At-A-Glance

Page 54: ADASAP2 Reduced PDF

American Diabetes Association Guidelines and Treatment Algorithms At-A-Glance Appendix52

Bibliography1. Byetta Prescribing Information. February 2007. www.byetta.com. Accessed March 2007.2. Fineman MS, et al: Effect on glycemic control of exenatide (synthetic exendin-4) additive to existing metformin and/or sulfonylurea

treatment in patients with type 2 diabetes. Diabetes Care 2003;26(8):2370–7.3. FDA website. www.fda.gov/medwatch. Accessed June 2005.4. Buse JB, et al. Effects of exenatide (exendin-4) on glycemic control over 30 weeks in sulfonylurea-treated patients with type 2 diabetes.

Diabetes Care 2004;27(11):2628–35.5. Triplitt CL, et al: Diabetes Mellitus. In Dipro J, et al(eds). Pharmacotherapy: Pathophysiologic approach. 6th ed.

New York, NY: McGraw-Hill Companies. 2005;1333–13676. The American Association of Clinical Endocrinologists medical guidelines for the management of diabetes mellitus:

The AACE system of intensive diabetes self-management 2002 update. Endocrine Practice 2002;8(Suppl. 1):40–65.7. American Diabetes Association. Diagnosis and classification of diabetes mellitus. Diabetes Care 2005;28(Suppl. 1):S37–S42.8. American Diabetes Association. Standards of medical care in diabetes. Diabetes Care 2005;28(Suppl. 1):S4–S36.9. Poulsen MK, et al: The combined effect of triple therapy with rosiglitazone, metformin, and insulin aspart in type 2 diabetic patients.

Diabetes Care 2003;26(12):3273–3279.10. Dailey GE, et al: Glycemic control with glyburide/metformin tablets in combination with rosiglitazone in patients with type 2 diabetes:

a randomized, double-blind trial. Am J Med 2004;116:223–229.11. DeFronzo RA, et al: Effects of exenatide (exendin-4) on glycemic control and weight over 30 weeks in metformin treated patients

with type 2 diabetes. Diabetes Care 2005;28(5):1092–1100.12. Kendall DM, et al: Effects of exenatide (exendin-4) on glycemic control over 30 weeks in patients with type 2 diabetes treated with

metformin and a sulfonylurea. Diabetes Care 2005;28(5):1083–1091.13. Klein S, et al: Weight management through lifestyle modification for the prevention and management of type 2 diabetes: rational

and strategies. Diabetes Care 2004;27(8):2067–2073.14. Chobanian AV, et al: Seventh Report of the Joint National Committee on prevention, detection, evaluation and treatment of high

blood pressure. Hypertension 2003;42:1206–1252.15. Heine RJ, et al: Exenatide versus insulin glargine in patients with suboptimally controlled type 2 diabetes. Annals of Internal Medicine

2005;143:559–569.16. Nathan DM, et al: Management of hyperglycemia in type 2 diabetes: A consensus algorithm for the initiation and adjustment of

therapy – A consensus statement from the American Diabetes Association and the European Association for the Study of Diabetes.Diabetes Care 2006;29(8)1963–1972.

Page 55: ADASAP2 Reduced PDF

53MCQs American Diabetes Association Self-Assessment Program

American Diabetes AssociationSelf-Assessment ProgramEducational Critiques

Educational (Learning) Critiques American Diabetes Association Self-Assessment Program

IN ORDER TO MAXIMIZE YOUR LEARNING EXPERIENCE IT IS IMPORTANT THAT YOU DO NOT LOOK AT THE EDUCATIONAL CRITIQUES UNTIL AFTER YOU HAVE COMPLETEDMARKING YOUR ANSWERS FOR THE MCQs ON THE ANSWER SHEET.

DO NOT CHANGE THE ANSWERS MARKED ON YOUR ANSWER SHEET WHILE READING THE CRITIQUES. THE INTENT OF ADA-SAP™ IS LEARNING NEW KNOWLEDGE ANDREINFORCING PREVIOUSLY LEARNED KNOWLEDGE. THERE IS NO PASS-FAIL SCORE.

Page 56: ADASAP2 Reduced PDF

American Diabetes Association Self-Assessment Program Educational (Learning) Critiques54

Educational (Learning) CritiquesItems 1–97

Item 1 True Answers A, C; False Answers B, D, E

The ADA Guidelines recommend an A1C test at least 2 times a year in patients who are meeting treatment goalsand who have stable glycemic control. This patient is more than 12 months past due for a follow-up A1C test.For her, the A1C test should be performed quarterly until her goal of �7% is achieved or as close to 6% withoutsignificant hypoglycemia. A1C levels do not fully reflect a change in glucose levels for approximately 3 months and, therefore, bimonthly measurement could be excessive and results in unnecessary additions of drugs or dose titrations.

Bibliography1. Summary of Revisions for the 2008 Clinical Practice Recommendations. Diabetes Care 2008;31:S3–S54.

Item 2 True Answers A, B, C, E; False Answer B

Since this patient has not seen a physician for at least one year, she requires her annual dilated and comprehensiveeye exam by an ophthalmologist or optometrist as well as a foot exam (ideally by a podiatrist) because she hasdeveloped loss of pain sensation in both feet. The ADA also recommends an annual test to assess urine albumin in all patients with type 2 diabetes starting at diagnosis. The preferred method of screening for microalbuminurea is a random spot collection for determination of a protein to creatinine ratio. A 24-hour or time collections are more burdensome and add little to prediction or accuracy. A spot urine for albumin only is less expensive butsusceptible to false readings. Creatinine and calculated glomerular filtration rate (GFR) should also be determinedannually using the MDRD equation or Cockroft-Gault equation. Tables 1 and 2 provide the definitions for abnormalities of albumin excretion and kidney disease. Medical nutritional therapy should be the cornerstone oftreatment for patients with type 2 diabetes mellitus and is necessary for patients that are not achieving A1C targets.Referral to a dietition is part of the Standard of Care.

Although this patient most likely has elevated liver enzymes due to hepatic steatosis, a liver ultrasound is not recommended unless she has more severe hepatic enzyme elevation or abnormal symptoms.

Table 1. Definitions of Abnormalities in Albumin Excretion

Category Spot collection (µg/mg creatinine)

Normal �30

Microalbuminuria 30–299

Macro (clinical)-albuminuria �300

Page 57: ADASAP2 Reduced PDF

55Educational (Learning) Critiques American Diabetes Association Self-Assessment Program

Table 2. Stages of Chronic Kidney Disease

* Kidney damage defined as abnormalities on pathologic, urine, blood, or imaging tests.

1. Levey AS, et al: National Kidney Foundation practice guidelines for chronic kidney disease: evaluation, classification, and stratification. Ann Intern Med 2003;139:137–147.

Bibliography1. Summary of Revisions for the 2008 Clinical Practice Recommendations. Diabetes Care 2008;31:S3–S54.

Item 3 True Answers A, D, E; False Answers B, C

Carbohydrate intake monitoring is an important component of medical nutrition therapy for patients with type 2diabetes mellitus. Effective strategies include carbohydrate counting, exchanges, or experience-based estimations.The use of the glycemic index and glycemic load may provide modest additional benefits for glycemic control overthat observed when total carbohydrates are considered alone. Saturated fat intake should be �7% of calories in thediet. Routine supplementation with antioxidants such as vitamins E and C and carotene or chromium supplemen-tation are not advised because of lack of evidence of efficacy and concern related to long-term safety.

Bibliography1. Summary of Revisions for the 2008 Clinical Practice Recommendations. Diabetes Care 2008;31:S3–S54.

Item 4 True Answers A, B, C, D, E

According to the ADA Guidelines, all of these medications/treatments would be recommended. Even though herLDL-C is �100 mg/dL (a goal of therapy), her non-HDL remains �130 mg/dL (a secondary goal of therapy iftriglycerides are �200 mg/dL). In addition, the ADA Guidelines also recommend that statin therapy should beadded to lifestyle therapy regardless of baseline lipid levels for patients with diabetes with cardiovascular disease(CVD) or without CVD who are the age of �40 and have one or more other CVD risk factors. Since this patient is over age 40, has a low HDL level and elevated blood pressure, and an ALT that is slightly above the upper limit of normal, a statin should be initiated to lower LDL-C level by 40% or more from the baseline level. In patients with type 2 diabetes and risk factors, a mild baseline elevation in LFT is not a reason for withholding a statin.She also has hypertension and, therefore, her blood pressure regimen should include either an ACE inhibitor or an angiotensin receptor blocker (ARB). If one class is not tolerated, the other should be substituted. Aspirintherapy (75–162 mg/dL) as a primary prevention strategy is recommended in patients with type 2 diabetes mellitusover age 40 who have other risk factors. Patients with type 2 diabetes mellitus should have the influenza vaccineannually and at least one lifetime pneumococcal vaccine should be administered. A one-time revaccination isrecommended for individuals �65 years of age previously immunized when they were �65 years of age if thevaccine was administered �5 years ago.

Bibliography1. Summary of Revisions for the 2008 Clinical Practice Recommendations. Diabetes Care 2008;31:S3–S54.

GFR (ml/min per 1.73 m2

Stage Description body surface area)

1 Kidney damage* with normal or increased GFR 90

2 Kidney damage* with mildly decreased GFR 60–89

3 Moderately decreased GFR 30–59

4 Severely decreased GFR 15–29

5 Kidney failure �15 or dialysis

Page 58: ADASAP2 Reduced PDF

American Diabetes Association Self-Assessment Program Educational (Learning) Critiques56

Item 5 True Answers A, B, C, D; False Answer E

The ADA A1C goal is a minimum of �7%. The blood pressure goal is �130/80 mm Hg. Since she has renalimpairment, consideration should be given to blood pressure goal of �120/80 mm Hg. The lipid goals over LDL �100 mg/dL (the primary target with statin) and the desirable levels for triglycerides is �150 mg/dL and HDL �40 mg/dL for men but �50 mg/dL for women.

Bibliography1. Summary of Revisions for the 2008 Clinical Practice Recommendations. Diabetes Care 2008;31:S3–S54.

Item 6 True Answers B, C, D, E; False Answer A

Since her creatinine level is �1.5 mg/dL, metformin should be discontinued due to increased risk of developinglactic acidosis. Options B, C, D, and E are all potential considerations to improve her elevated A1C level but sinceshe has renal impairment, there should be careful consideration of dosage adjustment for sitagliptin and exenatidebecause both drugs have significant renal clearance.

Bibliography1. Summary of Revisions for the 2008 Clinical Practice Recommendations. Diabetes Care 2008;31:S3–S54.

Item 7 True Answers A, B, D; False Answers C, E

Gemfibrozil will increase the AUC (area under the curve) by simvastatin and most often statins by 2–3 fold and,therefore, combination with a statin should be utilized cautiously and the statin dose should not exceed 10 mg forsimvastatin and rosuvastatin as stated in the package insert. Gemfibrozil is also a cytochrome P4502C8 inhibitor in the major metabolic pathway for repaglinide and rosiglitazone and will, therefore, increase the hypoglycemiceffect of these drugs. There have been case reports of gemfibrozil and repaglimide combination inducing severehypoglycemia.

Fenofibrate does not increase the AUC for statins. Therefore, fenofibrate has a much lower propensity to increase the risk of myopathy in combination with a statin and there are no dosage restrictions with statins, but the combination should still be used with caution. Fenofibrate, more than gemfibrozil, can increase creatinine and homocysteine. The clinical relevance of this effect is unknown. Fenofibrate is significantly renally metabolizedand, therefore, dosage adjustment is necessary in patients with severe renal impairment.

Bibliography1. Davidson MH. Statin/fibrate combination in patients with metabolic syndrome or diabetes: evaluating the risks of pharmacokinetic

drug interactions. Expert Opin Drug Saf 2006;5:145–156.2. Davidson MH, et al: Safety considerations with fibrate therapy. Am J Cardiol 2007;19;99(6A):3C–18C.3. Davidson MH, et al: Safety of aggressive lipid management. J Am Coll Cardiol 2007;49:1753–62.

Item 8 True Answers A, C, D; False Answers B, E

In patients with type 2 diabetes mellitus, hypertension and microalbuminurea, both ACE inhibitors and ARBs havebeen shown to delay the progression to microalbuminurea. However, the recent On Target Trial showed that bothdrugs (ACE and ARBs) used in combination were no better than each drug used alone on preventing CVD events.

For patients with type 2 diabetes mellitus, the blood pressure goal is �130/80 mm Hg. This patient needs moreaggressive blood pressure control especially in light of her renal impairment. Thiazide diuretics should be added to ACE or ARBs if needed to achieve blood pressure control with an estimated glomerular filtration rate (GFR)�50 ml/min per 1.73 m2, but in patients with GFR �50 ml/min per 1.73 m2, (such as this patient), a loop diureticshould be used instead. Reduction in protein intake is recommended for patients with diabetes and the earlieststages of chronic kidney disease. The ADA recommends continued monitoring of urine albumin excretion toassess both response to therapy and progression of disease.

Page 59: ADASAP2 Reduced PDF

57

Bibliography1. Summary of Revisions for the 2008 Clinical Practice Recommendations. Diabetes Care 2008;31:S3–S54.

Item 9 True Answers A, B; False Answers C, D, E

Diabetes is the leading cause of blindness in the United States. Optimizing both glycemic control and bloodpressure has been demonstrated to reduce the incidence and progression of retinopathy. The ADA recommendsdilated eye exams by an ophthalmologist or optometrist at diagnosis and annually thereafter for patients with type 2 diabetes mellitus.

The presence of retinopathy is not a contraindication to aspirin therapy for cardioprotection, as this therapy doesnot increase the risk of retinal hemorrhage. Two large trials, the Diabetic Retinopathy Study (DRS) and the EarlyTreatment Diabetes Retinopathy Study (ETDRS), provide strong support for the therapeutic benefits of photoco-agulation surgery. The DRS showed that panretinal photocoagulation surgery reduces the risk of severe vision lossfrom retinopathy from 15.9% in untreated eyes to 6.4% in treated eyes. Laser photocoagulation therapy is indicatedto reduce the risk of vision loss in patients with high risk proliferative diabetic retinopathy, clinically significantmacular edema, and in some cases of severe non-proliferative diabetic retinopathy.

Bibliography1. Summary of Revisions for the 2008 Clinical Practice Recommendations. Diabetes Care 2008;31:S3–S54.

Item 10 True Answers D, E; False Answers A, B, C

Patients with diabetes should be screened annually for distal symmetric polyneuropathy (DPN) using tests such aspin-prick sensation, vibration perception with a tuning fork, 10 g monofilament pressure at the distal planter aspectof both great toes and metatarsal joints, and assessment of ankle reflexes. These tests, recommended annually, are87% sensitive to diagnose neuropathy. Electrophysiological testing is rarely needed except in situations whereclinical features are atypical.

The first step in management of patients with DPN should be to aim for stable and optimal glycemic control.Although controlled trial evidence is lacking, several observational studies suggest that neuropathic symptomsimprove not only with optimization of control, but also with the avoidance of extreme blood glucose fluctuations.Patients with painful DPN may benefit from pharmacological treatment of their symptoms; many agents haveefficacy confirmed in published randomized control trials, with several FDA-approved for the management ofpainful DPN. Refer patients who smoke, have loss of protective sensation and structural abnormalities, or havehistory of prior lower-extremity complications to foot care specialists for ongoing preventive care and life-longsurveillance.

Initial screening for peripheral arterial disease (PAD) should include a history for claudication and an assessment of the pedal pulses. A diagnostic ABI should be performed in any patient with symptoms of PAD. Due to the highestimated prevalence of PAD in patients with diabetes and the fact that many patients with PAD are asymptomatic,an ADA consensus statement on PAD suggested that a screening ABI be performed in patients older than 50 yearsof age and be considered in patients younger than 50 years who have other PAD risk factors (e.g., smoking, hyper-tension, hyperlipidemia, or duration of diabetes �10 years). Refer patients with significant symptoms or a positiveABI for further vascular assessment, and consider exercise, medication, and surgical options.

Diabetic autonomic neuropathy is also associated with genitourinary tract disturbances. In men, diabeticautonomic neuropathy may cause erectile dysfunction and/or retrograde ejaculation. Evaluation of bladderdysfunction should be performed for individuals with diabetes who have recurrent urinary tract infections,pyelonephritis, incontinence, or a palpable bladder.

Bibliography1. Summary of Revisions for the 2008 Clinical Practice Recommendations. Diabetes Care 2008;31:S3–S54.

Educational (Learning) Critiques American Diabetes Association Self-Assessment Program

Page 60: ADASAP2 Reduced PDF

American Diabetes Association Self-Assessment Program Educational (Learning) Critiques58

Items 11–12 11) True Answers A, B, D, E; False Answer C 12) True Answers A, B, C, D; False Answer E

The HEDIS 2008 NCQA guidelines are used to evaluate physician performance and are increasingly beingutilized to institute “pay for performance.” The HEDIS 2008 NCQA guidelines are listed below:

The percentage of patients 18–75 years of age with diabetes (type 1 and type 2) who had each of the following:

� Hemoglobin A1C (HbA1C) testing.

� HbA1C poor control (�9.0%).

� HbA1C good control (�7.0%).

� Eye exam (retinal) performed.

� LDL-C screening.

� LDL-C control.

� Medical attention for nephropathy.

� Blood pressure control.

� Foot examination.

� Smoking status and cessation advice or treatment.

Triglycerides �150 mg/dL and HDL �40 mg/dL are desirable levels recommended by the ADA but are not part ofthe HEDIS 2008 performance measures.

Bibliography1. Summary of Revisions for the 2008 Clinical Practice Recommendations. Diabetes Care 2008;31:S3–S54.

Item 13 True Answers A, B, C, D; False Answer E

Insulin is a potential second line agent to treat hyperglycemia after metformin has failed to achieve goal A1C levelsin patients with type 2 diabetes mellitus. Advantages include no dose limit, inexpensive, and improved lipid profile.Disadvantages are the injections, required monitoring, hypoglycemia, and weight gain. Insulin in combination withglitazones may increase the risk of fluid retention. Insulin therapy has beneficial effects on peripheral insulin sensitivity.

Bibliography1. Summary of Revisions for the 2008 Clinical Practice Recommendations. Diabetes Care 2008;31:S3–S54.

Item 14 True Answers A, B, D; False Answers C, E

Thiazolidinediones (TZDs) or glitazones are also second line agents after metformin to improve hyperglycemia.Pioglitazone decreases triglyceride (TG) levels and raises HDL-C levels and therefore improves the lipid profile.Rosiglitazone does not decrease TG levels and raises HDL-C levels to a lesser degree than pioglitazone. Onlyrosiglitazone has a black box warning for increasing risk of MI, while both pioglitazone and rosiglitazone have a black box warning for increasing the risk of CHF.

Both pioglitazone and rosiglitazone have been shown to increase the risk of fractures associated with trauma but notvertebral fractures. Both glitazones are mostly hepatically metabolized, and dosage adjustment in patients with renalimpairment is not necessary.

Bibliography1. Summary of Revisions for the 2008 Clinical Practice Recommendations. Diabetes Care 2008;31:S3–S54.

Page 61: ADASAP2 Reduced PDF

59

Item 15 True Answers A, B; False Answers C, D, E

Sulfonylureas are the least expensive of the oral hypoglycemics and may be used second line after metformin butcan cause weight gain and hypoglycemia. Sulfonylureas also appear to decrease β-cell survival and have not beenshown to reduce CV events. Dosage adjustment is necessary for patients with renal impairment.

Bibliography1. Summary of Revisions for the 2008 Clinical Practice Recommendations. Diabetes Care 2008;31:S3–S54.

Item 16 Answer D

This patient has limited options available given his comorbid conditions of congestive heart failure and renal insuf-ficiency. There is an increased risk of lactic acidosis in patients with both of these conditions (though there is a rareincidence overall). Creatinine levels of 1.5 mg/dL in men (1.4 mg/dL in women) is the recommended thresholdabove which metformin should be discontinued, given the reduced clearance of the medication. With the patienttaking medications for heart failure, both metformin and pioglitazone would be inappropriate. The thiazolidine-diones (TZDs) are contraindicated in patients with class III/IV NYHA heart failure. For those with NYHA class IIheart failure, pioglitazone may be considered, but should be used with caution at low dose and increased gradually.The TZDs may increase fluid retention and precipitate heart failure. Despite the risk of heart failure known,it remains unclear what the overall risk of cardiovascular-related death is.

As the patient has an A1C level above goal (despite being on two additional oral agents prior to admission),additional therapy is needed beyond glyburide alone. The addition of insulin is the best option to assist the patientin reaching the recommended goal of 7.0%. With regard to continuing or discontinuing the sulfonylurea whenstarting insulin, there may be some debate.

Bibliography1. Riddle MC. Timely initiation of basal insulin. Am J Med 2004 Feb 2;116 Suppl 3A:3S–9S.2. Nathan DM, et al: Management of hyperglycemia in type 2 diabetes: a consensus algorithm for the initiation and adjustment of therapy:

update regarding thiazolidinediones: a consensus statement from the American Diabetes Association and the European Association forthe Study of Diabetes. Diabetes Care 2008 Jan;31(1):173–175.

3. Lago RM, et al: Congestive heart failure and cardiovascular death in patients with prediabetes and type 2 diabetes given thiazolidine-diones: a meta-analysis of randomised clinical trials. Lancet 2007 Sep 29;370(9593):1129–1136.

4. Lincoff AM, et al: Pioglitazone and risk of cardiovascular events in patients with type 2 diabetes mellitus: a meta-analysis of randomizedtrials. JAMA 2007 Sep 12;298(10):1180–1188.

5. DePalo VA, et al: Lactic acidosis. Lactic acidosis associated with metformin use in treatment of type 2 diabetes mellitus.Geriatrics 2005 Nov;60(11):36, 39–41.

6. Yki-Järvinen H. Combination Therapies with Insulin in type 2 Diabetes. Diabetes Care 2001 Jan;24:758–767.7. Johnson JL, et al: Efficacy of insulin and sulfonylurea combination therapy in type II diabetes. A meta-analysis of the randomized

placebo-controlled trials. Arch Intern Med 1996 Feb 12;156(3):259–264.

Item 17 Answer D

This patient’s blood glucose record and history indicate some hypoglycemia, especially in the middle of the nightand early morning. To correct this, she needs a better balance of her scheduled insulin. Most patients take 50% oftotal daily insulin as basal insulin (glargine), and 50% in bolus form (aspart) before meals (Figure 1). However, thispatient only receives 12 U total before meals (29% of total insulin). Possibly, her hypoglycemia, which also occursduring the day (per log) and during the night may be the result of too much glargine. A lower amount of glarginewould be worth pursuing to quickly address this problem. Following this adjustment, further monitoring would beappropriate to establish new trends in glycemia. Then, the premeal aspart levels may be increased accordingly witha more appropriate basal/bolus insulin balance.

Educational (Learning) Critiques American Diabetes Association Self-Assessment Program

Page 62: ADASAP2 Reduced PDF

American Diabetes Association Self-Assessment Program Educational (Learning) Critiques60

Figure 1. Examples of Physiologic Insulin Delivery Regimen

A. Once-daily glargine with lispro or aspart (shown in a ratio of 50:50) allows patients to skip meals or change mealtimes. Insulins lispro and aspart (rapid acting)are prandial insulins and glargine (long acting) is a basal insulin. This regimen is easier to use since it has true basal and prandial insulins. Dashed lineindicates the effective duration of glargine continuing through the following day. Glargine achieves steady state at approximately 2 hours.

B. Intermediate-acting neutral protamine Hagedorn (isophane insulin; NPH) and Lente (insulin zinc) are basal insulins. Rapid-acting lispro and aspart insulins are prandial insulins. This regimen (shown in a ratio of 50:50) is more difficult to adjust because NPH can act as both a basal and a prandial insulin. Dashed line indicates the effective duration of NPH or Lente continuing through the following day. Arrows indicate insulin injection.

DeWitt DE, et al: Outpatient insulin therapy in type 1 and type 2 diabetes mellitus: scientific review. JAMA May 7 2003;289(17):2254–2264.

Bibliography1. DeWitt DE, et al: Outpatient insulin therapy in type 1 and type 2 diabetes mellitus: scientific review.

JAMA 2003 May 7;289(17):2254–2264.2. Hirsch IB. Insulin analogues. N Engl J Med 2005 Jan 13;352(2):174–183.3. Devries JH, et al: Refining basal insulin therapy: what have we learned in the age of analogues?

Diabetes Metab Res Rev 2007 Sep;23(6):441–454.4. Hirsch IB. A Real-World Approach to Insulin Therapy in Primary Care Practice. Clinical Diabetes 2005;23:78–86.

Item 18 Answer B

The A1C level in this patient is well above the typical goal in therapy (7.0%). While her mean pre-prandial glucose levels are not significantly elevated, the postprandial levels are much more than the pre-meal values. Thepostprandial levels indicate substantial peaks from meals that basal insulin does not adequately address. The bestchange in therapy would be the addition of bolus insulin before meals to match these physiologic peaks in glucoselevels. While this does require the addition of multiple injections per day, it would most likely result in the patientachieving desired goal in glycemic control if she remains adherent to her prescribed therapy.

The addition of morning NPH insulin may provide a mid-afternoon peak, but would not assist in post-breakfastand post-dinner peaks in glucose levels. The addition of a second dose of glargine would increase insulin avail-ability throughout the day, and while this may reduce postprandial glucose levels, it would likely reduce fastinglevels further and increase the risk of hypoglycemia. This option, as well as the use of an alternative basal insulin(detemir), would not match the postprandial hyperglycemia as well in this situation. Increasing metformin wouldunlikely provide substantial benefit to overall glycemic control; and her current dose is close to maximum therapy(2500 mg/day).

Bibliography1. Hirsch IB. A Real-World Approach to Insulin Therapy in Primary Care Practice. Clinical Diabetes 2005;23:78–86.2. O’Keefe JH, et al: Postprandial hyperglycemia/hyperlipidemia (postprandial dysmetabolism) is a cardiovascular risk factor.

Am J Cardiol 2007 Sep 1;100(5):899–904.3. Hirsch IB. Intensifying insulin therapy in patients with type 2 diabetes mellitus. Am J Med 2005 May;118 Suppl 5A:21S–26S.

Breakfast Lunch Supper Bedtime

Lisproor Aspart

Lisproor Aspart

Lisproor Aspart

Glargine

6 am

8 10 2 4 6 8 10 12 am

2 4 6 am

12 p

m

Insu

lin E

ffect

Breakfast Lunch Supper Bedtime

Lisproor Aspart

Lisproor Aspart

NPH or LenteNPH or LenteLispro or Aspart

6 am

8 10 2 4 6 8 10 12 am

2 4 6 am

12 p

m

Insu

lin E

ffect

Page 63: ADASAP2 Reduced PDF

61

Item 19 Answer A

Individuals with A1C levels above goal but having fasting plasma glucose levels in the target range may be havingsignificant postprandial hyperglycemia. While there are no clear guidelines regarding the recommended measuresfor postprandial hyperglycemia, it is a very reasonable target of treatment for those with A1C levels at goal andfasting glucose levels in the target range. On one hand, there is great interest in improving therapy to aggressivelyreduce A1C levels to reduce the risk of diabetic complications. However, some clinicians are reluctant to do thisgiven concerns for hypoglycemia.

Unfortunately, the evidence remains unclear at this time regarding the true clinical impact of treating postprandialhyperglycemia. There is much epidemiologic evidence that shows associations between glucose levels two hoursafter an oral challenge and cardiovascular risk. This includes the Hoorn Study, the Honolulu Heart Study, theChicago Heart Study, and the DECODE study (and evidence is supported by meta-analyses). There is also othersupporting evidence that hyperglycemia during a myocardial infarction or stroke leads to a worse prognosis.However, there have been few studies evaluating the specific treatment of postprandial hyperglycemia, and manyinclude surrogate markers for atherosclerosis. Data are lacking on long-term studies (especially randomized,controlled trials) that clearly demonstrate that effective treatment of postprandial hyperglycemia will reduce cardiovascular events and other outcomes without increasing mortality.

Still, in the meantime, there are pathophysiological explanations for the potential impact of hyperglycemia onvascular disease. Studies show that hyperglycemia affects oxidative stress markers and endothelial function.Possibly, the combination of postprandial hyperglycemia and hyperlipidemia may represent a stronger risk factortogether for cardiovascular disease than each independently.

Bibliography1. Standards of medical care in diabetes—2008. Diabetes Care 2008 Jan;31 Suppl 1:S12–54.2. Ceriello A. Postprandial hyperglycemia and diabetes complications: is it time to treat? Diabetes 2005 Jan;54(1):1–7.3. Parkin CG, et al: Is Postprandial Glucose Control Important? Is It Practical In Primary Care Settings? Clinical Diabetes 2002;20(2):71.4. O’Keefe JH, et al: Postprandial hyperglycemia/hyperlipidemia (postprandial dysmetabolism) is a cardiovascular risk factor.

Am J Cardiol 2007 Sep 1;100(5):899–904.5. de Vegt F, et al: Hyperglycaemia is associated with all-cause and cardiovascular mortality in the Hoorn population: the Hoorn Study.

Diabetologia 1999 Aug;42(8):926–931.6. Donahue RP, et al: Postchallenge glucose concentration and coronary heart disease in men of Japanese ancestry. Honolulu Heart

Program. Diabetes 1987 Jun;36(6):689–692.7. Lowe LP, et al: Diabetes, asymptomatic hyperglycemia, and 22-year mortality in black and white men. The Chicago Heart Association

Detection Project in Industry Study. Diabetes Care 1997 Feb;20(2):163–169.8. Glucose tolerance and mortality: comparison of WHO and American Diabetes Association diagnostic criteria. The DECODE study

group. European Diabetes Epidemiology Group. Diabetes Epidemiology: Collaborative analysis of Diagnostic criteria in Europe.Lancet 1999 Aug 21;354(9179):617–621.

Item 20 Answer C

Hyperglycemia is common during acute illness and after major surgery. The hyperglycemia from these stressors hasa negative impact on outcome after cardiac surgery. Randomized clinical trials have shown that intensive treatmentof hyperglycemia in the surgical intensive care unit setting reduces morbidity and mortality. The clinical benefit ispresent independent of whether the patient has a history of previously diagnosed diabetes or the severity and typeof critical illness. Based on current evidence, it appears that the benefit comes from metabolic control rather thaninsulin dose itself. For this reason, surgical intensive care units commonly treat hyperglycemia aggressively with IVinsulin to keep glucose levels below 110 mg/dL. A sliding scale of regular insulin alone would not provide adequateglycemic control, and likely would increase hyper- and hypoglycemia during hospitalization. In fact, there is noevidence to support the use of sliding scale insulin therapy alone without long-acting insulin in the hospital setting.Next, daily basal insulin also would not provide adequate coverage to variability in glucose levels, and would onlybe adjusted once per day. Finally, oral agents and observation without treatment are not appropriate for thissituation, particularly to reach an aggressive goal of blood glucose level less than 110 mg/dL.

Educational (Learning) Critiques American Diabetes Association Self-Assessment Program

Page 64: ADASAP2 Reduced PDF

American Diabetes Association Self-Assessment Program Educational (Learning) Critiques62

Bibliography1. Van den Berghe GH. Role of intravenous insulin therapy in critically ill patients. Endocr Pract 2004 Mar–Apr;10 Suppl 2:17–20.2. Van den Berghe GH, et al: Intensive insulin therapy in the critically ill patients. N Engl J Med 2001 Nov 8;345(19):1359–1367.3. Ingels C, et al: Strict blood glucose control with insulin during intensive care after cardiac surgery: impact on 4-years survival,

dependency on medical care, and quality-of-life. Eur Heart J 2006 Nov;27(22):2716–2724.4. Clement S, et al: Management of diabetes and hyperglycemia in hospitals. Diabetes Care 2004 Feb;27(2):553–591.

Item 21 Answer E

The woman in this case has an average life expectancy of five years or less with known cardiovascular disease and comorbid conditions, and consequently has a high risk of additional cardiac event. She has an elevated risk of hypoglycemia on a sulfonylurea. In general, those older (above age 80) and on more than 5 medications are at greater risk. She also has a history of falls, and factors such as depression, poor mobility, as well as others(orthostasis, dementia, visual disturbance, neuropathy, and polypharmacy) increase the risk of future falls. Based on all of these factors together, tight glycemic control (e.g., goal of A1C below 7) may not be desirable, where therisks could outweigh the benefits. Based on guidelines from the American Geriatrics Society, a goal A1C of lessthan 8% may be reasonable to consider (thus, no change in therapy would be required). The other treatmentoptions may place her at greater risk of hypoglycemia, where risks would outweigh the benefits. In addition,she also has CHF class III that would preclude the use of pioglitazone.

From the perspective of the practicing clinician, diabetes complications, duration of diabetes, individual functionalstatus, and comorbid illnesses should all be considered in setting goals for glycemic therapy. In general, it is veryappropriate for providers to discuss with patients and assess individually what goals in therapy may be, dependingon patient life expectancy, burden of therapy, existing conditions, and patient preferences.

Bibliography1. Lee SJ, et al: Development and validation of a prognostic index for 4-year mortality in older adults. JAMA 2006 Feb 15;295(7):801–808.2. Olson DE, et al: Diabetes in older adults. Overview of AGS guidelines for the treatment of diabetes mellitus in geriatric populations.

Geriatrics 2004 Apr;59(4):18–24; quiz 25.3. http://prweb.com/releases/ADA/ACCORD/prweb681794.htm. Accessed March 17, 2008.4. Tinetti ME. Clinical practice. Preventing falls in elderly persons. N Engl J Med 2003 Jan 2;348(1):42–49.5. Shorr RI, et al: Incidence and risk factors for serious hypoglycemia in older persons using insulin or sulfonylureas.

Arch Intern Med 1997 Aug 11–25;157(15):1681–1686.6. Huang ES, et al: Self-reported goals of older patients with type 2 diabetes mellitus. J Am Geriatr Soc 2005 Feb;53(2):306–311.7. Huang ES, et al: The impact of functional status and comorbid illness on the expected benefits of intensive glucose control in older

diabetes patients. American Geriatrics Society Annual Scientific Meeting; 2007.8. http://professional.diabetes.org/News_Display.aspx?CID=60642&TYP=9. Accessed March 17, 2008.

Item 22 Answer B

Insulin therapy would be appropriate to reach this patient’s goal in glycemic management; however she hasconcerns of weight gain and alternative therapies are available to use. Exenatide, a glucagon-like peptide-1 mimetic,has multiple mechanisms for lowering glucose levels, including enhancement of insulin secretion, and is indicatedwith patients with type 2 diabetes who have β-cell function remaining. The medication is approved for use withmetformin and sulfonylureas and is under study for use with thiazolidinediones. Exenatide results in reduction of A1C level an average of 1%. It also has been shown to result in a small amount of weight loss. No studies havedemonstrated long-term effects of this newer therapy. One study showed comparable efficacy with glargine insulinfor those uncontrolled on metformin and sulfonylurea therapy; however, exenatide had greater withdrawal from thestudy (19.4% vs 9.7%) likely related to gastrointestinal side effects (57% of subjects on exenatide experiencednausea) (Figure 2).

Page 65: ADASAP2 Reduced PDF

63

Figure 2.

Time course for A1C level and body weight from week 0 to week 26 is shown for the exanatide group compared with the insulin glargine group; mean (± SE) is shown. Week 0 indicates baseline. Data are shown for intention-to-treat population. * P �0.0001 compared with insulin glargine measure at the same point.

Heine RJ,et al: Exenatide versus insulin glargine in patients with suboptimally controlled type 2 diabetes: a randomized trial. Ann Intern MedOct 18 2005;143(8):559–569.

Maintaining medication therapy at the current level is not appropriate in this patient; a goal of A1C of 7.0% or less remains most appropriate to prevent long-term complications. Acarbose can be considered, but would reduceA1C level only slightly. Pioglitazone is a reasonable option; however, given concern for weight gain, this might notbe as good a choice as exenatide. Finally, sibutramine has been shown to result in weight loss; however, it is unlikelyto have the same benefit in glycemic control. Other medications, such as orlistat and rimonabant, have been show to improve glycemic control in patients with diabetes, but implementation of these agents carries significant risks of other side effects.

Bibliography1. Amori RE, et al: Efficacy and safety of incretin therapy in type 2 diabetes: systematic review and meta-analysis.

JAMA 2007 Jul 11;298(2):194–206.2. Jones MC. Therapies for diabetes: pramlintide and exenatide. Am Fam Physician 2007 Jun 15;75(12):1831–1835.3. Heine RJ, et al: Exenatide versus insulin glargine in patients with suboptimally controlled type 2 diabetes: a randomized trial.

Ann Intern Med 2005 Oct 18;143(8):559–569.4. Comi RJ. Treatment of type 2 diabetes mellitus: a weighty enigma. Ann Intern Med 2005 Med Oct 18;143(8):609–610.5. Rucker D, et al: Long term pharmacotherapy for obesity and overweight: updated meta-analysis.

BMJ 2007 Dec 8;335(7631):1194–1199.6. DeFronzo RA, et al: Effects of exenatide (exendin-4) on glycemic control and weight over 30 weeks in metformin-treated patients

with type 2 diabetes. Diabetes Care 2005 May;28(5):1092–1100.

Items 23–25 Answers 23 (C); 24 (C); 25 (B)

This patient has been newly diagnosed with type 2 diabetes, and lifestyle modifications including diet and exerciseare necessary as they have clear benefit. This is always an appropriate first step in treating patients with type 2diabetes. The referral to a dietitian is also appropriate. However, despite these measures, lifestyle interventionsoften fail to achieve metabolic goals. For this reason, it would be reasonable to also start metformin at this time,and not wait longer to evaluate the impact of lifestyle changes. It would also have been appropriate to startmetformin three months prior upon diagnosis at the same time as the lifestyle intervention.

Metformin, in the absence of contraindications, is superior to other oral agents as it is less expensive, well studied,has a low level of side effects, and may be less likely to cause hypoglycemia or weight gain. Metformin offers goodpotential improvement in A1C level (approximately 1.5%). In comparison with pramlintide, alpha-glucosidaseinhibitors, and exenatide, metformin would most likely reduce A1C level the most as monotherapy (~1.5%; Table 3).Exenatide is FDA-approved for combination with sulfonylurea or metformin and may be considered as a secondagent in the future. It would require twice daily regular injections. Insulin would also be effective in reducing bloodglucose levels; however, an oral agent like metformin would remain the first line agent to use in this situation.

0 12 26

8.58.07.57.06.5

0.0

Hem

oglo

bin

A1C

Leve

l, %

Weeks 275 244 229 260 249 243

Exanatide group, nInsulin glargine group, n

Exanatide group (n = 275)Insulin glargine group (n = 260)

0 2 4 8 1812 26

8.58.07.57.06.5

0.0

Chan

ge in

Bod

y W

eigh

t, kg

Weeks 281 277 275 261 245 235 231 267 266 261 253 251 246 244

Exanatide group (n = 275)Insulin glargine group (n = 260)

* ** *

**

Educational (Learning) Critiques American Diabetes Association Self-Assessment Program

Page 66: ADASAP2 Reduced PDF

American Diabetes Association Self-Assessment Program Educational (Learning) Critiques64

Table 3. Summary of Glucose-lowering Interventions as Monotherapy

Nathan DM, et al. Management of hyperglycemia in type 2 diabetes: a consensus algorithm for the initiation and adjustment of therapy: update regarding thiazolidinediones: a consensus statement from the American Diabetes Association and the European Association for the Study of Diabetes. Diabetes Care Jan 2008;31(1):173–175.

The Diabetes Outcome Progression Trial (ADOPT) was a double blind, controlled trial of 4360 patients overapproximately 5 years which compared thiazolidinediones with metformin and glyburide in terms of maintainingglycemic control. The cumulative incidence of monotherapy failure was 15% with rosiglitazone, 21% withmetformin, and 34% with glyburide (Figure 3). Monotherapy failure was defined as fasting glucose of more than180 mg/dL. The relative improvement from rosiglitazone may, in part, be due to slowing the rate of loss of β-cellfunction. Rosiglitazone was associated with more weight gain and edema than the other agents, but with lessgastrointestinal side effects than metformin and less hypoglycemia than with glyburide. In addition, those partici-pants with a A1C level less than 7% was only 4% higher in the rosiglitazone group as compared with the metformingroup (40% vs 36%), the difference being of questionable significance. Overall, given side effects, cost, andevidence available, metformin remains the first choice for many patients as the initial therapy upon diagnosis of type 2 diabetes.

* Severe hypoglycemia is relatively infrequent with sulfonylurea therapy. The longer-acting agents (e.g. chlorpropamide and glibenclamide [glyburide]) are more likely to cause hypoglycemia than glipizide, extended-release glipizide, glimepiride, or gliclazide.

† Pioglitazone.‡ Rosiglitazone.§ Repaglinide is more effective at lowering A1C than nateglinide.GI Gastrointestinal.MI Myocardial infarction.

Expected Decrease Interventions in A1C (%) Advantages Disadvantages

Step 1: Initial

Lifestyle to decrease 1–2 Low cost, many benefits Fails for most in first yearweight and increase activity

Metformin 1–2 Weight neutral, inexpensive GI side effects, rare lactic acidosis

Step 2: Additional therapy

Insulin 1.5–3.5 No dose limit, inexpensive, Injections, monitoring, improved lipid profile hypoglycemia, weight gain

Sulfonylureas 1–2 Inexpensive Weight gain, hypoglycemia*

Thiazolidinediones 0.5–1.4 Improved lipid profile† Fluid retention, twofold increased(glitazones) Potential decreased risk of MI† risk of CHF, potential increased risk

of MI‡, atherogenic lipid profile,weight gain, expensive

Other drugsα-Glucosidase 0.5–0.8 Weight neutral Frequent GI side effects, inhibitors three times/day dosing, expensiveExenatide 0.5–1.0 Weight loss Injections, frequent GI side effects,

expensive, little experienceGlinides 1–1.5§ Short duration Three times/day dosing, expensive,

hypoglycemiaPramlintide 0.5–1.0 Weight loss Injections, three times/day dosing,

frequent GI side effects, expensive,little experience

Sitagliptin 0.5–0.8 Weight neutral Little experience, expensive

Page 67: ADASAP2 Reduced PDF

65

Figure 3. Kaplan-Meier Estimates of the Cumulative Incidence of Monotherapy Failure at 5 Years

Treatment was considered to have failed if a patient had a confirmed or adjudicated level of fasting plasma glucose of more than 180 mg per deciliter. Riskreduction is listed for comparisons of pairwise groups from a baseline covariate-adjusted Cox proportional-hazards model. Gray’s estimates of cumulative incidenceadjusted for all deaths were smaller than Kaplan–Meier estimates of treatment failure: 10% in the rosiglitazone group, 15% in the metformin group, and 25% in the glyburide group. I bars indicate 95% CIs.

Kahn SE, et al: Glycemic durability of rosiglitazone, metformin, or glyburide monotherapy. N Engl J Med Dec 7 2006;355(23):2427–2443.

Bibliography1. Nathan DM, et al: Management of hyperglycemia in type 2 diabetes: a consensus algorithm for the initiation and adjustment of therapy:

update regarding thiazolidinediones: a consensus statement from the American Diabetes Association and the European Association for the Study of Diabetes. Diabetes Care 2008 Jan;31(1):173–175.

2. Kahn SE, et al: Glycemic durability of rosiglitazone, metformin, or glyburide monotherapy.N Engl J Med 2006 Dec 7;355(23):2427–2443.

3. Nathan DM. Thiazolidinediones for initial treatment of type 2 diabetes? N Engl J Med 2006 Dec 7;355(23):2477–2480.

Item 26 Answer A

Sitagliptin, a dipeptidyl peptidase-4 (DPP-4) inhibitor, is a new oral agent for the treatment of patients with type 2diabetes. It increases levels of incretins, glucagon-like peptide 1 (GLP-1) and glucose-dependent insulinotropicpeptide (GIP) to reduce glucose levels. Studies have demonstrated efficacy of sitagliptin as monotherapy or incombination with metformin in lowering blood glucose. Markers of β-cell function improved with sitagliptintherapy. In general, the medication is well tolerated, has a neutral effect on weight, and has a low risk ofhypoglycemia or gastrointestinal side effects. Most of the studies have been less than one year in duration, and dataare lacking on long-term effects on the prevention or delay of diabetes complications.

Bibliography1. Karasik A, et al: Sitagliptin, a DPP-4 inhibitor for the treatment of patients with type 2 diabetes: a review of recent clinical trials.

Curr Med Res Opin 2008 Feb;24(2):489–496.2. Bonora E. Antidiabetic medications in overweight/obese patients with type 2 diabetes: drawbacks of current drugs and potential

advantages of incretin-based treatment on body weight. Int J Clin Pract Suppl 2007 Aug;(154):19–28.3. Amori RE, et al: Efficacy and safety of incretin therapy in type 2 diabetes: systematic review and meta-analysis.

JAMA 2007 Jul 11;298(2):194–206.

0 1 2 3 4 5

No. at Risk Rosiglitazone 1393 1207 1078 957 844 324 Metformin 1397 1205 1076 950 818 311 Glyburide 1337 1114 958 751 617 218

Years

10

20 Metformin

Rosiglitazone

Hazard Ratio (95% CI) Rosiglitazone vs metformin 0.68 (0.55–0.85); P �0.001 Rosiglitazone vs glyburide 0.37 (0.30–0.45); P �0.001

30

40

0

Glyburide

Cum

ulat

ive

Inci

denc

e

of M

onot

hera

py F

ailu

re (%

)

Educational (Learning) Critiques American Diabetes Association Self-Assessment Program

Page 68: ADASAP2 Reduced PDF

American Diabetes Association Self-Assessment Program Educational (Learning) Critiques66

Item 27 Answer C

This patient has elevated fasting and postprandial glucose levels based on his detailed log. However, thepostprandial levels are not far from goal, and the fasting levels are consistently elevated, and there is no evidence byhistory or log of hypoglycemia. To substantially improve the fasting glucose levels, the basal insulin dose should beincreased. In general, the split between daily basal insulin and pre-meal very short-acting insulin is approximately50%/50%. In this situation, the basal insulin constitutes 1/3 of the total daily insulin used, so it would be reasonableto increase. Increasing aspart insulin would improve postprandial levels; however, this is not as great a problem asthe fasting levels in this situation. In general, lowering the amount of either type of insulin, in the absence ofhypoglycemia, would worsen glycemic control and raise A1C levels. NPH insulin in the morning would improveafternoon glucose levels, but not improve the rest of the day substantially.

Bibliography1. Devries JH, et al: Refining basal insulin therapy: what have we learned in the age of analogues?

Diabetes Metab Res Rev 2007 Sep;23(6):441–454.2. Hirsch IB. Intensifying insulin therapy in patients with type 2 diabetes mellitus. Am J Med 2005 May;118 Suppl 5A:21S–26S.

Item 28 Answer B

This patient has evidence consistent with non-alcoholic steatohepatitis (NASH). In general, weight loss is appropriate for this patient (approximately 1–2 kg/week is reasonable). Low-fat diets should be avoided, and maybe associated with greater inflammation of the liver. Instead, low glycemic index food (or the Mediterranean diet)has been recommended for patients with type 2 diabetes with non-alcoholic fatty liver disease.

Various oral agents have been tested in patients with both type 2 diabetes and NASH. Oral medications that mayimprove insulin resistance may be preferable for this patient with NASH. Metformin may be a reasonable choice,but there would be an increased risk of lactic acidosis with renal insufficiency (progressive liver disease may furtherincrease this risk). As an alternative, pioglitazone, which also addresses insulin resistance, is a viable alternative.Some studies have shown improvement in ALT and liver histology with thiazolidinediones (TZDs). Still, it remainsimportant to consider weight gain and to monitor transaminase levels after initiation of medication, especially givenprevious hepatotoxicity of troglitazone (withdrawn from the U.S. market). In addition, it should also be noted thatthe glitazones are associated with increased risk of fluid retention, congestive heart failure exacerbation, andfractures in general.

As for other therapeutic options, sulfonylureas could be used with patients with liver disease, though may be not bepreferable to TZDs. Patients on sulfonylureas should be monitored closely for hypoglycemia. Those with moreadvanced liver disease may have reduced gluconeogenesis and ability to counteract hypoglycemia. Finally, insulin,while safe to use, should be reserved if oral agents have failed. It also would have greater risk of hypoglycemia andthe dosage is frequently more challenging.

Bibliography1. Tolman KG, et al: Spectrum of liver disease in type 2 diabetes and management of patients with diabetes and liver disease.

Diabetes Care 2007 Mar;30(3):734–743.2. Kang H, et al: Metabolic syndrome is associated with greater histologic severity, higher carbohydrate, and lower fat diet in patients

with NAFLD. Am J Gastroenterol 2006 Oct;101(10):2247–2253.3. Angelico F, et al: Drugs improving insulin resistance for non-alcoholic fatty liver disease and/or non-alcoholic steatohepatitis.

Cochrane Database Syst Rev 2007(1):CD005166.

Page 69: ADASAP2 Reduced PDF

67

Item 29 Answer D

This patient has uncontrolled type 2 diabetes with A1C well above 7.0%. The addition of a thiazolidenedionewould result in only a small expected improvement in A1C of approximately 1%, which would not likely result in reaching goal in therapy. Increasing metformin to 2500 mg (maximum daily dose) offers very limit benefit over 2000 mg/day therapy. Repeating the A1C test and continuing therapy would only prolong his time havingelevated glucose levels, and would contribute to clinical inertia (the problem where therapy is not intensified at an office visit when opportunity exists). He reports good adherence to his therapy, and there is no acute probleminterfering with making a change in therapy.

Instituting insulin therapy at this point is the best option. This patient has had type 2 diabetes for a significantamount of time and may be having β-cell failure (Figure 4). In the UKPDS study, 55% of newly diagnosed patients with type 2 diabetes allocated to treatment with sulfonylurea required insulin to maintain glycemic control duringsix years following diagnosis. In addition, early intensive insulin therapy offers benefits beyond glycemic control,including preservation of β-cell function. With a possible loss in β-cell function and difference of 2.2% above goal, exenatide would unlikely help him reach his glycemic goal, and would not mitigate the need for insulininjection therapy.

Figure 4.

Progressive β-cell failure from 0 to 6 years from diagnosis represents data from the UKPDS group population and was determined by the homeostatic assessmentmodel. These data were extrapolated both forward and backward in time. PPBG = postprandial blood glucose; FBG = fasting blood glucose; T2DM = type 2 diabetesmellitus; IGT = impaired glucose tolerance.

Garg SK, et al: Practical Strategies for Introducing Insulin Therapy in 2006. Supplement to the Journal of Family Practice. April, 2006.

Bibliography1. Garg SK, et al: Practical Strategies for Introducing Insulin Therapy in 2006. Supplement to the Journal of Family Practice. April, 2006.2. Lebovitz HE. Insulin secretagogues: old and new. Diabetes Reviews 1999;7:139–153.3. Nathan DM, et al: Management of hyperglycemia in type 2 diabetes: A consensus algorithm for the initiation and adjustment of therapy:

a consensus statement from the American Diabetes Association and the European Association for the Study of Diabetes.Diabetes Care 2006 Aug;29(8):1963–1972.

PPBG

FBG

Years From Diagnosis

β-ce

ll Fu

nctio

n (%

)

-120

25

50

75

100

-10 -6 -2 0 2 6 10 14

Postprandialhyperglycemia

T2DMdiagnosis

T2DMphase II

T2DMphase III

IGT

Educational (Learning) Critiques American Diabetes Association Self-Assessment Program

Page 70: ADASAP2 Reduced PDF

American Diabetes Association Self-Assessment Program Educational (Learning) Critiques68

Item 30 Answer B

In the pregnant patient with type 2 diabetes, insulin remains the best choice of therapy to provide intensiveglycemic control, especially in those unable to achieve target blood glucose levels with recommended diet andphysical activity. It would have been optimal for this patient to have been on insulin, and with better glycemiccontrol, prior to becoming pregnant. Tight glycemic control is necessary to reduce the risk of birth defects and fetal macrosomia and promote optimal health of both developing fetus and mother. NPH insulin plus multipleinjections of a rapidly-acting analog is the best combination in that it includes better studied insulin formulationsand is more likely to result in tightly controlled blood glucose levels. The long-acting insulins detemir and glargineare Category C in pregnancy. Lack of large randomized controlled trials remains a limitation. Also, daily premixedinsulin 70/30 is not the optimal choice of therapy, as this option does not provide flexibility in intensifying therapyto reach goals in blood glucose levels.

Continuing either of the oral agents (glyburide or pioglitazone) is less desirable, especially as these agents coupledwith glargine and detemir have not been studied in combination. It would seem that a glitazone might be desirableto address insulin resistance (especially during pregnancy). However, there are no clinical data available on its use in pregnancy; and, one study reported that rosiglitazone crossed the placenta in early human pregnancy at10–12 weeks. Finally, there is inadequate time available to monitor for improvement in glycemic control withadditional nutritional therapy and increased physical activity. Immediate changes in therapy are necessary,especially given the A1C level well above goal.

Bibliography1. Kitzmiller JL, et al: Managing preexisting diabetes for pregnancy: summary of evidence and consensus recommendations for care.

Diabetes Care 2008 May; 31(5):1060–79.

Item 31 Answer A

This patient has an A1C level above 7.0% (target). Her logbook reveals elevated postprandial blood glucose levels,which are most likely contributing to her overall suboptimal glycemic control and now elevated A1C level. Thespecific times of the day when postprandial glucose levels are elevated include 2 hours after breakfast and dinner.Repaglinide might be an appropriate option, to be taken before these two meals. The glinides are short-actinginsulin secretagogues, similar to sulfonylureas, that promote endogenous insulin secretion and affect early insulinrelease. This medication would help reduce postprandial glucose levels most efficiently, with low risk ofhypoglycemia, and can match the patient’s unique postprandial needs.

Increasing metformin to 1000 mg b.i.d. would unlikely improve blood glucose levels in a meaningful way, though it would help limit the number of medications she would need to take overall. However, additional metforminwould not adequately lower postprandial glucose levels as much as the addition of the glinide (which would helpincrease insulin secretion). Also, while increasing time exercising may be helpful in general and could reduceinsulin resistance, it is still preferable to intensify therapy immediately to avoid further clinical inertia (situationwhen providers are slow to react to needed changes in therapy). Certainly, increases in physical activity may berecommended in addition to medication changes. An alternative medication that may improve postprandial glucoselevels would be fast-acting insulin, such as lispro or aspart. However, this would require instituting insulin injec-tions which may not be necessary at this point. In addition, it increases the risk of hypoglycemia. The use of basalinsulin (glargine or detemir) would be less desirable, as this would also impact fasting glucose levels and increasethe risk of hypoglycemia.

Bibliography1. Gerich JE. Clinical significance, pathogenesis, and management of postprandial hyperglycemia.

Arch Intern Med 2003 Jun 9;163(11):1306–1316.2. Ceriello A. Postprandial hyperglycemia and diabetes complications: is it time to treat? Diabetes 2005 Jan;54(1):1–7.3. Parkin CG, et al: Is Postprandial Glucose Control Important? Is It Practical In Primary Care Settings? Clinical Diabetes 2002;20(2):71.4. O’Keefe JH, et al: Postprandial hyperglycemia/hyperlipidemia (postprandial dysmetabolism) is a cardiovascular risk factor.

Am J Cardiol 2007 Sep 1;100(5):899–904.

Page 71: ADASAP2 Reduced PDF

69

Item 32 Answer C

The patient is anticipating a heart catheterization with administration of contrast. This potentially would increasethe risk of lactic acidosis. The morning of the procedure, the patient should hold metformin, and then he may beable to restart it 48 hours after the procedure if renal function remains stable. Intravenous fluids are appropriate tomaintain hydration in this situation. If surgery or additional procedures are subsequently planned, the metforminshould remain held. Note that metformin should be held if his renal function deteriorates further (metformin iscontraindicated with creatinine level above 1.5 mg/dL in men and 1.4 mg/dL in women, with stage 4 chronickidney disease also being a contraindication based on estimated GFR). It is important to consider risk factors forlactic acidosis when using metformin particularly in the hospital, where patients may experience hypoperfusion,increased renal failure, and hypoxia. Yet, despite these concerns, metformin is relatively safe when appropriatelyprescribed. A Cochrane Review found no evidence from prospective comparative trials or observational cohortstudies that metformin is associated with increased risk of lactic acidosis, compared to other oral agents fortreatment of type 2 diabetes when prescribed under similar study conditions accounting for contraindications.Discontinuing pioglitazone is not necessary, as this medication is not associated with hypoglycemia (with a patientfasting for the procedure). Other therapies, such as insulin or sulfonylureas, may need to be modified if they arebeing utilized in a similar situation.

Bibliography1. Shaw JS, et al: Establishing pragmatic estimated GFR thresholds to guide metformin prescribing.

Diabet Med 2007 Oct;24(10):1160–1163.2. Salpeter S, et al: Risk of fatal and nonfatal lactic acidosis with metformin use in type 2 diabetes mellitus.

Cochrane Database Syst Rev 2006(1):CD002967.

Items 33–37 Answers 33 (A); 34 (B); 35 (D); 36 (E); 37 (C)

Figure 5. Algorithm for the Metabolic Management of Type 2 Diabetes. Reinforce Lifestyle Intervention at Every Visit.

a Check A1C every 3 months until �7% and then at least every 6 months. b Associated with increased risk of fluid retention, CHF, and fractures. Rosiglitazone, but probably not pioglitazone, may be associated with an increased

risk of myocardial infarction. c Although three oral agents can be used, initiation and intensification of insulin therapy is preferred based on effectiveness and lower expense.

Nathan DM, et al: Management of hyperglycemia in type 2 diabetes: a consensus algorithm for the initiation and adjustment of therapy: update regarding thiazolidinediones: a consensus statement from the American Diabetes Association and the European Association for the Study of Diabetes. Diabetes CareJan 2008;31(1):173–175.

Lifestyle Intervention + Metformin

Diagnosis

Add Sulfonylurea(Least expensive)

Add Glitazoneb

(No hypoglycemia)Add Basal Insulin(Most effective)

YesaNo A1C �7%

YesaNo A1C �7% YesaNo A1C �7% YesaNo A1C �7%

Intensify Insulin Add Glitazoneb,c Add Basal Insulin Add Sulfonylureac

YesaNo A1C �7%YesaNo A1C �7%

Add Basal or Intensify Insulin

Intensive Insulin + Metformin +/- Glitazoneb

Educational (Learning) Critiques American Diabetes Association Self-Assessment Program

Page 72: ADASAP2 Reduced PDF

American Diabetes Association Self-Assessment Program Educational (Learning) Critiques70

33. Patients newly diagnosed with type 2 diabetes should receive lifestyle interventions to modify diet, increasephysical activity, and promote weight loss. Small changes in weight loss (as low as 4 kg) will improve hyper-glycemia. However, there is limited success with such lifestyle programs, usually due to failure to lose weight(or weight regain) or progressive disease. For these reasons, metformin is currently recommended as initialtherapy at the same time lifestyle modifications are instituted. Metformin should be titrated over 1–2 monthsas tolerated. In addition, metformin may result in modest weight loss, in contrast to many other therapies thatresult in weight gain (especially sulfonylureas and insulin).

34. The patient has an elevated A1C level above goal (7%) and further change in therapy is appropriate. In thissituation, with cost being a potential concern, starting a sulfonylurea may be preferable to a thiazolidinedione.While the risk of hypoglycemia is introduced, severe episodes (those requiring need for assistance) tend tooccur less often, especially in his age group. Other oral medications, such as the glinides (e.g., repaglinide),could be considered as an alternative if hypoglycemia was a significant concern, depending on cost and avail-ability. It would be less desirable to start insulin injections at this point, especially basal insulin given likelypostprandial alterations.

35. The patient clearly has hyperglycemia that requires further therapy. Despite her age, she appears to have goodfunctional status without diabetes-related complications. It is appropriate to target an A1C level of 7.0%.Pioglitazone could be added; however, this would unlikely reduce her A1C level to her target goal. Instead,daily basal insulin should be instituted, and then insulin may be intensified as necessary.

36. This patient has a history of Alzheimer disease for six years, and given his age and life expectancy andcomorbid conditions, a higher target A1C level may be appropriate (such as 8.0% or less). Pioglitazone wouldnot be indicated due to congestive heart failure, and would not offer benefit in place of rosiglitazone. Heartfailure is also a relative contraindication for metformin given risk of lactic acidosis; however, a recent retro-spective review of patients in Canada found less morbidity and mortality by those with congestive heart failureon metformin when compared with sulfonylureas.

37. Her level of glycemic control is not much higher than goal (7.0%). A glitazone might be most appropriate in this situation, with the benefit of avoiding the risk of hypoglycemia, though it may take a month beforebecoming effective. Glipizide might cause less hypoglycemia than glyburide, but may not be the optimalchoice if a glitazone may be utilized. Basal insulin would also increase the risk of hypoglycemia. Metformin is contraindicated with the level of renal failure she currently has given the risk of lactic acidosis.

Bibliography 1. Nathan DM, et al: Management of hyperglycemia in type 2 diabetes: a consensus algorithm for the initiation and adjustment of therapy:

update regarding thiazolidinediones: a consensus statement from the American Diabetes Association and the European Association forthe Study of Diabetes. Diabetes Care 2008;31(1):173–175.

2. Kabadi UM. Cost-effective management of hyperglycemia in patients with type 2 diabetes using oral agents.Manag Care 2004 Jul;13(7):48–49, 53–46, 58–49.

3. Eurich DT, et al: Improved clinical outcomes associated with metformin in patients with diabetes and heart failure.Diabetes Care 2005 Oct;28(10):2345–2351.

Item 38 Answer B

Both the fasting glucose level and A1C level are both elevated, and instituting basal insulin therapy (detemir orglargine) would be the best option at this point. One potential advantage of detemir over other insulin formulationsis that it may be associated with less weight gain (especially when given once daily). When utilized, the dose ofinsulin can be increased to target fasting levels below approximately 120 mg/dL. Starting once daily NPH orpremixed insulin also would likely be somewhat effective in reducing fasting and average glucose levels; though in the treat to target studies, glargine and detemir daily insulin resulted in less hypoglycemia. The other options,while possibly reducing glucose levels in combination therapy, are less desirable. Pioglitazone would not signifi-

Page 73: ADASAP2 Reduced PDF

71

cantly further reduce glucose levels unless metformin was continued (while the patient in this case had micro-albuminuria, metformin may be continued in combination therapy with the current level of renal function).Pioglitazone potentially offers a positive impact on lipid profiles, has anti-inflammatory properties, and mayimprove endothelial function providing some advantage for those with higher coronary artery disease risk. Next,exenatide offers some benefit by reducing weight in this obese person while modestly improving glycemic control.However, this would be more appropriate in someone with persistently elevated postprandial glucose levels. Inaddition, it requires residual β-cell function that may not be present given the extended history of diabetes. Aspartinsulin with meals would also target postprandial glucose but this requires multiple injections throughout the day.

Bibliography1. Stojanovska L, et al: The anti-atherogenic effects of thiazolidinediones. Curr Diabetes Rev 2007 Feb;3(1):67–74.2. Rosenstock J, et al: A randomised, 52-week, treat-to-target trial comparing insulin detemir with insulin glargine when administered as

add-on to glucose-lowering drugs in insulin-naive people with type 2 diabetes. Diabetologia 2008 Mar;51(3):408–416.3. Meneghini LF, et al: Insulin detemir improves glycaemic control with less hypoglycaemia and no weight gain in patients with type 2

diabetes who were insulin naive or treated with NPH or insulin glargine: clinical practice experience from a German subgroup of thePREDICTIVE study. Diabetes Obes Metab 2007 May;9(3):418–427.

4. Hermansen K, et al: A 26-week, randomized, parallel, treat-to-target trial comparing insulin detemir with NPH insulin as add-on therapyto oral glucose-lowering drugs in insulin-naive people with type 2 diabetes. Diabetes Care 2006 Jun;29(6):1269–1274.

5. Riddle MC, et al: The treat-to-target trial: randomized addition of glargine or human NPH insulin to oral therapy of type 2 diabeticpatients. Diabetes Care 2003 Nov;26(11):3080–3086.

Items 39–41 Answers 39 (B); 40 (A); 41 (D)

The patient is at higher risk of hypoglycemia being of older age on a sulfonylurea. However, his prior glycemiccontrol is notable for absence of hypoglycemia on stable medication therapy. Recently, he was started on a fluoro-quinolone, and it is most likely that this combination of fluoroquinolone and sulfonylurea that increased his risk ofsevere hypoglycemia. Based on one study, gatifloxacin has a much greater risk of hypoglycemia than other agents(including other fluoroquinolones). The mechanism by which fluoroquinolones increase the risk of hypoglycemiais unclear, and likely complex and multifactorial.

As sulfonylureas are cleared by the kidneys, renal failure may lead to increased drug levels in the bloodstream.Providers must use sulfonylureas with caution in patients who may have chronic renal insufficiency or are at risk of developing acute renal disease. This may be related to an acute illness, poor oral intake, and/or dehydration;though this was not described in the case scenario. Adrenal insufficiency may also cause hypoglycemia; this couldbe a potential factor if the patient had been on long-standing prednisone for obstructive lung disease and thenabruptly stopped.

Appropriate management includes hospitalization since glyburide has a long half-life, and continuous infusion ofdextrose solution with frequent glucose level monitoring. Solumedrol is not necessary as adrenal insufficiency isless likely. Glucagon is appropriate for home therapy if patient has altered mental status and is unable to eat or drinkanything when experiencing hypoglycemia, but unnecessary when intravenous access is available. Octreotide hasbeen used successfully in select situations of sulfonylurea overdose when hypoglycemia is refractory to dextrosebolus/infusion.

Typical risk factors for iatrogenic hypoglycemia are based on insulin excess (from injected or secreted sources).These include situations when: insulin dose is excessive, such as with intensive insulin regimens, exogenousglucose delivery is decreased (for example, missed meal or snack), endogenous glucose production is decreased(alcohol ingestion), glucose utilization is increased (exercise), sensitivity to insulin is increased (insulin sensitizer),or insulin clearance is decreased (renal insufficiency). Longer history of diabetes and older age, degree of glycemiccontrol, and history of prior hypoglycemia all further increase the risk of developing future hypoglycemia.Hypoglycemia unawareness is thought to be related to reduced neurogenic symptom response to low bloodglucose levels. Neurogenic symptoms include adrenergic (tremulousness, palpitations) and cholinergic (sweating,hunger, and paresthesia) components. Exenatide, an incretin mimetic, is typically used with oral agents and not asmonotherapy, with an expected decrease in A1C level of up to 1%. However, a study of monotherapy withexenatide and a report of exenatide overdose showed no evidence of severe hypoglycemia.

Educational (Learning) Critiques American Diabetes Association Self-Assessment Program

Page 74: ADASAP2 Reduced PDF

American Diabetes Association Self-Assessment Program Educational (Learning) Critiques72

Bibliography1. Bussing R, et al: Severe hypoglycemia from clarithromycin-sulfonylurea drug interaction. Diabetes Care 2002 Sep;25(9):1659–1661.2. Lin G, et al: Refractory hypoglycemia from ciprofloxacin and glyburide interaction. J Toxicol Clin Toxicol 2004;42(3):295–297.3. Park-Wyllie LY, et al: Outpatient gatifloxacin therapy and dysglycemia in older adults. N Engl J Med 2006 Mar 30;354(13):1352–1361.4. Carr R, et al: Octreotide for sulfonylurea-induced hypoglycemia following overdose. Ann Pharmacother 2002 Nov;36(11):1727–1732.5. Cryer PE, et al: Hypoglycemia in diabetes. Diabetes Care 2003 Jun;26(6):1902–1912.6. Nelson P, et al: The incretin mimetic exenatide as a monotherapy in patients with type 2 diabetes.

Diabetes Technol Ther 2007 Aug;9(4):317–326.7. Cohen V, et al: Acute exenatide (Byetta®) poisoning was not associated with significant hypoglycemia.

Clin Toxicol (Phila) 2007 Aug 15:1–2.

Item 42 Answer B

This patient has inadequately controlled blood glucose levels, A1C above goal (7.0%) and postprandial bloodglucose in the 200–300 mg/dL range. The patient is already taking three oral agents and basal insulin; the nextappropriate step would be to add pre-prandial rapid-acting insulin three times a day, in addition to basal insulin.A basal/bolus insulin combination (50% in bolus form, 50% in basal form) would likely help the patient achieve his goals in glycemic control. Adding a second dose of basal insulin (glargine) would be less helpful, and mayincrease the risk of hypoglycemia, since the fasting blood glucose levels are closer to goal. With the addition ofbolus insulin before meals, less glargine may be required each day as well.

There is no consensus on whether to continue or discontinue sulfonylurea therapy while taking insulin. In thissituation, someone with long-standing type 2 diabetes may have less β-cell function, offering little contribution in terms of postprandial glucose control. Switching to repaglinide would offer limited relative benefit. With theaddition of pre-meal insulin, a secretatogue in general would not then be appropriate. Finally, increasing piogli-tazone is unlikely to reduce A1C level to less than 7.0%. It also would contribute to lower extremity edema (currentedema possibly arising from pioglitazone and/or amlodipine). Still, continuing insulin sensitizers is appropriate in combination with intensive insulin therapy.

Bibliography1. Hirsch IB. Intensifying insulin therapy in patients with type 2 diabetes mellitus. Am J Med 2005 May;118 Suppl 5A:21S–26S.2. Raskin P. Why insulin sensitizers but not secretagogues should be retained when initiating insulin in type 2 diabetes.

Diabetes/Metabolism Research and Reviews 2008;24(1):3–13.3. Hirsch IB. Insulin analogues. N Engl J Med 2005 Jan 13;352(2):174–183.4. Devries JH, et al: Refining basal insulin therapy: what have we learned in the age of analogues?

Diabetes Metab Res Rev 2007 Sep;23(6):441–454.

Item 43 Answer E

Resistance training in combination with aerobic exercise will likely reduce blood glucose levels. As she has triedvarious aerobic exercises, the addition of strength training offers additional benefit. In one study of 251 adults aged39–70 years with type 2 diabetes, the absolute change in A1C among participants receiving combined aerobic andresistance training was -0.51% lower than aerobic training only participants, and -0.38% lower than resistancetraining only participants. The pursuit of swimming or water aerobics may be preferable given her level of obesityand knee pain. Resistance exercises are safe and effective and may be complimentary to her current efforts. Someevidence suggests that home-based resistance training is less desirable than community-based programs due toadherence, equipment, and social support-related factors necessary for long-term improvement in glycemic control.Repetitive stretches or weight lifting should be done targeting multiple muscle groups in a graduated fashion,preferably after receiving professional instruction. Since she has met regularly with a dietitian and experiencedsuccess in weight loss, she may also succeed with exercise instruction and support.

Page 75: ADASAP2 Reduced PDF

73

The addition of medications may help her achieve her glycemic goal of therapy, but is likely less preferable thanexercise. A combined medication/exercise intervention may be offered, though this was not a choice given for this item. Pioglitazone may result in weight gain, though this would reduce blood glucose levels without risk of hypoglycemia. Exenatide may result in mild weight loss; however, she indicated her interest in avoidance of injections of insulin, so this may not be the best option at this point. A sulfonylurea may result in weight gain —approximately 2 kg is common with initiation of therapy. It also introduces the risk of hypoglycemia which mayresult with further efforts at lifestyle change. Finally, increasing metformin from 2000 mg to 2500 mg (maximum)will have limited benefit, and alone would not be superior to instituting resistance exercise.

Bibliography1. Sigal RJ, et al: Effects of aerobic training, resistance training, or both on glycemic control in type 2 diabetes: a randomized trial.

Ann Intern Med 2007 Sep 18;147(6):357–369.2. Dunstan DW, et al: Home-based resistance training is not sufficient to maintain improved glycemic control following supervised training

in older individuals with type 2 diabetes. Diabetes Care 2005 Jan;28(1):3–9.3. Dunstan DW, et al: Community center-based resistance training for the maintenance of glycemic control in adults with type 2 diabetes.

Diabetes Care 2006 Dec;29(12):2586–2591.4. Dunstan DW, et al: High-intensity resistance training improves glycemic control in older patients with type 2 diabetes.

Diabetes Care 2002 Oct;25(10):1729–1736.

Item 44 Answer A

There is increasing evidence that supports the use of intensive glucose control in hospitalized patients, especiallythose in critical care units. However, sliding scale of insulin protocols continues to be prevalent in use despite littleevidence to support its use. Physicians continue to utilize sliding scale of insulin protocols as it is convenient to implement, simple to apply, and provides quick treatment in response to elevated glucose levels. It is widelyaccepted by nursing staff and covering physicians so that additional contact and communication is unnecessary tomanage hyperglycemia. Some physicians may fear hypoglycemia and turn to sliding scale of insulin monotherapyor inadequately scheduled insulin treatment.

The sliding scale of insulin, usually with short-acting but not long-acting insulin, has never been associated withimproved clinical outcomes. A MEDLINE review found 52 publications between 1966–2003, none of whichshowing benefit via improved glycemic control or positive clinical outcomes. In general, it is felt that this approachis more likely dangerous to employ. The sliding scale is a response to previous hyperglycemia experienced and doesnot take into account trends in glucose levels. This approach has been described as “reactive” and leads to morerapid changes in glucose levels, exacerbating hypoglycemia and hyperglycemia. Instead, a standard insulin order setthat includes basal replacement insulin therapy and scheduled prandial, short-, or rapid-acting insulin therapy ismore appropriate during hospitalizations.

Bibliography1. Queale WS, et al: Glycemic control and sliding scale insulin use in medical inpatients with diabetes mellitus.

Arch Intern Med 1997 Mar 10;157(5):545–552.2. Umpierrez GE, et al: Sliding scale insulin use: myth or insanity? Am J Med 2007 Jul;120(7):563–567.3. Umpierrez GE, et al: Randomized study of basal-bolus insulin therapy in the inpatient management of patients with type 2 diabetes

(RABBIT 2 trial). Diabetes Care 2007 Sep;30(9):2181–2186.4. Clement S, et al: Management of diabetes and hyperglycemia in hospitals. Diabetes Care 2004 Feb;27(2):553–591.

Educational (Learning) Critiques American Diabetes Association Self-Assessment Program

Page 76: ADASAP2 Reduced PDF

American Diabetes Association Self-Assessment Program Educational (Learning) Critiques74

Items 45–47 Answers 45 (B); 46 (C); 47 (A)

The extended-release glipizide recordings had the highest average blood glucose levels (with only oral medicationtherapy without insulin) during the washout period (11.3 mean µmol/L, Option [A]). Both NPH (Option [B]) andglargine (Option [C]) combination with glipizide participants had significant improvement with lower glucoselevels throughout the day. Compared with the NPH combination group, the pre-supper and bedtime group haddecreased glucose levels in the glargine combination group. Also, the 3 a.m. glucose levels decreased less in thebasal (glargine combination) group. Overall, there were fewer fluctuations in glucose levels with glargine atbedtime. Of note, there were no episodes of severe hypoglycemia in either insulin treatment group.

In general, glycemic control includes three measures to address: fasting plasma glucose, postprandial glucose, andA1C. However, there is interest in exploring ‘glucose variability’ as this may also be an important consideration.In one study, glucose fluctuations (based on mean amplitude glucose alterations) were found to be activators ofoxidative stress in people with type 2 diabetes (including for those only on oral medication therapy). Continuousglucose monitoring may provide a means to assess this further for adults with type 2 diabetes, though its clinicalutility in this population remains to be determined. In addition, continuous blood glucose data may help in patienteducation and feedback, and possibly help motivate patients to transition from oral medication to insulin soonerwhen glycemic goals have not been achieved.

Bibliography1. Muggeo M, et al: Fasting plasma glucose variability predicts 10-year survival of type 2 diabetic patients: the Verona Diabetes Study.

Diabetes Care 2000 Jan;23(1):45–50.2. Wang XL, et al: Evaluation of the superiority of insulin glargine as basal insulin replacement by continuous glucose monitoring system.

Diabetes Res Clin Pract 2007 Apr;76(1):30–36.3. Monnier L, et al: Continuous glucose monitoring in patients with type 2 diabetes: Why? When? Whom?

Diabetes Metab 2007 Sep;33(4):247–252.4. Monnier L, et al: Activation of oxidative stress by acute glucose fluctuations compared with sustained chronic hyperglycemia in patients

with type 2 diabetes. JAMA 2006 Apr 12;295(14):1681–1687.

Item 48 Answer A

Though one might encourage this patient to follow a better diet and exercise, often this advice is not followed and it delays care. Though it used to be appropriate to wait 3 months between interventions, ‘therapeutic inertia’,(the “watch ‘em” disease of physicians), it is now preferable to make changes every 2 to 4 weeks after increasing a dose or adding a new drug. One may choose to wait for a longer period of time, but in most situations external life stresses do not dissipate quickly. This patient needs DSME and also help with life stresses.

Threatening the start of insulin therapy in order to achieve better compliance with diet and exercise rarely works,and adds to the patient’s future fear of insulin therapy, so this is strongly discouraged.

Exercise increase alone will rarely lower fasting blood glucose levels from the �180 mg/dL range. It is importantwhen a patient is on a sulfonylurea to make sure unrecognized hypoglycemia does not develop overnight thusresulting in a rebound elevated fasting blood glucose level. Clinically, if there is certainty that this is not the case,treatment with insulin should be initiated.

Bibliography1. Cefalu WT. Pharmacotherapy for the Treatment of Patients with Type 2 Diabetes Mellitus: Rationale and Specific Agents.

Clin Pharmacol Ther 2007 May;81(5).2. Shah BR, et al: Clinical inertia in response to inadequate glycemic control: do specialists differ from primary care physicians?

Diabetes Care 2005;28:600.3. Tibaldi J, et al: Why, when and how to initiate insulin therapy in patients with type 2 diabetes.

International J Clinical Practice 2007 April;61:633–644.

Page 77: ADASAP2 Reduced PDF

75

Items 49–53 Answers 49 (C); 50 (A); 51 (E); 52 (B); 53 (D)

When a patient with type 2 diabetes fails 3 or 4 non-insulin modalities, insulin therapy is recommended. Whenbasal-bolus in insulin is being used to treat both fasting and postprandial elevated glucose, there are logical goalreasons to maintain several of the non-insulin modality treatments as one uses basal-bolus insulin therapy.

Metformin has been shown to reduce AGE production. It may also result in lower doses of basal insulin requiredfor control.

Thiazolidinediones may preserve remaining β-cell production as evidenced by the DREAM trial showing a slowerneed to start a second non-insulin therapy in patients with type 2 diabetes. Pioglitazone may have a possible (as yetunproven) cardiovascular benefit as suggested by the proactive trial, the Chicago trial, and the Periscope trial.However, rosiglitazone has a black box warning given by the FDA for possibly increasing adverse cardiac outcomesespecially in patients with ischemic CV disease, on nitrates, or insulin. The Canadian as well as European drugagencies also have released warnings for possible increased cardiovascular risk. Rosiglitazone also has an adverselipid pattern of increased triglycerides, increased non-HDL-C, and increased LDL particle number. Thus, there isno logic to maintain rosiglitazone therapy, but it is quite reasonable to switch these patients to pioglitazone andmaintain it when starting insulin therapy so as to get lipid benefits, likely preservation of β-cells, and reduction ofinsulin doses required (by increasing insulin sensitivity).

Exenatide, an injectable incretin, has evidence in animals for preserving remaining β-cells and possibly increasingproduction of new β-cells. It decreases required doses of insulin and provides smoother control of postprandialglucose and has a side benefit of weight loss. However, this would be off-label use of exenatide, as it is not approvedin combination with insulin.

Sitagliptin, a DPP-4 inhibitor, increases endogenous incretin levels and will have the same benefits as exenatide,except for the weight loss. But the use of sitagliptin would be off-label, as it is not approved in combination with insulin.

Acarbose does result in reduced postprandial glucose and thus smoother control by delaying glucose absorption byshifting absorption to distal small intestinal sites; however, tolerance and patient compliance of this drug is difficultgiven the diarrhea that often accompanies its use.

Sulfonylureas can control postprandial glucose, but this can be accomplished quite well with bolus insulin therapyalone. Discontinuing sulfonylureas avoids an undue risk of postprandial hypoglycemia and further destruction of any residual β-cells.

Glinides can control postprandial glucose, but this can be accomplished quite well with bolus insulin therapyalone. Discontinuation avoids an undue risk of postprandial hypoglycemia.

Bibliography1. Riddle MC. The treat-to-target trial and related studies. Endocr Pract 2006;12 suppl 1:71.2. Bergenstal RM. Treatment models from the international diabetes center: advancing from oral agents to insulin therapy in type 2

diabetes. Endocr Practice 2006;12 suppl 1:98.3. DECODE Study Group. Glucose tolerance and cardiovascular mortality: comparison of fasting and 2-hour diagnostic criteria.

Arch Intern Med 2001;161:397–405.4. Goldberg RB, et al: For the GLAI Study Investigators. A comparison of lipid and glycemic effects of pioglitazone and rosiglitazone

in patients with type 2 diabetes and dyslipidemia. Diabetes Care 2005;28:1547–54.5. Deeg MA, et al: For the GLAI Study Investigators. Pioglitazone and rosiglitazone have different effects on serum lipoprotein particle

concentrations and sizes in patients with type 2 diabetes and dyslipidemia. Diabetes Care 2007;30:2458–64.6. Dormandy JA, et al: Secondary prevention of macrovascular events in patients with type 2 diabetes in the PROactive Study

(PROspective pioglitAzone Clinical Trial In macro Vascular Events): a randomized, controlled trial. Lancet 2005;366:1279–89.7. Wilcox R, et al: for the PROactive Investigators. Effects of pioglitazone in patients with type 2 diabetes with or without previous stroke:

results from PROactive (PROspective pioglitAzone Clinical Trial In macroVascular Events 04). Stroke 2007;38:865–73.8. Erdmann E, et al: for the PROactive Investigators. The effect of pioglitazone on recurrent myocardial infarction in 2445 patients

with type 2 diabetes and previous myocardial infarction: results from the PROactive (PROactive 05) study.J Am Coll Cardiol 2007;49:1772–80.

9. Mazzone T, et al: Effect of pioglitazone compared with glimepiride on carotid intima–media thickness in type 2 diabetes: a randomized trial. JAMA 2006;296:2572–81.

Educational (Learning) Critiques American Diabetes Association Self-Assessment Program

Page 78: ADASAP2 Reduced PDF

American Diabetes Association Self-Assessment Program Educational (Learning) Critiques76

10. Nissen SE. Comparison of Pioglitazone vs Glimepiride on Progression of Coronary Atherosclerosis in Patients With Type 2 Diabetes The PERISCOPE Randomized Controlled Trial. JAMA 2008;299(13):1561–1573.

11. Riddle MC. Glycemic Management of Type 2 Diabetes: An Emerging Strategy with Oral Agents, Insulins, and Combinations,Endocrinol Metab Clin N Am 2005;34:77–98.

Item 54 Answer C

A normal subject secretes a small amount of insulin almost continuously for 24 hours. This is to prevent ketoge-nesis when fasting. In the face of the need to store exogenous food, glucose, and calories, the pancreas secretes, veryprecisely, extra insulin at the time of any p.o. intake. One can sometimes discern an increase in the small continuousamount of insulin roughly between 3 a.m. and 8 a.m. due to increased renal clearance of insulin during that timeand increased insulin requirement associated with the normal cortisol diurnal variation, i.e., increased cortisol seenearly in the morning.

“Traditional” insulins used in the past in regard to basal insulin therapy had significant peaks, certainly did not last24 hours and thus do not look like the normal basal insulin secretion pattern in the normal subject.

The traditional regular human insulin takes 1 to 2 hours to peak and can last at least 5 and 6 hours and sometimeslonger. The fast analog insulins peak within 1 hour and last 3 to 4 hours mimicking much more closely the normalβ-cell response to mealtime intake. An insulin pump with the ability to continuously put out sparing amounts of“basal” insulin given as a continuous 2 infusion of a fast analog insulin combined with adjusting boluses of the fastanalog insulin each time one eats, can mimic the normal human pattern of insulin secretion more closely than anyother standard regimen that is available at the present time. Thus, ideal insulin therapy will attempt to mimic what a normal pancreas does, will be best achieved by basal insulinization that has minimal peaks even if given once ortwice a day, will mimic the fast release and relatively short duration of bolus insulin with a meal that can be donebest with an insulin pump and bolus fat-insulin analogs, and, most importantly, the combination of basal and bolusinsulin can be adjusted to match the pharmacokinetics of the insulin one chooses to the lifestyle and nutritionalsource or eating pattern of the patient.

In this regard, patterns of administration can mimic and deal with the normal increased insulin requirementbetween 3 and 8 a.m. (called the dawn effect) and can match mealtime patterns where people delay or miss meals,travel internationally, do shift work, wake up at different times weekends and weekdays. However, because of theslower peak and longer duration of regular human insulin in comparison to fast analog insulin, ideal insulin therapywill in the vast majority of situations not include regular human insulin.

Bibliography1. Dewitt DE, et al: Out-patient Insulin Therapy in Type 1 and Type 2 Diabetes Mellitus. JAMA 2003;289:2254.2. Gough S. A review of human and analogue insulin trials. Diabetes Research and Clinical Practice 2007;77:1–15.3. Mooradian AD, et al: Narrative Review: A Rational Approach to Starting Insulin Therapy.

Ann Intern Med 2006 July 18;145(2):125–134.4. Bode. Incorporating Postprandial and Fasting Plasma Glucose into Clinical management Strategies Insulin 2008;3:17.5. Raccah D. When basal insulin therapy in Type 2 Diabetes is not enough-what next? Diabetes Metabolism Res Rev 2007;23:257.6. Rosenstock J, et al: A randomized 52 Week, Treat-To-Target Trial Comparing Insulin Detemir With Insulin Glargine When

Administered As Add-On To Glucose Lowering Drugs In Insulin-Naive People With Type 2 Diabetes, Diabetologica 2008;51:408.7. Devries JH, et al: Refining basal insulin therapy: what have we learned in the age of analogues? Diabetes Metab Res Rev 2007;23:441.8. LeRoith D. Our evolving understanding of getting to goal using insulin in Type 2 Diabetes. Endo Metab Clinics 2007 Dec;suppl 2:9.9. Tibaldi J, et al: Why, when and how to initiate insulin therapy in patients with type 2 diabetes.

International J Clinical Practice 2007 April;61:633.10. Hirsch IB, et al: A real-world approach to insulin therapy in primary care practice. Clin Diabetes 2005;23:78–86.11. Davis T, et al: Insulin therapy in type 2 diabetes. Med Clin N Am 2004;88:865–895.

Item 55 Answer A

Regular human insulin starts working in 30 minutes to 1 hour, peaks in 2 hours and can last up to 6 and even morehours. Since patients with normal gastric emptying will absorb calories faster than the peak effect of the regularhuman insulin, these patients will be subject to hyperglycemia in the first 2 hours and, are subject to late (4–6 hours) postprandial hypoglycemia.

Page 79: ADASAP2 Reduced PDF

77

Since patients with type 2 diabetes given fast analog insulin are responsive within 30 minutes, peak in one hour, andlast 3 or a maximum 4 hours, thus more closely mimicking the normal gastric emptying and appearance of glucosein the blood stream, there is a better match of insulin to absorb the calories resulting in less 2-hour hyperglycemiaand less 4 hour hypoglycemia compared to regular human insulin. As a result, area under the curve postprandialglycemic patterns are significantly reduced with the use of fast analogs rather than with the use of regular insulin.

Moreover, patients with regular insulin are often told to take their insulin at least 30 minutes prior to the meal to tryto reduce the postprandial hyperglycemia. However, if they do not know how much they are going to eat and musttherefore delay insulin until after eating, they further accentuate postprandial hyperglycemia and late postprandialhypoglycemia. However, patients using fast analogs, because it works so quickly can actually be given it immedi-ately after eating when they see and can calculate how much carbohydrate they ate, and thus can calculate the right amount of fast analog insulin to match what they ate, again getting the best control of their postprandialglycemic patterns.

Fast analog insulin does not address the issue of gastroparesis better than regular insulin. Patients who have gastro-paresis due to diabetic autonomic neuropathy will have significantly delayed gastric emptying, such that fast analoginsulin in these patients is likely to cause immediate postprandial hypoglycemia as well as significant absorption ofcalories because of delayed emptying of contents into the small intestine. The best treatment of significant gastro-paresis in these patients is with an insulin pump where boluses can actually be given as a square wave over severalhours after eating.

Bibliography1. Leahy JL. In: Leahy JL, Cefalu WT, eds. Insulin Therapy New York, NY:Marcel Dekker, Inc;2002:87–112.2. Bolli GB, et al: Diabetologia 1999;42:1151–1167.

Item 56 Answer D

Differences in the pharmacokinetics and dynamics of regular insulin vs fast analog insulin are often highlighted by problems and situations often seen in hospitalized patients.

Thus, a patient may be given regular subcutaneous insulin at breakfast, be taken for a laboratory study, and have a significant delay in lunch and, as such, because of the prolonged action of regular insulin, there is an increasedrisk of hypoglycemia. Because fast analog insulin effect has dissipated 3 to 4 hours after injection, a significant delay in lunch has a markedly reduced risk of hypoglycemia vs patients treated with regular insulin in the samecircumstances.

Another situation is that patients who have hyperglycemia are often given “correction factor doses” of subcuta-neous short-acting insulin (the old sliding scale).Typically house staff will order a dose of regular insulin, nursingstaff will be uncomfortable as they see no significant reduction 3 and 4 hours later. House staff will be asked andwill administer another dose of regular insulin and similarly a third event will occur 3 or 4 hours after the seconddose. Repeated administration of an insulin that lasts for 6 or longer hours can result in “stacking” effects of therecurrent regular insulin dosing” and markedly increases the risk of hypoglycemia by the third or more dose ofregular insulin. Use of the fast analog where usual effect is gone after 3 to 4 hours, minimizes the risk for hyper-glycemia due to stacking.

In the hospital, patients frequently have inconsistent p.o. intake. This may be the elderly with altered mental statusand altered and varying appetite for other reasons e.g.: this may be somebody getting chemotherapy, etc. Again,regular insulin is usually given at least 30 minutes before meal and if usual doses are given and the patient eats less,they are highly likely to become hypoglycemic after eating. Fast analog insulins in patients who have inconsistentp.o. intake in a predictable fashion can have their preprandial insulin held, and after it is observed how manycalories or grams of carbohydrate they actually do eat, the nurse/physician can adjust the dose of the fast analog to compensate for the reduced intake.

However, regular and fast analog insulin when given IV have equal effects and thus, there is no advantage to one or the other in terms of hypoglycemia, which is the answer D.

Educational (Learning) Critiques American Diabetes Association Self-Assessment Program

Page 80: ADASAP2 Reduced PDF

American Diabetes Association Self-Assessment Program Educational (Learning) Critiques78

Bibliography1. Leahy JL. Insulin management of diabetic patients on general medical and surgical floors. Endocr Pract 2006;12 suppl 1:86.2. Thompson CL, et al: Hyperglycemia in the Hospital. Diabetes Spectrum 2005;18:20.3. Tomky D. Detection, prevention, and treatment of hypoglycemia in the hospital. Diabetes Spectrum 2005;8:39.4. Braithwaite S. Hospital Hypoglycemia: not only treatment but prevention. Endocr Pract 2004;10 suppl 2:89.

Item 57 Answer C

The most important diagnosis that one must assume exists before one considers or even treats other possibilities to explain the increased intermittent fasting glucose in the presence of (new) basal insulin therapy is the possibility of middle of the night hypoglycemia with an a.m. rebound. When the glucose level goes too low overnight, mostpeople do not wake up with it, but the brain tells the liver to make extra glucose and the fasting levels are elevated.Many patients and physicians are not aware of this and will keep on increasing the basal insulin dose and this canresult in increased weight gain because one has to eat more to avoid hypoglycemia during the day. Patients mightnotice waking up in the morning with headache, having occasional nightmares, having bed sheets soaked withsweat, or waking up with a profound malaise, a ‘hangover’ sensation without drinking alcohol. If it proceeds too far, seizures can occur in the middle of the night.

About 80% of patients referred for insulin therapy actually have required reductions in their current insulin doses.

Thus, the pattern of increasing insulin with increased weight and unusual morning symptoms should make aphysician consider middle of the night hyperglycemia with a rebound as the most plausible explanation. Certainly,the patients deserve increased education to double check their injection technique, see the nutritionist to reinforcecompliance with diet, and learn more about high glycemic index foods. The likelihood of the “dawn effect” as anexplanation is quite small, (that is, the pattern where middle of the night numbers are good, but glucose is higherbecause of lack of ability of the patient’s own pancreas to produce extra insulin to prevent the increased insulinresistance that is apparent in some truly insulin-deficient patients first thing in the morning).

Bibliography1. Devries JH, et al: Refining basal insulin therapy: what have we learned in the age of analogues?

Diabetes Metab Res Rev 2007;23:441–454.2. Cryer PE. Insulin therapy and hypoglycemia in type 2 diabetes. Insulin 2007;2:127.

Item 58 Answer A

The most important therapeutic intervention for this patient is to reduce the patient’s glargine 10% to 20% fromthe last stable dose, which has been 26 U. So a dose of 22 to 23 U would be quite appropriate. It would certainly be appropriate to advise the patient to check 3 a.m. glucose levels whenever changing the glargine dose andcertainly even periodically once every week or two, even when it is apparently perfectly stable. The patient shouldbe instructed on the proper insulin injection and glucose monitoring techniques, and understanding of nutritionand high glycemic index foods.

Item 59 Answer B

To get better control of postprandial glucose levels, the best therapy for glycemic control and reducing the risk of hyperglycemia, less weight gain, better lifestyle, or adaptation to different lifestyles would be the start of bolus fastanalog insulin with each meal. However, the patient preferred not to take 4 injections a day and so a twice dailybefore breakfast and before dinner fixed mixture of an intermediate acting insulin with fast analog was started.

Premixed preparations are available as combinations of intermediate-acting with regular insulin, but given theadvantages of fast analogs discussed above, the preference is to use an intermediate-acting insulin with the fastanalog. It is certainly more convenient to use a premixed preparation than to self-mix NPH and the short-actinginsulin, and it is much more accurate than mixing NPH and short-acting insulin in the same syringe.

Premixed preparations do require the patient to eat the same amount of food at the same time each day to try to adapt to the peaks of the NPH insulin.

Page 81: ADASAP2 Reduced PDF

79

So, he was started and titrated, i.e., 6 mixture of NPH and a fast analog before breakfast and before dinner, and hewas advised to eat the same amount of food each day and to wake up at the same time each day to minimize hypoand hyperglycemia due to the peaks of the intermediate-action insulin component

Bibliography1. Raskin P. Can glycemic targets be achieved-in particular with two daily injections of a mix of intermediate and short-acting insulin?

Endocr Pract 2006;12 supp 1:52.

Item 60 Answer C

One of the most common reasons to use insulin pump therapy is that patients, typically type 1 patients or type 2patients who are very insulin deficient, i.e., type 1-1/2,“might with even the best, appropriate adjustment of basaldoses,” have reasonable glucose levels at 3 a.m., but without hypoglycemia intervening, may have elevated glucoselevels on awakening in the morning. This is called the dawn phenomenon. It is due to the increased insulin require-ments first thing in the morning, which is actually definable in normal subjects due to normal diurnal variation ofcortisol as well as increased renal excretion of insulin early in the morning.

Thus, the ability to adjust basal rates hour by hour, which is achievable with insulin pump therapy, but is impos-sible with MDI, can easily solve this phenomenon of the dawn phenomenon.

MDI can deal with regular exercise by reducing bolus amounts prior to the meals that are taken before exercise.However, insulin pumps allow basal rates to be altered as well as boluses, especially under the more difficult situa-tions of intermittent exercise, and thus make it easier than MDI to deal with intermittent exercise.

Insulin pump therapy makes it easier to deal with variations in carbohydrate absorption that occur with somecomplex carbohydrate, higher fat meals, higher fiber meals, and most especially with gastroparesis by takingadvantage of being able to give boluses as square waves or dual square waves.

Glycohemoglobin is likely to be better with insulin pump therapy than with MDI.

However, if there is a pump malfunction, the patient is not receiving any basal insulin and thus, onset of diabeticketoacidosis can quickly develop, especially if the patient is very insulin-deficient or absolutely insulin-deficient.This is markedly less likely to occur if the patient is taking the usual basal therapy and MDI regimens.

Bibliography1. Raskin, et al: Diabetes Care 2003;26(9):2598–2603.2. Testa, et al: Diabetes 2001;50 suppl 2:1781.3. Weissberg-Benchell J, et al: Insulin pump therapy: a meta-analysis. Diabetes Care 2003;26:1079–1087.

Items 61–66 Answers 61 (A); 62 (B); 63 (E); 64 (D); 65 (D); 66 (C)

Metformin reduces insulin resistance at the liver. Pioglitazone and rosiglitazone increase insulin sensitivity atmuscle and fat. Xenical works only by decreasing the rate of glucose absorption from the gut; however, we do know that exenatide does slow down gastric emptying and delays absorption of glucose from the gut with meals.

Both exenatide and sitagliptin reduce the paradoxical rise of glucagon seen in patients with type 2 diabetes.Both exenatide and sitagliptin in animal studies seem to increase insulin production with transplant medicines and steroids. These incretins increase insulin secretion of any remaining β-cells in a glucose-dependent mannerthat means when the glucose returns to normal, there is no further extra increase in this insulin and so the risk of hypoglycemia is quite minimal.

The secretagogues, sulfonylureas, and glinides increase some secretion of the remaining β-cells, but they do this ina glucose-independent manner, so patients have a low blood glucose, e.g., 40 mg/dL. They will continue to secreteinsulin as long as the drug is circulating in the blood stream.

Educational (Learning) Critiques American Diabetes Association Self-Assessment Program

Page 82: ADASAP2 Reduced PDF

American Diabetes Association Self-Assessment Program Educational (Learning) Critiques80

Bibliography1. Dunning BE, et al: The Role of beta-Cell Dysregulation in Fasting and Postprandial Hyperglycemia in Type 2 Diabetes and Therapeutic

Implications. Endocr Rev 28(3):253–283.2. Sheffield, et al: Safety and Efficacy of exenatide in combination with insulin in patients with Type 2 diabetes mellitus.

Endocr Pract 2008;14:285.3. Yoon. Exenatide added to insulin. ADA abstracts 2008 ADA meeting.4. Inzucchi. New drugs for the treatment of diabetes: Incretin based therapy and beyond. Ciculation 2008;117:574.

Item 67 Answer B

Lifestyle changes including weight loss and increased activity have both short- and long-term benefits, if they can bemaintained. In recognition that many times lifestyle changes cannot be maintained, metformin is recommended asthe initial pharmacologic therapy (in the absence of contraindications) given its efficacy (usual absolute decrease inA1C of 1.5%), absence of weight gain or hypoglycemia and relatively low cost compared to the other agents listed.

Bibliography1. Nathan DM, et al: Management of Hyperglycemia in Type 2 Diabetes: A Consensus Algorithm for the Initiation and Adjustment

of Therapy: A consensus statement from the American Diabetes Association and the European Association for the Study of Diabetes.Diabetes Care 2006;29:1963–1972.

Item 68 Answer C

Substitution of intermediate-acting insulins with long-acting insulins actually decreases the risk of hypoglycemia.A missed or delayed meal can cause an excess of insulin relative to plasma glucose, causing hypoglycemia.Elderly patients are at greater risk of hypoglycemia. Factors that may predispose them to hypoglycemia includepolypharmacy, erratic nutrition, impaired renal or hepatic function, and medication noncompliance. Renal insuffi-ciency predisposes to hypoglycemia because of reduced renal clearance of insulin as well as medications such assulfonylureas. Alcohol consumption is known to have a delayed hypoglycemic effect.

Bibliography 1. Briscoe VJ, et al: Hypoglycemia in Type 1 and Type 2 Diabetes: Physiology, Pathophysiology, and Management.

Clin Diabetes 2006;24:115–121.

Item 69 Answer E

The original Diabetes Control and Complications Trial (DCCT) was conducted in patients with type 1 diabetesbetween 1983 and 1993. With a mean follow-up of 6.5 years, 1422 patients completed the study with one halfassigned to intensive therapy and the remainder to conventional therapy. The average A1C level was 7.4% in theintensive group versus 9.1% in the conventional group.

Development of new retinopathy was decreased by 76% in the intensive treatment group compared to the conven-tional group and the progression of established retinopathy was deceased by 54% in the intensive group comparedto the conventional group. Intensive treatment also decreased the occurrence of microalbuminuria by 39% andalbuminuria by 54% as well as occurrence of clinical neuropathy by 60%. However, there was no significant changein macrovascular events between the intensive and conventional groups at the end of the study although there was a trend towards improvement.

The Epidemiology of Diabetes Interventions and Complications (EDIC) was an observational study that followedthe DCCT cohort of patients, all of whom were encouraged to transition to intensive therapy at the end of theDCCT. After 11 years, glycemic control was comparable between the two groups; the mean A1C was 7.9% in thegroup that had been the intensive group during the DCCT and 7.8% in the group that had been the conventionalgroup. Micro- and macroalbuminuria continued with increasing frequency in the previous conventional groupversus the previous intensive group. At the end of 17 years of DCCT and EDIC, the risk of nonfatal MI, stroke,and death from cardiovascular disease was 57% lower in the previous intensive group compared to the previousconventional group. Intensive diabetes management for 6.5 years early in the course of disease was able to decreasethe risk of cardiovascular events several years later and reinforces the belief that patients with type 1 diabetes benefitfrom the implementation of intensive insulin therapy.

Page 83: ADASAP2 Reduced PDF

81

Bibliography1. The effect of intensive treatment of diabetes on the development and progression of long-term complications in insulin-dependent

diabetes mellitus. The Diabetes Control and Complications Trial Research Group. N Engl J Med 1993;329(14):977–86.2. Nathan DM, et al: Intensive diabetes treatment and cardiovascular disease in patients with type 1 diabetes.

N Engl J Med 2005;353(25):2643–53.3. ACP Journal Club. 2006 May–Jun;144:63.

Item 70 Answer B

There can be significant overlap between diabetic ketoacidosis (DKA) and hyperglycemic state (HHS) clinically,but generally, patients with HHS are more profoundly dehydrated, have a significantly higher plasma osmolality,a significantly higher glucose level, and are not ketonemic. The treatment of both conditions is similar and includesaggressive hydration, intravenous insulin, and potassium and phosphate repletion with care not to correct thehyperosmolar state too quickly. In experienced centers, the mortality rate in patients with DKA has decreased overtime and is currently less than 5%. The mortality rate for HHS remains at 11%. In the majority of cases mortality is not due to these metabolic complications, but to the underlying precipitating cause.

Bibliography1. Kitabchi AE, et al: Hyperglycemic Crises in Adult Patients With Diabetes: A consensus statement from the American Diabetes

Association. Diabetes Care 2006;29:2739–2748.

� Items 71–74

Item 71 Answer D

Data from the Diabetes Prevention Program showed that intensive lifestyle modification with 7% reduction in initial body weight with healthy diet and physical activity of moderate intensity (i.e. brisk walking) for at least 150 minutes/week improved hyperglycemia and cardiovascular risk factors. A recent meta-analysis of studiesinvolving exercise and glycemic control in diabetics showed that A1C improved on average by 0.66% with regularexercise without any change in weight. The large majority of patients will require additional medications over thecourse of their diabetes despite adequate lifestyle changes.

Bibliography1. The Diabetes Prevention Program Research Group. Impact of Intensive Lifestyle and Metformin Therapy on Cardiovascular Disease

Risk Factors in the Diabetes Prevention Program. Diabetes Care 2005;28:888–894.2. Boule NG, et al: Effects of exercise on glycemic control and body mass in type 2 diabetes mellitus: a meta-analysis of controlled clinical

trials. JAMA 2001;286(10):1218–1227.3. Nathan DM, et al: Management of Hyperglycemia in Type 2 Diabetes: A Consensus Algorithm for the Initiation and Adjustment of

Therapy: A consensus statement from the American Diabetes Association and the European Association for the Study of Diabetes.Diabetes Care 2006;29:1963–1972.

Item 72 Answer C

Metformin should not be used given this patient’s renal insufficiency. Exenatide would be a good option but wouldimprove postprandial hyperglycemia more significantly than fasting glucose levels. Either the addition of basalinsulin or a thiazolidinedione would be appropriate after consideration and discussion of possible side effects.

Bibliography1. Nathan DM, et al: Management of Hyperglycemia in Type 2 Diabetes: A Consensus Algorithm for the Initiation and Adjustment

of Therapy: A consensus statement from the American Diabetes Association and the European Association for the Study of Diabetes.Diabetes Care 2006;29:1963–1972.

Educational (Learning) Critiques American Diabetes Association Self-Assessment Program

Page 84: ADASAP2 Reduced PDF

American Diabetes Association Self-Assessment Program Educational (Learning) Critiques82

Item 73 Answer E

Factors that increase the risk of hypoglycemia with sulfonylureas include longer-acting formulations, renal or hepatic disease, missed meals or poor nutrition, and concomitant therapy with salicylates, and fibric acid derivatives. The risk of hypoglycemia is not increased in overweight patients.

1. Riddle MC. Combined Therapy With Insulin Plus Oral Agents: Is There Any Advantage?: An argument in favor.Diabetes Care 2008;31:S125–130S.

2. Bressler P, et al: Drugs and diabetes. Diabetes Reviews 1994;2:53.

Item 74 Answer B

This patient has microalbuminuria, defined as urinary albumin to creatinine ratio between 30–300 mg/g. His bloodpressure is not at the current goal for all patients with type 2 diabetes of less than 130/80 mm Hg. The ADA guide-lines for treatment of hypertension in diabetes recommend either an ACE-I or ARB as the first line agent withsubsequent addition of a thiazide diuretic if needed, with the recognition that most patients will require multipleagents to achieve control.

Both ACE-Is and ARBs have been shown in multiple studies to slow development and progression of diabeticnephropathy, but one agent has not been clearly shown to be more effective than the other. Although there havebeen some preliminary trials that the combination of ACE-I and ARB may be effective, this combination is notroutinely recommended. A recent study showed that over a five-year follow-up period, intensive blood pressurecontrol (approximately 128/75 mm Hg) in patients with type 2 diabetes slowed the progression to micro- andmacroalbuminuria, decreased the progression of diabetic retinopathy, and also decreased the incidence of stroke.

Bibliography1. Hypertension Management in Adults With Diabetes. Diabetes Care 27:65S–67S.2. Executive Summary: Standards of Medical Care in Diabetes—2008. Diabetes Care 31:S5–11S.3. Schrier RW, et al: Effects of aggressive blood pressure control in normotensive type 2 diabetic patients on albuminuria, r

etinopathy and strokes. Kidney Int 2002 Mar;61(3):1086–97.

Items 75–76 Answers 75 (B); 76 (D)

Weight loss improves hepatic steatosis. Nonalcoholic fatty liver disease (NAFLD) encompasses the entire spectrumfrom steatosis alone to steatohepatitis (nonalcoholic steatohepatitis or NASH) and cirrhosis. While once thought to be a benign process, NASH in patients with diabetes has been shown to have the propensity to progress to more severe inflammation and to cirrhosis. The reason why some patients progress and others do not is notcurrently understood.

By some estimates, diabetes is now the most common cause of liver disease in the U.S. In asymptomatic patients,the most common causes of elevations in liver transaminases are nonalcoholic fatty liver disease (NAFLD) andchronic hepatitis. The prevalence of NAFLD is estimated to approach almost 100% in obese patients withdiabetes. There is an increased prevalence of hepatitis C in patients with diabetes compared to the generalpopulation. Patients with hepatitis C are at higher risk of developing diabetes compared to patients with hepatitis B. Statins appear to be safe and may even be hepatoprotective in patients with diabetes with liver disease.NAFLD can be diagnosed by ultrasound. The use of liver biopsy to diagnose nonalcoholic steatohepatitis iscurrently controversial given that there are currently no clear pharmacologic treatments that alter the natural history of the disease.

Bibliography1. Tolman KG, et al: Spectrum of Liver Disease in Type 2 Diabetes and Management of Patients With Diabetes and Liver Disease.

Diabetes Care 2007;30:734–743.2. Chalasani N, et al: Patients with elevated liver enzymes are not at higher risk for statin hepatotoxicity.

Gastroenterology 2004;126:1287–1292.

Page 85: ADASAP2 Reduced PDF

83

Item 77 Answer C

Hypertension, advanced age, co-administration with insulin and coronary artery disease are all risk factors for thedevelopment of heart failure in patients with type 2 diabetes treated with thiazolidinediones (TZDs). Other riskfactors include history of heart failure or prior MI, left ventricular hypertrophy, long-standing diabetes (�10 years),pre-existing edema or current treatment with loop diuretics. Additionally, the development of edema or weight gainon TZD therapy is a risk factor. Lastly, chronic renal failure (creatinine �2.0 mg/dL) appears to be associated withhigher risk of development of heart failure in patients treated with TZDs.

Bibliography1 Nesto RW, et al: Thiazolidinedione Use, Fluid Retention, and Congestive Heart Failure: A consensus statement from the American

Heart Association and American Diabetes Association. Diabetes Care 2004;27:256–263.2. Elasy TA, et al: Thiazolidinedione Use, Fluid Retention, and Congestive Heart Failure: a consensus statement from the American Heart

Association and American Diabetes Association: Response to Nesto. Diabetes Care 2004;27:2096.

� Items 78–81

Item 78 Answer A

Several studies have established the independent association between diabetes and the incidence of heart failure,including the Cardiovascular Health Study and ALLHAT. Diabetes has also been identified as a risk factor for theprogression and worse outcome in patients with heart failure. Not only is diabetes a risk factor for the developmentof heart failure, but heart failure has been identified as a risk factor for the development of insulin resistance anddiabetes.

Item 79 Answer C

Any patient with type 2 diabetes on a thiazolidinedione (TZD) and admitted with heart failure should have theirTZD stopped. Metformin has recently been recognized as safe for patients with stable heart failure but should bestopped during an acute episode. Many argue that metformin should be discontinued in all hospitalized patientsgiven the high risk of acute renal insufficiency in this population. Sulfonylureas, especially long-acting formulations,have an increased risk of hypoglycemia in hospitalized patients who may be eating erratically. Insulin is recognizedas the safest, most effective and most quick-acting treatment for hyperglycemia in hospitalized patients.

Item 80 Answer C

Most of the oral hypoglycemic agents are safe for patients with type 2 diabetes with heart failure, except formetformin and thiazolidinediones (TZDs) which have relative contraindications in severe heart failure. Metforminseems to be safe in patients with mild heart failure, who are clinically stable, and is not associated with weight gain.In a patient such as this with mild heart failure, a TZD should be prescribed with caution and close clinical followup. Additionally, regimens including insulin or TZD are likely to cause weight gain. Sulfonylureas also generallycause mild weight gain, although less than TZDs or insulin. Sitagliptin appears to be weight neutral. Exenatide is the only medication listed that may cause weight loss. As a caveat, neither sitagliptin nor exenatide has beenstudied specifically in heart failure patients.

Item 81 Answer A

Although there is an independent association between type 2 diabetes and heart failure, there are currently noprospective studies showing what range of A1C is beneficial in these patients. At this time, patients with mild heartfailure should be treated according to ADA guidelines to goal of less than 7%. There is some concern in patientswith severe heart failure that lower A1C may be associated with higher mortality but this may be for several reasonsbeyond glycemic control and further studies are needed to make any recommendations in this population withsevere heart failure.

Educational (Learning) Critiques American Diabetes Association Self-Assessment Program

Page 86: ADASAP2 Reduced PDF

American Diabetes Association Self-Assessment Program Educational (Learning) Critiques84

Bibliography1. Masoudi FA, et al: Diabetes mellitus and heart failure: epidemiology, mechanisms, and pharmacotherapy.

Am J of Cardiology 2007 Feb 99(4A):113B–132B.2. Gottdiener JS, et al: Predictors of congestive heart failure in the elderly: the Cardiovascular Health Study.

J Am Coll Cardiol 2000;35:1628–1637.3. Davis BR, et al: Role of diuretics in the prevention of heart failure: the Antihypertensive and Lipid-Lowering Treatment to Prevent Heart

Attack Trial. Circulation 2006;113:2201–2210.4. Das SR, et al: Effects of diabetes mellitus and ischemic heart disease on the progression from asymptomatic left ventricular dysfunction

to symptomatic heart failure: a retrospective analysis from the Studies of Left Ventricular Dysfunction (SOLVD) prevention trial.Am Heart J 2004;148:883–888.

5. Tenenbaum A, et al: Functional class in patients with heart failure is associated with the development of diabetes.Am J Med 2003;114:271–275.

6. Eshaghian S, et al: An unexpected inverse relationship between HbA1C levels and mortality in patients with diabetes and advancedsystolic heart failure [clinical investigation]. Am Heart J 2006;151:91.

7. Nathan DM, et al: Management of Hyperglycemia in Type 2 Diabetes: A Consensus Algorithm for the Initiation and Adjustment of Therapy: Update regarding thiazolidinediones: a consensus statement from the American Diabetes Association and the EuropeanAssociation for the Study of Diabetes. Diabetes Care 2008;31:173–175.

Item 82 Answer B

There have been meta-analyses showing a 2-fold increase in the risk of fluid retention and heart failure in patientswith type 2 diabetes compared to similar patients on a regimen not containing thiazolidinedione.

1. Nathan DM, et al: Management of Hyperglycemia in Type 2 Diabetes: A Consensus Algorithm for the Initiation and Adjustment of Therapy: Update regarding thiazolidinediones: a consensus statement from the American Diabetes Association and the EuropeanAssociation for the Study of Diabetes. Diabetes Care 2008;31:173–175.

2. Lago RM, et al: Congestive heart failure and cardiovascular death in patients with prediabetes and type 2 diabetes given thiazolidine-diones: a metaanalysis of randomized clinical trials. Lancet 2007;370:1129–1136.

3. Singh S, et al: Thiazolidinediones and heart failure: a teleoanalysis. Diabetes Care 2007;30:2248–2253.

Item 83 Answer A

Thiazolidinedione (TZD) use has recently been linked to bone loss and increased risk for fracture, especially inwomen. However, the majority of these fractures were not at typical osteoporosis sites, rather they were in the distalupper arm (forearm and below) and lower limb (below the knee).

TZD use is associated with a change in fat distribution from visceral to subcutaneous location. There is noincreased risk of hypoglycemia with TZDs. TZDs are associated with weight gain when used in monotherapy but the weight gain can be even greater when used in combination with insulin.

Bibliography1. Schwartz AV, et al: Thiazolidinedione use and bone loss in older diabetic adults. J Clin Endocrinol Metab 2006;91:3349–3354.2. Kahn SE, et al: ADOPT Study Group: Glycemic durability of rosiglitazone, metformin, or glyburide monotherapy.

N Engl J Med 2006;355:2427–2443.3. Nathan DM, et al: Management of Hyperglycemia in Type 2 Diabetes: A Consensus Algorithm for the Initiation and Adjustment

of Therapy: Update regarding thiazolidinediones: a consensus statement from the American Diabetes Association and the EuropeanAssociation for the Study of Diabetes. Diabetes Care 2008;31:173–175.

Item 84 Answer E

DPP-IV inhibitors are currently approved for use in monotherapy and in combination with metformin and pioglitazone. They were also recently approved for utilization with a sulfonylurea (glimepiride) or with metforminand sulfonylurea. The main caution is an increased risk of hypoglycemia when sitagliptin is used with glimepiride (12% in combination group vs 2% in placebo group).

Page 87: ADASAP2 Reduced PDF

85

DPP-IV inhibitors are currently not approved for use in combination with the GLP-1 agonist exenatide. Thepremise upon which both of these treatments are based is that GLP-1 levels are low in patients with type 2 diabetesand restoration of normal or even supranormal GLP-1 levels are beneficial. GLP-1 acts to improve hyperglycemia by increasing glucose-stimulated insulin secretion, suppressing inappropriate glucagon secretion and slowinggastric emptying. Currently, there is no evidence to show that DPP-IV inhibitors and GLP-1 agonist would haveadditive effects.

Bibliography1. Deacon CF. Incretin-based treatment of type 2 diabetes: glucagon-like peptide-1 receptor agonists and dipeptidyl peptidase-4 inhibitors.

Diabetes, Obesity and Metabolism 2007;9:(s1)23–31.2. Hermansen K, et al: Efficacy and safety of the dipeptidyl peptidase-4 inhibitor, sitagliptin, in patients with type 2 diabetes mellitus

inadequately controlled on glimepiride alone or on glimepiride and metformin. Diabetes, Obesity and Metabolism 2007;9:733–745.

Item 85 Answer B

The highest risk of hypoglycemia is with a combination of exenatide at 10 mcg b.i.d. dose in combination withmetformin and sulfonylurea. In the large initial trial with exenatide, the rate of hypoglycemia was 27.8% in themetformin + sulfonylurea group that received exenatide versus 12.6% in the group that received metformin +sulfonylurea with placebo. This is higher than the observed rate of hypoglycemia in the trial evaluating sitagliptinwith glimepride, where the incidence of hypoglycemia was in the range of 12%.

Bibliography1. Hermansen K, et al: Efficacy and safety of the dipeptidyl peptidase-4 inhibitor, sitagliptin, in patients with type 2 diabetes mellitus inade-

quately controlled on glimepiride alone or on glimepiride and metformin. Diabetes, Obesity and Metabolism 2007;9:733–745.2. Kendall DM, et al: Effects of Exenatide (Exendin-4) on Glycemic Control Over 30 Weeks in Patients With Type 2 Diabetes Treated

With Metformin and a Sulfonylurea. Diabetes Care 2005;28:1083–1091.

Item 86 Answer A

Exenatide is not recommended in patients with significant renal insufficiency (creatinine clearance less than 30 ml/min) or in patients with significant gastroparesis (since it slows gastric emptying). Exenatide can causeweight loss but there was more weight loss in patients that were obese. There is no specific contraindication to its use in patients with normal weight. Currently, exenatide is believed to be safe in patients with mild hepaticdysfunction. Use of exenatide in patients already on insulin has not been studied but there is no contraindication.

Bibliography1. Kendall DM, et al: Effects of Exenatide (Exendin-4) on Glycemic Control Over 30 Weeks in Patients With Type 2 Diabetes Treated

With Metformin and a Sulfonylurea. Diabetes Care 2005;28:1083–1091.

Item 87 Answer C

Addition of sitagliptin to metformin monotherapy in patients with mean A1C of 8% decreased the A1C by anaverage of 0.65% in the initial study evaluating this combination. Meta-analysis showed an average A1C reductionof 0.74%.

Bibliography1. Charbonnel B, et al: Efficacy and Safety of the Dipeptidyl Peptidase-4 Inhibitor Sitagliptin Added to Ongoing Metformin Therapy

in Patients With Type 2 Diabetes Inadequately Controlled With Metformin Alone. Diabetes Care 29:2638–2643.2. Amori RE, et al: Efficacy and Safety of Incretin Therapy in Type 2 Diabetes: Systematic Review and Meta-analysis.

JAMA 2007 July 11;298(2):194–206.

Educational (Learning) Critiques American Diabetes Association Self-Assessment Program

Page 88: ADASAP2 Reduced PDF

American Diabetes Association Self-Assessment Program Educational (Learning) Critiques86

Item 88 Answer C

In a study of 552 patients suboptimally controlled on metformin and sulfonylurea, the addition of either exenatideor glargine was able to reduce A1C in a similar range of 1.11%. The rates of symptomatic hypoglycemia weresimilar in both groups. GI side effects such as nausea, vomiting, and diarrhea were more likely in the exenatidegroup, especially nausea (by 7-fold). Patients started on exenatide lost an average of 2.3 kg at the end of 26 weekscompared to 1.8 kg weight gain with glargine. Glargine reduced fasting glucose more effectively while exenatide ismore effective in reducing postprandial glucose.

Bibliography1. Heine RJ, et al: Exenatide versus Insulin Glargine in Patients with Suboptimally Controlled Type 2 Diabetes.

Annals Internal Medicine 2005;143:559–569.

Item 89 Answer C

In an analysis of patients on exenatide in combination with metformin and/or sulfonylurea over 82 weeks, theaverage decrease in A1C was close to 1% with a progressive reduction in weight over time, averaging 3.5–4.4 kg at 82 weeks. About a quarter of patients had more significant weight loss, in the neighborhood of 11.9 kg.Generally, those with the greatest weight loss had the greatest reduction in A1C.

Bibliography1. Blonde L, et al: Interim analysis of the effects of exenatide treatment on A1C, weight and cardiovascular risk factors over 82 weeks

in 314 overweight patients with type 2 diabetes. Diabetes, Obesity and Metabolism 2006;8(4):436–447

Item 90 Answer B

Given her renal insufficiency and recent hospitalization, she is clearly at increased risk for hypoglycemia withsulfonylurea treatment and this would not be a good choice for her. Given that her A1C is above the ADA goal of 7%, some treatment should be intitiated if she is optimized with diet and exercise. Metformin is contraindicatedgiven her renal failure. Basal insulin is an option but with a fasting glucose in the low hundreds, this may againpredispose her to hypoglycemia. Sitagliptin would be effective given that her A1C is only mildly elevated and it has the benefit of no increased risk of hypoglycemia when used as monotherapy.

Bibliography1. Raz I, et al: Efficacy and safety of the dipeptidyl peptidase-4 inhibitor sitagliptin as monotherapy in patients with type 2 diabetes

mellitus. Diabetologia 2006;49(11):2564–2571.

Item 91 Answer B

Pramlintide is currently approved for use in patients with type 1 and type 2 diabetes. It is a synthetic analog ofamylin, a peptide normally produced by the pancreas. Effects of amylin include inhibition of glucagon secretion,delayed gastric emptying and increased satiety. It is injected prior to each meal as an adjunct to insulin treatment.Average reduction in A1C has been close to 0.6%. It has been associated with very mild weight loss, in the rangeof 1 to 1.4 kg.

Bibliography1. Hollander PA, et al: Pramlintide as an Adjunct to Insulin Therapy Improves Long-Term Glycemic and Weight Control in Patients

With Type 2 Diabetes: A 1-year randomized controlled trial. Diabetes Care 26:784–790.2. Schmitz O, et al: Amylin Agonists: A Novel Approach in the Treatment of Diabetes. Diabetes 2004;53:S233–238.

Page 89: ADASAP2 Reduced PDF

87© 2008 Professional Evaluation, Inc. All rights reserved.

Professional Evaluation, Inc.

Answer Sheet & Program EvaluationADA-SAP™

� Module 2 Pharmacological Treatment of Hyperglycemia

� General DirectionsIn oder to receive a CME/CE certificate of participation you must return this completed Answer Sheet & ProgramEvaluation form. Your completed Answer Sheet & Program Evaluation form must be returned to Professional Evaluation, Inc. (PEI) in the accompanying postage-paid envelope.

� Personal & Practice Information (Please type or print)

Name (First) (M.I.) (Last)

Address

City State ZIP

Phone eMail (required—upon completion certificate will be emailed)

Care

fully

fold

bac

k an

d cr

ease

thes

e fo

ur p

ages

bef

ore

rem

oval

from

boo

klet

.

Please check which credit you intend to apply for:� M.D.� DO � PA � FP (AAFP credit) � RN � NP (AANP credit) � RD � RPh/Pharm.D.� Other Health Care Professional

What is your professionaldegree? (Check all that apply)� M.D.� M.D./Ph.D.� M.D./MPH� DO� PA� NP/ANP� RPh/Pharm.D.� Ph.D.� RD� Other

Are you a Certified Diabetes Educator?� Yes� No

Which term best describes thehealthcare system where youpractice? (Choose one)� Health Maintenance Organization� Fee-For-Service� Hospital� Government� University� Individual/Group

Practice Association

Approximately what percentage of your patients have diabetes?� 1–5%� 6–10%� 11–20%� More than 20%

Primarily what age group of people with diabetes do you treat?� Younger than 18� 19–25� 26–45� Older than 46

How many years have you worked in your currentprofession?� Less than 1 year� 1–2 years� 3–5 years� 5–10 years� More than 10 years

What is your primary area of practice? (Check one)� Adult Endocrinology� Basic Science� Dietetics� Epidemiology� Family Practice� Geriatrics� Internal Medicine� Nephrology� Nursing� Obstetrics/Gynecology� Opthalmology� Pediatrics� Pediatric Endocrinology� Pharmacy� Podiatry� Psychology� Public Health� Other

Your gender (Optional)� Male� Female

Your age (Optional)� 25–34� 35–44� 45–54� 55–64� 65–74� Over 75

Your race/ethnicity (Optional)� African American� Asian� Caucasian� Hispanic/Latino� Native American� Pacific Islander� Other

Page 90: ADASAP2 Reduced PDF

© 2008 Professional Evaluation, Inc. All rights reserved.88

� Directions for Recording AnswersUse a black lead pencil (No. 2 or softer).

Make each mark heavy enough to completely obliterate the letter within the oval. Marks should fill the oval completely.

Erase clearly any answers you wish to change.

Make no stray marks on this answer sheet.

A B D E

A B C D E

Correct Marks

Incorrect Marks

Multiple Choice Answer Sample

True False Answer Sample

A AT F

B B

C C

D D

E E

1

A B C D E A A T F

B B

C C

D D

E E

1

A A T F

B B

C C

D D

E E

6

A A T F

B B

C C

D D

E E

11

A A T F

B B

C C

D D

E E

2

A A T F

B B

C C

D D

E E

7

A A T F

B B

C C

D D

E E

12

A A T F

B B

C C

D D

E E

3

A A T F

B B

C C

D D

E E

8

A A T F

B B

C C

D D

E E

13

A A T F

B B

C C

D D

E E

4

A A T F

B B

C C

D D

E E

9

A A T F

B B

C C

D D

E E

14

A A T F

B B

C C

D D

E E

5

A A T F

B B

C C

D D

E E

10

A A T F

B B

C C

D D

E E

15

Section A Section B

16

A B C D E 17

A B C D E 18

A B C D E 19

A B C D E 20

A B C D E 21

A B C D E 22

A B C D E 23

A B C D E 24

A B C D E 25

A B C D E 26

A B C D E 27

A B C D E 28

A B C D E 29

A B C D E 30

A B C D E 31

A B C D E 32

A B C D E 33

A B C D E 34

A B C D E 35

A B C D E 36

A B C D E 37

A B C D E 38

A B C D E 39

A B C D E 40

A B C D E 41

A B C D E 42

A B C D E 43

A B C D E 44

A B C D E 45

A B C D E 46

A B C D E 47

A B C D E 48

A B C D E 49

A B C D E 50

A B C D E 51

A B C D E 52

A B C D E 53

A B C D E 54

A B C D E 55

A B C D E 56

A B C D E 57

A B C D E 58

A B C D E 59

A B C D E 60

A B C D E 61

A B C D E 62

A B C D E 63

A B C D E 64

A B C D E 65

A B C D E 66

A B C D E 67

A B C D E 68

A B C D E 69

A B C D E 70

A B C D E 71

A B C D E 72

A B C D E 73

A B C D E 74

A B C D E 75

A B C D E 76

A B C D E 77

A B C D E 78

A B C D E 79

A B C D E 80

A B C D E 81

A B C D E 82

A B C D E 83

A B C D E 84

A B C D E 85

A B C D E 86

A B C D E 87

A B C D E 88

A B C D E 89

A B C D E 90

A B C D E 91

� ADA-SAP 2 Answer Sheet

Page 91: ADASAP2 Reduced PDF

89

Program EvaluationPlease complete this evaluation to receive your CME/CE credit.

1. After completing this activity, please rate your ability to:

Interpret and utilize ADA Guidelines and Treatment Algorithms.Outstanding Above Average Average Below Average Poor

Incorporate combination therapy for management of patients with advanced type 2 diabetes and achieve A1C targets.Outstanding Above Average Average Below Average Poor

Utilize insulin for management of patients with acute hyperglycemia.Outstanding Above Average Average Below Average Poor

Manage medication-induced hypoglycemia.Outstanding Above Average Average Below Average Poor

Regulate secondary disorders in patients with severe hyperglycemia, such as those with chronic renal insufficiency, congestive heart failure, and hepatic dysfunction.

Outstanding Above Average Average Below Average Poor

2. How many hours of CME/CE are you claiming? (Claim only the number of hours which you actually participated.)3 hours 4 hours 4.5 hours 5 hours

Please describe how this content did or did not meet these objectives.

ADA strives to present activities that are independent, balanced, objective, and scientifically rigorous.

Do you believe this activity met this objective? � Yes � No

Please explain your response.

ADA strives to utilize educational formats that are appropriate and adequate to the content and method of delivery of the program and that areappropriately equipped for personal and educational needs.

Do you believe ADA has met this objective? � Yes � No

Please explain your response.

Educational Activity ElementsPlace an “X” in the appropriate box to provide your feedback on the educational activity.

Strongly Disagree Neutral Agree Strongly Don’t Disagree Agree Know

The topic(s) were informative and relevant.

The format was suitable for the content.

The instructional materials were of high quality.

The length of the course was appropriate for the material presented.

DCBA

EDCBA

EDCBA

EDCBA

EDCBA

EDCBA

© 2008 Professional Evaluation, Inc. All rights reserved.

Page 92: ADASAP2 Reduced PDF

© 2008 Professional Evaluation, Inc. All rights reserved.90

Program Evaluation (continued)

Knowledge/Behavior ChangePlace an “X” in the appropriate box to indicate if you will change your practice based on your participation in this educational activity.

Strongly Disagree Neutral Agree Strongly Don’t Disagree Agree Know

My understanding of the subject matter increased.

I will incorporate what I have learned into my practice.

I will change the way I interact with my patients.

Personal SatisfactionPlace an “X” in the appropriate box to indicate your level of agreement with the following statements.

Strongly Disagree Neutral Agree Strongly Don’t Disagree Agree Know

My personal objectives were met.

My professional development was enhanced.

My expectations were met.

I am encouraged to participate in future ADA education activities.

List three components to take back to your practice.

What topics would interest you for future clinical education programs?

Do you have any comments or suggestions you wish to make?

Completion of Answer Sheet & Program Evaluation Form1. Provide all your Personal/Practice information. Email address is required.2. Complete the Answer Sheet.3. Complete the Program Evaluation questions; and4. Add your signature, date and return to PEI in accompanying postage-paid envelope.

Signature Date

Carefully fold back and crease these four pages before removal from

booklet.

Page 93: ADASAP2 Reduced PDF
Page 94: ADASAP2 Reduced PDF

Developed and Published by: Professional Evaluation, Inc.Developing Medical Specialty Board Category 1 CME Programs for Over Three Decades

Professional Evaluation Inc.4 Midland Avenue, Suite 105, Berwyn, PA 19312-1687 | www.proevalinc.com